0% found this document useful (0 votes)
755 views287 pages

Financial Reporting

Uploaded by

Adebayo Olayinka
Copyright
© © All Rights Reserved
We take content rights seriously. If you suspect this is your content, claim it here.
Available Formats
Download as PDF, TXT or read online on Scribd
0% found this document useful (0 votes)
755 views287 pages

Financial Reporting

Uploaded by

Adebayo Olayinka
Copyright
© © All Rights Reserved
We take content rights seriously. If you suspect this is your content, claim it here.
Available Formats
Download as PDF, TXT or read online on Scribd
You are on page 1/ 287

THE INSTITUTE OF CHARTERED ACCOUNTANTS OF

NIGERIA
SKILLS LEVEL EXAMINATION - NOVEMBER 2014
FINANCIAL
REPORTING Time Allowed: 3 hours SECTION A:
COMPULSORY
QUESTION
QUESTION 1 (30 Marks)
The Trial Balance of Excellent Pic. as at 30 June 2014 is as follows:

N'000 N'000
Salaries & wages 80,000
Plant and machinery maintenance cost 44,800
Irrecoverable debt Plant and machinery at 14,700
cost Provision for receivables Provision 75,500
for depreciation: 21,220
- Plant and machinery 41,150
- Furniture Delivery van at cost Provision 37,000
for depreciation:
- Delivery Van Delivery van expenses 49,000
Purchases/Revenue 10% Loan Notes Bank
balance Furniture at cost Sundry expenses 12,000
11,970 940.000
Trade receivables/payables Goodwill
195,000 100.000
Inventories: 30/6/2013
20,900
Dividend paid to ordinary shareholders
64,000
Retained earnings
39,500
Ordinary shares of N1 each
48.000
Revaluation surplus (Furniture) 30,000
45.000
Freehold land
16,000
15,000
30,000
100,000
8,000
600,000 1.319.370
1.319.370
The following notes are relevant:

i. Inventories as at 30/6/2013:

Units Unit Cost


'000 Price (N) N'000
Commodity 1 300 20 6,000
Commodity 2 350 15 5,250
Commodity 3 475 10 4r750
16,000

The net realisable values of these commodities per unit are as follows:
Commodity 1 Commodity 2 Commodity 3
Net Realisable Value per unit N15 N20 N8
Any increase or decrease in the value of opening inventories is to be adjusted to cost of
sales.
ii. Inventories on 30 June 2014 amounted to N9,000,000
iii. Prepaid salaries and wages were N10,000,000
iv. Included in the plant and machinery maintenance cost was depreciation of N14,800,000.
v. The allowances for receivables are no longer required. The outstanding 10% loan notes
interest was paid on 30 June 2014 and this has not been accounted for. The fair value of
goods is N40,000,000 at the end of the year.
vi. The value in use of delivery van for the year 30 June 2014 is N31,000,000. The
prevailing market interest rate is 21% per annum and the Discounting Factor for this
year is 0.8264.
vii. The fair value of delivery van at an arm's length transaction as at 30 June 2014 was
N28,000,000 and the cost to sell was N2,000,000. All non-current assets were
depreciated at 10% per annum on reducing balance basis.
viii. Current tax provision for the year is N165,000,000.
Required:
a. Identify any FOUR of the cost items that are EXCLUDED in the valuation of
inventories under IAS 2. (4 Marks)
b. Calculate the following:

(i) Value of opening inventories to be included in the Statement of Profit or Loss and
Other Comprehensive 1ncome. (2 Marks)
(ii) The present value in the use of delivery van (1 Mark)
(iii) The fair value and recoverable amount of delivery van (2 Marks)

(iv) The carrying amount and impairment if any on delivery van (2 Marks)

c. Prepare the Statement of Profit or Loss and Other Comprehensive Income (OC1) and
Statement of Changes in Equity for the year ended 30 June 2014.
(11 Marks)
d. Prepare the Statement of Financial Position as at 30 June 2014. (8 Marks)
Show all relevant workings (Total 30 Marks)

SECTION B: ATTEMPTTWOQUESTIONS FROM THIS SECTION (40 MARKS)


QUESTION 2
a. A non-accountant friend of yours attended a seminar for non-accounting executives on
interpretation of financial statements.
Though, he enjoyed the seminar especially the aspect on the uses of accounting ratios,
he strongly believes that they have their limitations.
State and explain the limitations of ratios for the purpose of interpreting financial
statements. (5 Marks)
b. You are provided with the following set of amended published Financial Statements of
HAMMED Plc for the year ended 31 December 2013:
Consolidated Statement of Profit or Loss and Other Comprehensive Income
2013 2012
N'000 N'000
Revenue 257,520 254,444
Cost of sales (126.796) (127.458)
Gross Profit 130,724 126,986
Net operating expenses (97.910) (95.728)
32,814 31,258
Net finance income 1,948 1,316
Net profit before tax 34,762 32,574
Income tax expense (10,256) (9,170)
Profit for the period 24.506 23.404
Dividend per share paid 58.50k 52.50k
Total dividend paid (N'000) 11,256 10,216
Dividend per share proposed 44k 39k
Total dividend proposed (N'000) 8,564 7,590
Basic earnings per share 130.12k 125.22k
Consolidated Statement of Financial Position
2013 2012
N'000 N'000
Non-current assets:
Property, plant & equipment (PPE) 69,864 74,630
Motor Vehicles 11.554 11.200
81.418 85.830
Current assets:
Inventories 16,548 18,344
Trade and other receivables 59,092 41,982
Cash at bank 62,824 69,916
Assets classified as held for sale 1,874 -
140,338 130,242
Total assets: 221.756 216.072

Current liabilities 48,090 49,432


Non-current liabilities 43,778 45,276
Capital and Reserves Attributable to
equity shareholders:
Called-up share capital 9,730 9,730
Share premium account 1,810 1,810
Own shares held (8,596) (6,200)
Share option reserve 2,832 1,652
Retained earnings 124.112 114.372
129,888 121,364
221.756 216.072

Additional information
i. The issued share capital of the company consists of 50k ordinary shares.
ii. The market price of the ordinary shares was N17 at 31 December 2012
and N19.16 at 31 December 2013.
iii. There were no preference shares and no loan notes.
iv. The cost of purchases plus production cost was N124,966,000 in 2012
and N125,000,000 in 2013.
v. Other opening and closing balances:

Closing Closing Opening


2013 2012 2012
N'000 N'000 N'000
PPE accumulated depreciation 37,046 129,540 122,288
Inventories 16,548 18,344 20,836
Trade receivables 40,486 37,160 35,678
Trade payables 9,604 12,882 11,412
Other taxes and social security 3,822 3,640 3,818
Accruals 30,740 27,810 27,680
Equity 129,888 121,364 106,274

Required:
i. Calculate performance (efficiency) and investment ratios for each of the
two years as far as the available information permits. (10 Marks)
ii. Comment on the company's financial performance for the year ended 31
December 2013 based on the ratios. (5 Marks)
(Total 20 Marks)
QUESTION 3
USMAN Plc.
Statement of Profit or Loss and Other Comprehensive Income for the year ended
31 December 2013
N'000 N'000
Revenue 1,600,000
Interest income 10,000
Gain on sale of plant 8,000
1,618,000
Expenses:
Cost of sales 960,000
Wages and salaries expenses 240,000
Depreciation on plant and equipment 50,000
Interest expense 8,000
Other expenses 152.000 (1.410.000)
Profit before tax 208,000
Income tax expense (60.000)
Profit for the year 148,000
Other comprehensive income:
Gain on available for sale Investment 4,000
Income tax (1,200) 2,800
Total comprehensive income for the year 150.800
USMAN Plc.
Comparative Statements of Financial Position as at:
31 Dec. 31 Dec. Increase/
2013 2012 (Decrease)
N'000 N'000 N'000
Cash at bank 113,100 120,000 (6,900)
Accounts receivable 158,000 140,000 18,000
Inventory 140,000 130,000 10,000
Prepayments 19,000 16,000 3,000
Interest receivable 200 300 (100)
Plant and equipment 330,000 300,000 30,000
Investment (Available for sale) 28,000 24,000 4,000
Intangible assets 30,000 - 30,000
818.300 730.300 88.000
Accounts payable 90,000 84,000 6,000
Wages and salaries payable 10,000 8,000 2,000
Accrued interests 400 - 400
Other expenses payable 3,600 6,000 (2,400)
Current tax payable 32,000 28,000 4,000
Deferred tax liability 17,200 10,000 7,200
Long-term borrowings 140,000 120,000 20,000
Share capital 400,000 400,000 -
Retained earnings 122,300 74,300 48,000
Available for sale reserve 2,800 - 2,800
818.300 730.300 88.000

Additional information extracted from the company's records are:


(i) Plant which had a carrying amount of N20,000,000 was sold for N28,000,000 cash and
new equipment was purchased for N100million.
(ii) Intangibles valued at N30,000,000 were acquired for cash.
(iii) Borrowings of N20,000,000 were made during the year and received in cash.
(iv) Dividends paid in cash amounted to N100,000,000.
Required:
Prepare Statement of Cash Flows for USMAN Plc for the year ended 31 December 2013 in
accordance with 1AS7 using direct method. (20 Marks)

QUESTION 4
a. International Financial Reporting Standards (1FRS) through the International
Accounting Standard Board (1ASB) sets out the definition and essential characteristics
of assets and liabilities in the presentation of financial statements which users of the
statements are likely to rely on when making major economic decisions.
Required:
Identify the essential characteristics of assets and comment on the features of liabilities
in accordance with provisions of 1AS1 on presentation of financial statements. (10
Marks)
b. 1sland Plc is an international airline which operates in Nigeria. The entity plans to enter
into a lease agreement with KLM Leasing Limited for the use of a
Boeing 747 aircraft. This agreement should not involve sale and lease back transactions.
The contract terms include the following:
■ Lease period - 5 years
■ Quarterly lease rental payments - N150 million.
■ Cost of buying Boeing 747 Aircraft - N500 million.
■ Economic useful life - 20 years
■ Scrap value after 20 years - nil
■ KLM Limited maintains the aeroplane
Required:
i. Recommend the type of lease arrangement that 1sland Plc should enter
into giving reasons for your choice. (2 Marks)
ii. 1llustrate, in tabular form, the basic differences between the type of lease
recommended in b(i) above and any other type of lease arrangements under IAS
17. (5 Marks)
c. Based on the information supplied in the question, calculate the following:
i. Total lease rental over the lease period. (1 Mark)
ii. Finance charge. (2 Marks)
(Total 20 Marks)
SECTION C: ATTEMPTTWO QUESTIONSFROMTHIS SECTION (30 MARKS)
QUESTION 5
1n accordance with 1AS 23, Borrowing Costs that are directly attributableto the acquisition,
construction or production of a qualifying asset form part of the cost of that asset while other
borrowing costs are recognised as an expense.
Required:
a. State the conditions wherein capitalisation of borrowing costs:
i. Commence
ii. Should not be suspended

iii. Should cease (6 Marks)

b. VITAMAX Plc. is constructing a factory that will take about 18 months to


complete. The company commenced construction on 2 January 2013.
The following payments were made during the year:
N'000

31 40.000
January 90.000
31 March 20.000
30 June 40.000
31 October 50.000
The30first payment on 31 January was funded from the company's pool of debts.
November
However, the company succeeded in raising Medium-Term Loan Notes for an amount
of N160,000,000 on 31 March 2013 at a simple interest rate of 9 percent per year,
calculated and payable monthly in arrears. These funds were specifically used for the
construction. Excess funds were temporarily invested at 6 percent monthly in arrears
and payable in cash. The pool of debts were again used for a N40,000,000 payment on
30 November 2013 which could not be funded from the Medium-Term Loan Notes.
The construction project was temporarily halted for three weeks in May 2013 when
substantial technical and administrative work was carried out.
The following amounts of debts were outstanding at the reporting date of 31 December
2013:

N'000
Medium-Term Loan Notes 160,00
Bank overdraft 0
10% 7-year Notes 1 October 2018 with simple interest 1,800,000
240,00
payable annually at 31 December 0
For the bank overdraft, the weighted average amount outstanding during the year was
N150,000,000 and the total interest charged by the bank amounted to N6,760,000 for
the year.
Required:
Calculate the total amount of interest to be capitalised. (9 Marks)
Note: All workings should be to the nearest thousand naira.
(Total 15 Marks)
QUESTION 6

Skelewu Nigeria Limited owns the following Property, Plant and Equipment as at 31 December
2011.
Cost Accumulated Carrying
Depreciation Amount
N'000 N'000 N'000
Plant & Machinery 45,000 9,000 36,000
Land 25,000 - 25,000
Office Buildings 75,000 15,000 60,000

Additional pieces of information are:


(i) Plant and Machinery are depreciated on a straight-line basis over 5 years. The plant &
machinery was acquired on 1 January 2011.
(ii) Land is not depreciated
(iii) Buildings are depreciated on a straight-line basis over 25 years.
(iv) Depreciation on office building is not deductible for tax purposes but for the
plant and machinery; tax deductible is granted over a period of 3 years in the
ratio 50:30:20 percent of cost consecutively.
(v) The accounting profit before tax amounted to N15,000,000 for the 2012 financial year
and N20,000,000 for year 2013. These figures include non- taxable revenue of
N4,000,000 in year 2012 and N5,000,000 in year 2013.
(vi) Skelewu Nig. Ltd had a tax loss on 31 December 2011 of N12,500,000. The tax rate for
year 2011 was 35% and 30% for each of years 2012 and 2013.
Required:
a. In accordance with IAS 12 on Income Taxes, differentiate between Current Tax
and Deferred Tax. (2 Marks)
b. Prepare the Deferred Tax Account for the year ended 31 December 2013.
(10 Marks)
c. Advise the Directors of Skelewu Nigeria Limited on the reasons why it is
necessary to recognise or make provision for Deferred Tax in the company's Financial
Statements. (3 Marks)
(Total 15 Marks)

QUESTION 7
a. IAS 16 covers all aspects of accounting for Property, Plant and Equipment (PPE)
including its measurement and qualification for recognition as an asset. The standard
also described the elements of cost, stating that some costs are directly attributable costs
of PPE while some other costs failed to qualify as costs of an item of PPE.
Required:
In the context of IAS 16, identify the elements of cost of an item of Property, Plant and
Equipment, giving SIX examples of directly attributable costs.
(5 Marks)
b. The following details are extracted from the non-current assets register of Kwali
Nigeria Plc at the year ended 30 September 2013:

N'000
Freehold property at cost 586,700
Leasehold property at valuation 30 Sept. 2012 229,500
Construction in progress 355,800
Plant and equipment at cost 198,600
Plant and equipment (Leased) at cost 85,200
Accumulated depreciation 30 Sept. 2012:
Freehold property 264,015
Plant and equipment 86,888
Plant and equipment (Leased) 21,300
Additional information:

(i) During the year ended 30 September 2013, the company incurred the sum of
N106,000,000 on the construction work in progress and this resulted in the
completion of a warehouse costing N325,000,000. The warehouse was put to use
on 1 June, 2013. The freehold property is depreciated at a flat rate of 15% per
annum on a straight-line basis.
(ii) The leasehold property was acquired on 1 October 2011 on 15 years lease at a
cost of N300,000,000. The company's policy is to revalue the property at market
value at each year end. At 30 September 2013, the property was valued at
N204,600,000.
(iii) Plant acquired is depreciated at 25% per annum using the reducing balance
method while the leased plant is also depreciated at 25% using the straight-line
method.
(iv) One item of plant acquired for N48,000,000 on 1 October 2010 was disposed on
30 September, 2013 for N36,000,000 while a new plant with a higher capacity
was acquired as a replacement for N65,000,000 on the same date.
(v) All the additional pieces of information above are yet to be adjusted for in the
books of Kwali Nigeria Plc.
Required:
Prepare a statement of changes in Property, Plant and Equipment for inclusion in the
Financial Statements for the year ended 30 September 2013. (10 Marks)
(Total 15 Marks)
SOLUTION 1
(a) Excluded costs in IAS 2 on inventories include the following:
• Abnormal amount of wasted materials, labour or other production cost.
• Storage costs, unless those costs that are necessary in the production process
before a further production stage.
• Administrative overheads that do not contribute to bringing
inventories to their present location or condition.
• Selling costs.
• Interest cost/charges that may arise when inventory is bought on terms that
allow for settlements at a later date.
• Foreign exchange differences that may arise when inventory bought
is payable to agent for arranging imported inventories.

• After sales warranty costs.


• Borrowing costs not qualified for capitalization under IAS 23.

Inventory
Commodity (a) Units Unit cost N (b) Value (lower
of a or b)
N'000 '000 N'000 N'000
1 6,000 300 15 4,500 4,500
2 5,250 350 15 7,000 5,250
3 4r750 475 8 3r800 3r800
16.000 15.300 13.550

ii. The Present Value in use remains at N31,000,000


iii Fair value of delivery van = N28,000,000
Recoverable amount is N31million being the higher of
Fair value less cost to sell (N28million - N2million)
N000
Value in use (PV of future CF)
26,000
iv. The carrying amount as at 30/6/2014 N000
31.000
Cost
N000
Less Accum. Depreciation at 30/6/2013 12,000
49.000
Depreciation for 2014 is (10% of ( N49,000 - N12,000) 3.700
(15.700)
Impairment loss
33,300
Carrying amount after impairment as at 30/6/2014
Impairment loss is excess of: (2.300)
Carrying amount (before possible impairment) as at 30/6/2014 31.000
Recoverable amount

N000
33,300
31,000
2.300
c.
EXCELLENT PLC
Statement of Profit or Loss and Other Comprehensive Income for the year ended
30 June 2014
N000
Revenue 940,000
Cost of sale (w1) (199.550)
Gross profit 740,450
Personnel cost (N80million - N10million prepayments) (70,000)
Depreciation, amortization &impairment charges (w2) (23,500)
Admin, and other operating expenses (w3) (74,950)
Operating profit 572.000
Finance costs (10.000)
Profit Before Tax (PBT) 562,000
Income tax expenses (165,000)
397.000,
EXCELLENT PLC.
Statement of Changes in Equity for the year ended 30 June 2014
Ordinary Share Retained Revaluation Total
Capital Earnings Surplus
N'000 N'000 N'000 N'000
B/fwd 100,000 30,000 8,000 138,000
Prior year Adjustment (2.450) - (2.450)
Restated B/fwd 100,000 27,550 8,000 135,550
Profit for the year - 397,000 - 397,000
Dividend paid - (15,000) - (15,000)
100.000 409.550 8.000 517.550

WORKINGS: N'000
1. Cost of sales:
Opening inventories as adjusted 13,550
Purchases 195.000
Cost of goods available for sale 208.550
Closing inventories Cost of sales (9.000)
199.550

2. Depreciation, Amortisation & Impairment Charges


Depreciation: Plant & Machinery 14,800
Delivery van 3,700
Furniture (10% x (N64m -N37m) 2,70
Impairment charge for delivery van 0
Admin Expenses/Other operating: 2.30
0
3. Maintenance Cost-Plant (N44,800 -N14,800) 30,000
Irrecoverable debts 14,700
Reversal of impairments on receivables (21,220)
Delivery van expenses 11,970
Sundry expenses 39,500
74.950
EXCELLENT PLC Statement of Financial Position as at 30 June 2014
ASSET N'000
Non-Current Assets:
Property Plant & Equipment (W1) 689,650
Intangible Asset: Goodwill 45,000
734.650

Current Assets:
Inventories (lower of N9m and N40m) 9,000
Trade and other receivables 48,000
Cash & Cash equivalents (N20.9m-N 10m) 10,900
Other Assets (i.e prepaid salary) 10,000
77.900
Total Assets 812.550

EQUITY & LIABILITIES N'000


Share Capital & Reserves:
Ordinary Share Capital (N1 per share) 100,000
Retained Earnings (N30 million- N2.45m- N15m + N397m) 409,550
Revaluation Reserve 8,000

Non -Current Liabilities:


10% Loan notes .100,000
Current Liabilities:
Trade and other payables 30,000
Current Tax payable 165.000
195/000
Total Equity and Liabilities 0 812.550
WORKINGS:

1. Statement of Changes in Property, Plant and Equipment


Cost Acc Depr. Acc Impair. Carrying Amount
N'000 N'000 N'000 N'000
Freehold Land 600,000 - - 600,000
Plant & Machinery 75,500 (41,150) - 34,350
Furniture 64,000 (39,700) - 24,300
Delivery van 49,000 (15,700) (2,300) 31,000
788.500 (96.550) (2.300) 689.650

EXAMINER'S REPORT

The question tests candidates' understanding of the provisions of 1AS2 on inventory, IAS 36 on
impairment of assets and their application to the preparation and presentation of published
financial statements. It specifically requires candidates to value the opening inventory,
determine impairment, if any, on the delivery van and to prepare the Statement of Profit or Loss
and Other Comprehensive Income, Statement of Changes in Equity and Statement of Financial
Position.
The candidates' performance was very poor as only about 10% of the candidates obtained up to
40% of the allocated marks.
The candidates' commonest pitfalls were their lack of understanding of the provisions of the
relevant accounting standards tested and their inability to present the required financial
statements in IFRS prescribed format.
Candidates are advised to study all aspects of the preparation of published financial statements
of companies including groups. They should also acquaint themselves with all the accounting
standards stated in syllabus for the Financial Reporting paper.
SOLUTION 2

The limitations of accounting ratios include the following:


(i) Heterogeneity or Homogeneity:
A company may have various divisions operating in many different industries. This
can make it difficult to find comparative industry ratios to use for comparison
purposes.
(ii) Need to determine whether the results of the ratio analyses are consistent:
One set of ratios may indicate a problem, whereas another set may indicate that the
potential problem is only short-term in nature.
(iii) Need to use judgment:
Although financial ratios are used to help assess the growth, political and risk profile
of a company, they cannot be used alone; the entire operation of the company must
be examined, and the external economic and industry setting in which it is operating
must be considered when interpreting financial ratios.
(iv) The use of alternative accounting methods:
Companies frequently have latitude when choosing certain accounting methods.
Ratios obtained from financial statements that employ different accounting choices
may not be comparable unless adjustments are made.
(v) Management Assumptions, Basis of Estimation and Judgment:
IFRS is principles-based; it requires significant judgement from management of
entities, this could result in material differences in the financial statements of the
entities. It can also further limit the financial analyses of financial statements with
accounting ratios unless further adjustments are made to the accounts.
(vi) Differences in Accounting Policies:-
IFRS as a principles-based financial reporting system, allows alternative treatments
of transactions via accounting policy choice, which invariably makes comparison of
company's performance via accounting ratios more difficult and less meaningful
unless alignment of the accounting policies of the entities are considered.
(vii) Use of Historical Cost Data:

Decisions about future expectations are based on historical data, which in some
cases, make it difficult and insensitive to reaching economic decision that are likely
to take into consideration changes in underlying variables that determine company's
performance.
(viii) Inflation can distort the financial statements:
Inflation can distort the financial statements (particularly the statement of financial
position). Any problem in the financials caused by inflation can be passed on to
ratios.
(ix) Difference in ratio definitions may make it difficult to compare ratios from different
sources:
Differences in ratio definitions make it difficult to compare ratios from different
sources. There can be many different ways to compute the same ratio. This can
cause confusion or different answers.
(x) Use of Industrial average:
Comparison against industry average may not be subjected to factors that are not
common in the industry.
(xi) Lack of comparative figure for a new entity:
In the company's first year of trading, there will be no comparative figures, hence no
indicator to compare with.
(i) EFFICIENCY RATIOS

S/N RATIOS FORMULAE YEAR 2013 YEAR 2012

(i) Gross Profit Gross profit x 100/1 130,724 126,458


% Sales 257,520 254,444
50.76% 49.70%
(ii) OPERATIN Profit before Interest &. Tax 32,814 31,258
PROFIT
G Sales 257,520 254,444
MARGIN 12.74% 12.28%
(iii) NET Profit after Tax Sales 24,506 23,404
PROFIT 257,520 254,444
MARGIN 9.52% 9.20%
(iv) INVENTOR Cost of goods sold Average 126,796 127,458
Y Inventory 17,446 19,590
TURNOVE 7.27 6.51
(v) R
AVERAGE Average Receivables days (40,486 + 37,160) (37,160 +
COLLECTI -------- ---------------------x 365 ------------------ x365 35,678)/ 254,444
ON Credit Sales 257,520 = 52 days
(vi) PERIOD
AVERAGE Average payables days ^ 365 = 55
(9,604 + days (12,882 + 11,412)
PAYMENT Credit Purchases 12,882)/125,00 124,966
PERIOD 0 33days 35days
(vii) RETURN Profit before Interest and Tax 32,814 31,258
CAPITAL Capital Employed x100 129,8888 121,364
ON 25.26% 25.76%
EMPLOYE
D
(viii ASSET Revenue 257,520 254,444
) TURNOVE Capital Employed 129,888 121,364
R 1.98 2.10
INVESTME FORMULAE YEAR 2013 YEAR 2012
NT
(i) EARNINGS
RATIOS Earnings per share (EPS) 1.302 1.2522
YEILD (%) Market Price per Share (MPS) 19.1600 7.37%
1.98 2.10

(ii) DIVIDEND Dividend per share (Dps)^ 100 0.4400 0.3900


YIELD % Market Price per Share (Mps) 1 19.1600 17.000
2.3% 2.29%
(iii) PRICE Market Price per share (MPS) 19.16000 17.0000
EARNINGS Earnings per share (EPS) 1.3012 1.2522
RATIO 14.72% 13.58%
(iv) DIVIDEND Dividend per share (DPS) 0.4400 0.3900
PAYOUT Earnings per share (EPS) 1.3012 1.2522
33.81% 31.15%
(v) RETENTIO EPS-DPS x 100/1 0.8612 0.8622
N
EPS 1 1.302 1.2522
RATIO
66.19% 68.859%
(vi) DIVIDEND Earnings per share (EPS) 1.3012 1.2522
COVER Dividend per share (DPS) 0.4400 0.3900

2.96 3.21

ANALYSIS AND REVIEW OF THE ACCOUNTING


RATIOS PERFORMANCE (EFFICIENCY) RATIOS
The analysis of performance via the "profitability Ratio" revealed that the performance of 2013
is marginally better than the performance observed for the preceding period (2012) as every
profitability measures/ratios showed an enhanced performance, which include:
Gross Profit Margin of 51% in 2013 as against 50% in 2012, operating profit margin of 13% in
2013 from 12% in 2012, and Net Income Margin of 10% in 2013 from 9% in 2012.
All indications with respect to profitability in 2013 are clear evidence of management efficiency
with respect to efficient utilisation of its assets to generate economic benefits and enhanced
returns (even though marginal).
The analysis of performance via the "working Capital Efficiency" revealed increase in
inventory turnover from 6.51 times to 7.27 times. Furthermore, there was insignificant
difference/variation with respect to average payment period to vendors/suppliers with 2 days
shortened payment period observed with respect to the management of the entity's working
capital, there exists consistency in its short term solvency and liquidity position based on slight
improvement. The performance of the entity via its liquidity, has continued to demonstrate the
original decrease, is recorded in the return on capital employed and net asset turnover. The
return on capital employed decreased from 25.76% in 2012 to 25.26% in 2013 while the assets
turnover decreased from 2.10 times in 2012 to 1.98 times in 2013.
INVESTMENT RATIOS
The analysis of investment ratios equally demonstrated the applaudable performance of the
company with respect to meeting investors' expectations as regards its enhanced returns as
shown by the increase in Price Earnings from 13.58 in 2012) to
14.72 in 2013. Increase in dividend yield (2.30% in 2013 as compared with 2.29% in 2012),
dividend payout of 34% in 2013 from 31% in 2012. There was slight exception in performance
with respect to drop in earnings from 7.37% in 2012 to 6.79% in 2013, likewise was a further
drop in dividend cover from 3.21 times to 2.96 times. Overall, it is perceived that on an average
basis, the market expectations of the investors may have been satisfied based on the analysis
and review of the company's performance for 2013.

EXAMINER'S REPORT
The question tests candidates' knowledge of accounting ratios and the limitations of using
accounting ratios in the analysis and interpretation of Financial Statements.

Candidates are required to compute performance (efficiency) and investment ratios and to
comment on the performance of the entity based on the computed ratios.

Candidates' performance was fair as over 40% of the candidates who attempted the question
obtained up to 50% of the marks allocated.

Candidates' commonest pitfall was their inability to identify the relevant accounting ratios
which measure performance and investments of an entity. Some candidates in addition, had
problem with stating the correct formulae for computing the ratios. Candidates are therefore
advised to study all aspects of accounting ratios including its limitations.

SOLUTION 3
USMAN PLC.
STATEMENT OF CASHFLOW FOR THE YEAR ENDED 31 DECEMBER 2013

NOTES N'000 N'000


CASHFLOW FROM OPERATING
ACTIVITIES: (1) 1
Cash Receipts from customers Cash paid to ,582,000
supplier and employers Cash generated (2) 222,600 (
from operations Interest Received Interest (3) 1
1,359,400)
paid Income Tax paid 0,100
Net cash from operating activities (4) (
7,600) 175,100
(5) (
♦ 50,000) ♦
Cash flow from investing activities:
Purchase of intangibles (30,000)
Purchase of plant (100,000)
Proceeds from sale of plant 28,000
Net cash used in Investing Activities (102,000)
Cash flow from Financing Activities:
Proceeds from borrowing 20,000
Dividends paid (100.000)
Net cash used in Financing Activities (80,000)
Net Decrease in cash and Equivalents (6,900)
Cash and Cash equivalent at the beginning of the
year 120.000
Cash and cash equivalent at the end of the year 113.100
Notes:
1. Cash Receipts from customers:
N'000
Revenue 1,600,000
Beginning account receivable 140,000
1,740,000
Ending account receivable (158.000)
Cash receipts from customer 1.582.000
OR
ACCOUNT RECEIVABLES
N'000 N'000
Opening bal 140,000 Cash receipts 1,582,000
Sales Revenue 1,600,000 Closing bal 158,000
1,740,000 1.740.000

2(a) Payment to suppliers for Purchases:


N'000
Cost of sales 960,000
Increase in inventory 10.000
970,000

22
Increase in account payables (6,000)
Payment for purchases 964.00
0
(b) Payment for other services
Other expenses 152,000
Increase in prepayments 3,000
Decrease in other expenses payable 2,400
157.400
(c) Payment to employees
Wages and salaries expenses 240,000
Increase in wages and salaries payable (2.000)
238.000
(d) Summary of total payment to suppliers and
employees:
For purchases 964,000
For services 157,400
For salaries and wages 238.000
(3) Interest received: 1.359.400
Interest revenue -increase in interest receivable or decrease ii receivable =
N10.000.000 + N100.000 = N10.100.000 interest

(4) Interest paid: N'000


Interest expenses 8,000
Decrease in accrued interest 400
7.600
(5) Income Tax paid:
Beginning balance 28,000
Income tax expenses 54,000
Income tax paid (50.000)
Ending balance 32.000
(a) Deferred Tax
N'000 N'000
Balance c/d 17,200 Bal b/d 10,000
Tax on OCI 1,200
Income tax expenses 6.000
17.200 17.200
N'000
OCI = Other Comprehensive Income (b) Income Tax expenses 60,000
Less: Deferred Tax (5a) 6,000
54.000
EXAMINER'S REPORT

The question tests candidates' ability to prepare and present a statement of Cash Flows using the
direct method.
Candidates' performance was poor as about 20% of those who attempted the question obtained
up to 40% of the marks allocated.
Candidates' commonest pitfall was their inability to correctly classify the cash flows.
Candidates are advised to study the different classification of cash flows against future
examinations.

SOLUTION 4

(a) Essential characteristics of Assets.


IASB defines asset as a resource controlled by an entity as a result of past events and
from which future economic benefits are expected to flow to the entity.
The characteristics of this definition are as follows:

■ Emphasis on control rather than ownership of asset. It means ability to restrict


other entity from using the asset.
Also, it places emphasis on substance of the transactions rather than their legal
■ form.
When there is control of an asset, it is recognised in the statement of financial
position e.g. Finance lease, contractual rights.

The definition makes reference to past events, thereby excluding asset that may
■ occur in future.
Essential characteristics of liabilities:

IASB defines liability as a present obligation of the entity arising from past
event, the statement of which is expected to result in an outflow from the entity
of resources embodying economic benefits.
From this definition, the characteristics of a liability are as follows:
■ It is opposite or mirror image of asset
■ It is a present obligation
■ The obligations are legally enforceable
■ Such obligations may also arise on provisions, contigent liabilities, contigent assets e.g.
replacing faulty assets, warranty.
4(bi)
Island Plc should enter into Operating Lease arrangement for the following reasons:
• The lease period of 5 years is less than 20 years of useful life.
• The lessor KLM Ltd. maintains the asset.
• The Company will be able to get maximum tax deduction via periodic
expensing of the lease rentals
4(bii)

OPERATING LEASE FINANCE LEASE


Ownership is not transferred at the end of the Ownership is transferred at the end of the lease
lease period period
No bargain purchase option in the contract Bargain purchase option exists

The present value of the minimum lease does The present value of the minimum lease
not equal the asset fair value payment substantially equal to the asset fair
value
The asset can be leased out again after the The asset cannot be leased out again after the
expiration of the initial lease period expiration of the initial lease period (the asset
would have been used for substantial part of
the useful life)
The lessor maintains and is responsible for the The lessee maintains and up keeps the asset
upkeep of the asset
Lease rentals is expensed Lease Rental is used to reduce lease obligation
while some portion is expensed as finance cost.

No value of asset is recognised in the books of Value of asset is recognised in the books of the
the lease, and no depreciation nor impairment lessee as an asset, and depreciation and
of the asset is charged impairment of the asset are charged.

4(ci)

TOTAL LEASE RENTAL OVER THE LEASE PERIOD


Lease Rental (quarterly) is N150m
Lease period (in years) is 5
Number of payments is 20
Lease rental for 1 year is N150m x 4 = N600m
:. For Five years, the total lease rental =N600m x 5 = N3billion.
4(cii)
FINANCE CHARGE (TOTAL)
The total value of minimum lease payments =
Less: cost (fair value) of the Asset
Total Lease rentals over the 5 years lease period = N'000
EXAMINER'S REPORT m
The question tests candidates' knowledge of IAS 17 on leasing and3,000 IAS 1 provision on essential
characteristics of assets and the features of liabilities. Candidates5002.5 are also required to
differentiate between types of leases and to calculate the total lease rentals
00 and finance charge.
The candidates' performance was above average as over 70% of the candidates obtained up to
50% of the marks allocated.
The candidates' commonest pitfall was their inability to identify the essential characteristics of
assets and the features of liabilities. Some candidates failed to properly distinguish between the
types of leases.
Candidates are advised to pay special attention to accounting standards while
preparing for examinations.
SOLUTION 5
a(i)Condition that must exist before capitalisation of borrowing costs can commence
are as follows:
Expenditure on qualifying asset are being incurred.
Borrowing Costs are being incurred
Activities necessary to prepare the assets for their intended sales or use are
■ in progress
■ When Capitalisation of Borrowing Costs should not be suspended will be

When all of the components required before any part of the asset can be sold
or used though not yet completed.
(ii)
For brief interruptions in activities
During the periods when substantial technical and administrative work is

being carried out, or for delays that are inherent in the assets acquisition
process.


Capitalisation of Borrowing Costs should Cease when
the assets are materially ready for their intended use or sale, or
Construction is complete in part and the completed part can be used
independently.
(iii)


BORROWING COSTS TO BE CAPITALISED


SPECIFIC LOAN N'000
N160,000,000 X 9% X 9/12 10,800
Interest earned on unused portion of loan available during the year:- April 1
to June 30 (160,000,000 - 90,000,000) x 3/12 x 6%
1 July to 31 October (160,000,000 - 110,000,000) x 4/12 x 6% November 1
(1,050)
to November 30 (160,000,000 - 150,000,000) x 1/12 x 6%
(1,000)
(50)
8.700
GENERAL POOL OF FUND

Capitalisation rate is 9.58% (wi)


Paid 31 January (40,000,000 x 11/2 x 9.58%) 3,513
Paid 30 November (40,000,000 x y2 x 9.58%) 320
Total Amount to be capitalised Workings: 3.833
1. Interest paid on borrowings :- Bank 12.533
overdraft:.
7 year Note (1,800,000,000 x 10%)

6,760
180.000
186,760

Weight average of total borrowings:


Bank overdraft 7 year Note 150,000
1.800.000
1,950,000

Capitalisation Rate 186,760/1,950,000 = 0.0958


r

2. Note
Although no activities had been interrupted by technical administrative
work during May 2013, Capitalisation is not suspended for this period
according to IAS 23

EXAMINER'S REPORT
The question tests candidates' knowledge of IAS 23 on borrowing costs. Candidates are
required to state the conditions for capitalization of borrowing costs and to calculate the total
amount of interest to be capitalized based on the information available.
Candidates' performance was poor as only about 15% of those who attempted the question
obtained up to 40% of the marks allocated.
The candidates' commonest pitfall was their inability to state correctly when capitalization of
borrowing costs commences, should not be suspended and should cease. In addition, most of
them could not calculate the amount of interest to be capitalized.
Candidates are advised to study relevant accounting standards stipulated in the new ICAN
syllabus as testable in Financial Reporting.

SOLUTION 6
(a) Difference Between Current Tax & Deferred Tax
Current Tax: is the amount actually payable to the tax authorities in relation to the
trading activities of the entity during the period.
Deferred Tax is the amount payable to or refundable by the tax authorities in respect of
the current and previous periods on taxable profit. It is also considered as an accounting
measure used to match the tax effect of transactions with their accounting impact and
thereby produce less distorted results.

Deferred Tax Account as at 31 December 2013


N'000
1 January 2012 balance
Plant & Machinery (13,500,000 x 35%) (4,725)
Tax less carried forward (12,000,000 x 35%) 4r375
Balance 1 January 2012 (350)
Rate of change (350,000 x 5/35 ) 50
Temporary difference: Plant & Machinery (w1) (1,350)
Loss utilised (w2) 9,500,000 x 30% (2.850)
Balance 31 December 2012 (4,500)
Temporary difference plant & Machinery (wl) —
Loss utilised (2) N3,000,000 x 30% (900)
Balance 31 December 2013 (5.400)

1. Workings:
Plant
Carrying
Tax Base Temporary Deferred Tax difference N'000
Amount
N'000 N'000 N'000
1 Jan 2011 -cost 45,000 45000 - -
(22.500) 13.500
Depreciation (9.000)
31 Dec. 2011 36,000 4.725
Rate charge (5/35 x 4,725) - 22,500 13,500 4,725
Depreciation (9,000) - - (675)
(13.500) 4,500 1.350
31 December 2012 27,000
Depreciation (9.000) 9,000 18,000 5,400
31 Dec 2013 18.000 (9,000) - ____ -
_____ -.
_____ 18.000 5r400

2. Income Tax Expense


31 Dec. 2013 31Dec. 2012
N'000 N'000
Accounting profit before Tax 20,000 15,000
Tax effect of item not deductable
Non-taxable revenue (5,000) (4,000)
Depreciation on building 3.000 3.000
18,0 14,000
Depreciation Accounting 9,000 9,000
Depreciation/allowable tax (9.000) (13.500)
Taxable profit 18,000 9,500
Assessed loss b/fwd (3.000) (12,500)
Taxable profit/Loss 15.000 (3,000)
Tax loss c/fwd - (3,000)
Tax payable/benefit at 30% 4.500 (900)
c. Reasons why deferred Tax should be recognised or provided for by Skelewu
Nigeria Limited are as follows:
(i) Adjustment for deferred tax are made in accordance with the actual concept and in
accordance with the definition of a liability in the conceptual framework i.e a past
event has given rise to an obligation in the form of increased taxation which will be
payable in the future. The amount can be reliably estimated.
(ii) If the future tax consequences of transaction are not recognised or provided for,
profit can be overstated leading to overpayment of dividend and distortion of share
price and Earnings Per Share (EPS).
EXAMINER'S REPORT
The question tests the candidates' knowledge and understanding of IAS 12 on Income Taxes.
Specifically, candidates are required to differentiate between Current Tax and Deferred Tax,
prepare Deferred Tax Account and to profer reasons why it is necessary to make provision for
deferred tax in Company's Financial Statements.
Candidates' performance was very poor as only about 10% of those who attempted the question
obtained up to 30% of the marks allocated to it.
Candidates' commonest pitfall was their inability to prepare Deferred Tax Account. Candidates
are advised to study all the relevant Accounting Standards before presenting themselves for
examinations.

SOLUTION 7
(a) Elements of Cost of an item. Property, Plant and Equipment (PPE) include the

following.

(i) Its purchase price, including import duties and non-refundable purchase taxes, after
deducting trade discount and rebates.
(ii) Any costs directly attributable to bringing the asset to the location and condition
necessary for it to be capable of operating in the manner intended by management.
(iii) The initial estimate of the costs of dismantling and removing the item and restoring
the site on which it is located, the obligation for which an entity incurs, either when
the item is acquired or as a consequence of having used the item during a particular
period for purposes other than to produce inventories during that period.
Examples of attributed costs include the following:

(i) Costs of employees' benefits arising directly from the construction or acquisition of
the item of Property, Plant and Equipment.
(ii) Costs of site preparation
(iii) Initial delivery and handling costs
(iv) Installation and assembly costs
(v) Cost of testing whether the asset is functioning properly, after deducting the net
proceeds from selling any item produced while bringing the asset to that location
and condition, such as samples produced when testing equipment
(vi) Professional fees
(vii) Borrowing costs (IAS 23)
(b) Kwali Nigeria Plc.
Schedule for Property, Plant and Equipment for the year ended
30 September 2013

Freehold Lease Plant & Leased Constructio Total


property hold Equipment Plant & in progressn
Cost/Valuations property Equipment
N'000 N'000 N'000 N'000 N'000 N'000
Bal. at 1/10/2012 586,700 229,500 198,600 85,200 355,800 1,455,800
Additions - - 65,000 - 106,000 171,00
Reclassifications 325,000 - - - (325,000) 0 -
Disposals - - (48.000) - - (48.000)
Bal. At 911.700 229.500 215.600 85.200 136.800 1.578.800
Depreciation:
30/9/2013
Bal at 1/10/2012 264,015 - 86,888 21,300 - 372,203
Depreciation for the
year 104,255 16,393 27,928 21,300 - 169,87
On disposals - - (27,750) - - (27,750)
6
On revaluation - 8.507 - - - 8.507
Bal. at 30/9/13 368.270 24.900 87.066 42.600 - 522,836
Carrying amount at
30/9/2013 543.430 204.600 128.534 42.600 136.800 1.055.964

N
Workings: Calculation of Depreciation
1. Freehold property:- N586,700 x 15% = 88,005,000
Warehouse: N325,000 x 15% x 4/12 = 16.250.000
104.255.000
2. Property at Valuation 1/10/12 229,500,000
Depreciation(14 yrs remaining) (16,393,000)
Carrying amount before valuation 213,107,000
Valuation at 30/9/13 (204.600.000)
Revaluation loss 8.507.000
3. Plant and Equipment (N198,600,000-N86,888,000) x 25% = N27.928.000
N N
4. Plant & Equipment:
Accumulated Depreciation on disposal
Cost @ 1/10/2010 N48,000,000
25% Depreciation 30/9/2011 (12.000.000)

36,000,000 12,000,000 25% depreciation30/9/2012 (9.000.000)


27,000,000
9,000,000
25% depreciation 30/9/2013 6.750.000 6.750.000
Accumulated Depreciation on Plant & Equipment disposed
on 30/9/2013 27.750.000
EXAMINER'S REPORT

The question tests the candidates' knowledge and application of IAS 16 on Property, Plant and
Equipment, thus they are required to prepare the statement of changes in Property Plant and
Equipment and also to identify the elements of cost of an item of Property, Plant and
Equipment.
Candidates' performance was very poor as only about 12% of those who attempted the question
obtained up to 30% of the marks allocated.
Candidates' commonest pitfall was their inability to differentiate between elements of cost and
attributable costs of PPE. They could also not prepare the schedule of Property, Plant and
Equipment.
Candidates are therefore advised to familiarize themselves with all the relevant Accounting
Standards stipulated in the ICAN Syllabus for Financial Reporting paper.
SKILLS LEVEL EXAMINATION - NOVEMBER 2015
FINANCIAL REPORTING
Time Allowed: 3 hours
ANSWER FIVE QUESTIONS IN ALL
SECTION A: COMPULSORY (30 Marks)
QUESTION 1

Statement of financial position as at December 31, 2014


Hapu Plc Sege Plc
H000 H000
Assets
Non-current assets
Property, plant and equipment 32,000 25,000
Investments 33,500 -
65,500 25,000
Current assets
Cash at bank and in hand 9,500 2,000
Trade receivables 20,000 8,000
Inventory 30,000 18,000
125.000 53.000
Equity and liabilities
Share capital 40,000 10,000
Share premium 6,500 -
Retained earnings 55.000 37.000
101,500 47,000
Current liabilities 23.500 6,000
125.000 53.000

Statement of profit or loss for the year ended December 31, 2014
Hapu Sege
Plc Plc
N000 N000
Revenue 125,000 117,000
Cost of sales (65,000) (64,000)
Gross profit 60,000 53,000
Distribution costs (21,000) (14,000)
Administrative expenses (14,000) (8,000)
Profit before taxation 25,000 31,000
Income tax expense (10,000) (9,000)
Profit for the year 15.000 22.000
Statement of changes In equity (extract) for the year ended December 31, 2014

Hapu Sege Plc


Plc
H000 H000
Retained earnings brought forward 40,000 15,000
Retained profit for the year 15,000 22,000
Retained earnings carried forward 55.000 37.000

You are given the following additional information.


(i) Hapu Plc owns 80% of Sege's shares. These were purchased in 2011 for
N20.5million cash, when the balance on Sege's retained earnings stood at
N7million.
(ii) Included in the inventory of Sege Plc at December 31, 2014 were goods purchased
from Hapu for N3.9million. Hapu aims to earn a profit of 30% on cost. Total sales
from Hapu Plc to Sege Plc were N6million.
(iii) Hapu Plc and Sege Plc each proposed a dividend before the year end of N2million
and N2.5million respectively. No accounting entries have yet been made for these.
(iv) Hapu Plc has carried out annual impairment tests on goodwill in accordance with
IFRS 3 and IAS 36. There was no impairment of goodwill.
Required
Prepare the consolidated statement of profit or loss for the year ended December 31,2014.
Prepare the consolidated statement of financial position at that date.
(30 marks)
SECTION B: ANSWER ANY TWO OUT OF THREE QUESTIONS FROM THIS SECTION
(40 MARKS)
QUESTION 2
The following trial balance has been extracted from the books of Well-Being Plc as at March 31,
2014.
N'000 N'000

Land at cost 360


Building at cost 750
Equipment at cost 588
Vehicles at cost 852
Goodwill 900
Accumulated depreciation:
At April 1, 2013:
Buildings 270
Equipment 228
Vehicles 396
Inventory at April 1, 2013 321
Trade receivables and payables 549 351
Allowance for receivables 24
Bank balances 171
Current taxation 18
Ordinary shares of N1 each 600
Retained earnings at April 1, 2013 1,509
Revenue 4,296
Purchases 1,464
Directors' fees 450
Wages and salaries 828
General distribution costs 303
General administrative expenses 558
Dividend paid 60
Rent received 90
Disposal of vehicle ____ 30
7,983 7,983
The following information is also available:
1. The company's non-depreciable land was valued at N900,000 on March 31, 2014 and this
valuation is to be incorporated into the accounts for the year ended March 31, 2014.
2. The company's depreciation policy is as follows:
Building, 4% p.a. straight line
Equipment, 40% p.a. reducing balance Vehicles, 25% p.a. straight line
In all cases, a full year's depreciation is charged in the year of disposal. None of the assets
had been fully depreciated by March 31, 2013.
3. On February 1, 2014, a vehicle used entirely for administrative purposes was sold
for N30,000. The sale proceeds were banked and credited to a disposal account, but no
other entries were made in relation to this disposal. The vehicle had cost N132,000 in
August 31, 2010. This was the only disposal of a non-current asset made during the year
ended March 31, 2014.

4. Depreciation is apportioned as follows:

Distribution Administrative
cost expenses
Buildings 50% 50%
Equipment 25% 75%
Vehicles 70% 30%

5. The company's inventory at March 31, 2014 is valued at N357,000.


6. Trade receivables include a debt of N24,000 which is to be written off. The allowance for
receivables is to be adjusted to 4% of the receivables which remain after the debt has been
written off.
7. Current tax for the year ended March 31, 2013 was over-estimated by N18,000. The current
tax liability for the year ended March 31, 2014 is estimated to be N90,000.
8. One-quarter of wages and salaries, was paid to the distribution staff and the remaining
three-quarters were paid to the administrative staff.
9. General administrative expenses include bank overdraft interest of N27,000.
Required:
Prepare a statement of profit or loss and other comprehensive income for the year ended March 31,
2014.
(20 Marks)
QUESTION 3
The summarized final accounts of Omosigho Ltd, manufacturer of Aluminum roofing sheets and
its accessories for two years ended December 31, 2013 and 2014 were as follows:

Operating results 2014 2013


Revenue N'000 N'000 N'000 N'000
Profit before Interest and taxation 3,364,720 2,750,355
Interest payable 117,060 104,410
Taxation 12,500 12.500
Dividend 52,530 43,735
18.750 17.500

Statement of financial position:


Non-current Assets 81,110 34,705
Current Assets:
Inventories & W.I.P Receivables 311,470 260,980
& prepayments Cash and bank 274,295 250,260
balances 36,170 72,825

621,935 584,065
Current Liabilities:
Payables and accruals 268,545 241,770

Net current Assets 353.390 342.295


434.500 377.000

Financed by:
62,500,000 ordinary shares of N1 each
62,500 62,500
Retained earnings 179.370 148.935

Shareholders' funds 10% 241,870 211435


Loan notes(2020-2032) 125,000 125,000
Deferred taxation 67.630 40.565
434.500 377.000

The shares of the company were quoted at N1.20 at December 31, 2013 and December 31,
2014.
Required
a. Calculate TWO accounting ratios each that will be of interest to the following
stakeholders:
i. Creditors
ii. Management
iii. Shareholders (15 Marks)
b. Comment briefly on the changes between the ratios arrived at in 2013 and 2014.
(5 Marks) (Total
20 Marks)

QUESTION 4
The purpose of IAS 36: Impairment of Assets is to provide entities with guidance to determine
whether an asset is impaired and how the impairment should be recognised.
Required:
a. In assessing whether there is an indication that an asset may be impaired, what factors
should an entity consider?
(5 Marks)
b. The following information relates to individual plant and equipment used by Phonex
Nigeria Limited for its telecommunication operations as at December 31, 2014.

Plant and Equipment Carrying Fair Value less Value in


Amount cost to sell use

N'000 N'000 N'000


1. Mast 297,500 302,500 285,000
2. Generators 592,500 517,500 512,500
3. Computer equipment 287,500 292,500 307,500
4. Credit card machines 207,500 187,500 197,500
5. Motor vehicles 77,500 65,000 -

Additional information
i. The Mast and the Generator are carried at revalued amount and the cumulative revaluation
surplus in other comprehensive income for the equipment are N30,000,000 and
N15,000,000 respectively both equipment are field equipment.
ii. The motor vehicles are buses used for transporting employees in the morning and evening and it
is not possible to determine the value in use of the buses separately because the buses do not
generate cash inflow from continuing use that are independent of the cash flows from other assets.
Required:
Draft a memo addressed to your boss indicating whether each of the plant and equipment is
impaired or not and also explaining how the impairment loss should be treated in the books of
Phonex Nigeria Limited as at December 31, 2014.
(15 Marks)
(Total 20 Marks)

SECTION C: ANSWER ANY TWO OUT OF THREE QUESTIONS FROM THIS SECTION (30 MARKS)

QUESTION 5
The trial balance of UDO Plc and its subsidiary, ALOMA Plc as at December 31, 2014 are given
below:

UDO Plc ALOM LA Plc


DR CR DR CR
NM NM NM NM
Property, Plant & Equipment 260 6,000
Loan to ALOMA Plc 4,600 -
Other investments 1,200 -
Current assets 1,600 278
N1 equity shares 400 200
Share premium 1,000 170
Retained earnings 5,800 662
Revaluation surplus 60 -
Loan from UDO Plc - 4,600
Sundry payables 400 646
7,660 7,660 6,278 6,278

UDO Plc on January 1, 2014 acquired 75% of the equity of ALOMA Plc for N1,300,000,000,
when the retained earnings was N600million and share premium N170million. Neither this
transaction nor the loan notes for the same amount obtained to finance the purchase were recorded
in the trial balance above.
There had been no impairment in the value of the goodwill, nor a change in share capital since
acquisition.
It is the group policy to value the non-controlling interest at the date of acquisition at fair value.
The fair value of the non-controlling interest in ALOMA Plc at the date of acquisition was
estimated to be N160,000,000.

Required:
Prepare the consolidated statement of financial position of UDO Group Plc as at December 31,
2014. (15 Marks)

QUESTION 6
Global Plc is an entity quoted on the Nigerian Stock Exchange.

You are provided with the following set of summarised published financial statements of the
company for the year ended September 30, 2014.

Statement of profit or loss and other comprehensive income for the year ended September 30, 2014.

2014
N'000
Revenue 500,000
Cost of sales (300,000)
Gross profit 200,000
Administrative expenses (29,000)
Finance cost (1.000)
Profit before taxation 170,000
Income tax expense (40.000)
Profit for the period 130.000
Statement of financial position as at September 30, 2014 together with their comparative figures:

2014 2013
N'000 N'000
Assets:
Non-current assets:
Property, plant and equipment 200,000 220,000
Goodwill - 10,000
Inventories 100,000 80,000
Trade receivables 75,000 60,000
Bank balances 20.000 5.000
395,000 375,000
Equity and liabilities:
Ordinary shares @ N1.25k each 10,000 8,000
Retained earnings 250.000 197.000
260,000 205,000

Non-current liabilities:
10% loan notes 10,000 -
Current liabilities:
Trade payables 60,000 90,000
Other payables 20,000 40,000
Taxation 20,000 30,000
Bank overdrafts 25,000 10,000
135.000 170.000
Total equity and liabilities 395,000 375,000

The following information is relevant:


i. During the financial year, the company paid dividend of N87,000,000 to equity holders and
this had been accounted for during the year. The current market price of the company is
N10 per share.
ii. The company is planning to take a long term loan of N400,000,000 from consortium of
banks. The company's financial statements and loan applications have already been
submitted to the bank.
Required:
a. Prepare the company's statement of cash flows submitted to the bank in accordance with the
provisions of IAS 7.
(10 Marks)
b. Comment briefly on the cash flow management strategies of Global Plc.
(5 Marks) (Total
15 Marks)
QUESTION 7

a. The objectives of IAS 40- Investment Property is to prescribe the accounting treatment and
disclosure requirements for investment property. The main issue in accounting for
investment properties is to distinguish these properties separately from owner-occupier
properties.
Required:
Explain how treatment of an investment property carried at fair value model differs from an
owner-occupier property carried under revaluation model.
(5 Marks)
b. KOLA N1TDA Nigeria Plc is a company engaged in the manufacturing of hand sanitizer to
prevent Ebola disease. The following information relates to property owned by the
company.

N'000
Land - Plot 404 Apapa Industrial Area 32,000
Building therein (acquired June 30, 2013) 84,000
Improvement to the building to extend rented floor capacity 16,000
Repairs and maintenance to investment property for the year 2,000
Rental received for the year 6,400
Approximately six percent of the property floor space is used as the administrative head
office of the company. The property can be sold only as a complete unit. The remainder of
the building is leased out under operating leases. The company provides lessees with
security services.
The company values investment property using the fair value model on December 31, 2014
which is the company's year end. Tewogbade & Co. (an independent valuer) valued the
property at N144,000,000 on that date.
Required:
i. Advise the Directors of KOLA NITDA Nigeria Plc on how the property should be
treated in the financial statements of the company as at December 31, 2014 in order
to ensure strict compliance with provisions of IAS 40.
(5 Marks)
ii. Calculate the value of investment property that should be disclosed in the
statement of financial position as at December 31, 2014 and the amount
that should be charged to the statement of profit or loss and other
comprehensive income for the period then ended. (5 Marks)
(Total 15 Marks)
SOLUTION 1

(a)
Hapu Plc
Consolidated statement of profit or lossfor the year ended December 31.
2014
N'000
Revenue (N125,000 + N117,000 - N6,000) 236,000
Cost of sales (N65,000+-N64,000--N6,000 +N900) (123.900)
Gross profit 112,100
Distribution costs (21,000 + 14,000) (35.000)
Administrative expenses (14,000+8,000) profit (22.000)
before taxes 55,100
Income tax expenses (10,000 + 9,000) (19.000)
Profit for the year 36.100
Profit attributable to: 31,700
Owners' of the parent (bal. figure) 4.400
Non-controlling interest (20% x 22,000) 36.100

(b) Hapu PLC


Consolidated Statement of Financial Positionas at December 31. 2014
Assets N'000
Non-Current Assets Goodwill (w 1) N'000
Investments (33,500 - 20,500) 57.0
6,900
13.0
Current Assets 76,900
Inventory (w 5) 47,100
Trade Receivables 28,000
Bank and cash 11,500 86.600
Total Assets 163,500
N'000
Equity &Liabilities
40,000
Share capital
6,500
Share premium
78.100
Retained Earnings (w3)
124,600
Equity attributable to owners of the parent
9,400
Non-controlling interest (w 4)
29.500
Current Liabilities 163.500
Workings

(1) Goodwill on consolidation: N'00


Cost of investment/consideration 0
FV of NCI at acquisition (20% of N17,000,000) 20,5
00
Less Sege Plc Net Asset at acquisition: 3,40
Share Capital 10,000 0
Retained earnings 7.000 (17.000)
23,9
Goodwill 6.900
00
Hapu Plc Sege Plc
(2) Consolidated Retained earnings: N'000 N'000
Retained earnings per question 55,000 37.0 (7,0
Pre acquisition - sege - 00)
post acquisition - sege - 30.000
Groups share at 80% 2,400
Less unrealized profit in inventory(w3) (900)
78.100
(3) Calculation of unrealized profit:
.30 x N3,900,000
130 = N900,000
(4) Non-controlling Interest:
N'000
Fair value at acquisition date
3,400
Share of post-acquisition retained earnings
6,000
(20% of N30,000,000)
9.400
(5) Consolidated Inventory N'000
Hapu 30,000
Sege 18,000
Less unrealized profit (900)
47.100
(6) The dividends proposed for the year ended December 31, 2014 were Hapu Plc N2
million and Sege Plc N-2.5 million.
a. Consolidated Statement of Profit or Loss for the year ended December 31.2014
Mar
- Calculation of Gross Profit (GP)
ks
• Revenue
1
• Cost of sale
1
• GP
1
- Ascertainment of PBT:
3
• Distribution Cost
1
• Admin Exp
1
• PBT
1
- Ascertainment of Profit for the year
3
• Income Tax
1
• Profit for the year
1
- Calculation of attributable profit
2
• to parent
1
• to NCI
1
b.Consolidated Statement of Financial Position as at December 31. 2014 2
- Calculation of:
• Non-current Assets 2
• Investments 2
• Goodwill 2
- Ascertainment of Current Assets: 6
• Inventory 4
• Receivables 1
• Cash & bank 1
- Calculation of equity 6
• Share Capital 1/2
• Share Premium 1/2
• Retained earnings 4
- Calculation of non-controlling interest: 5
• Workings 1
• Final figure 1
- Stating of Current 2
Liabilities Total marks for 1
the question 30
EXAMINER‟S REPORT
The question tests candidates‟ knowledge of group accounts. Candidates are required to prepare a
Consolidated Statement of Profit or Loss and Statement of Financial Position.
Majority of the candidates attempted the question and performance was good as over 60% of them
obtained above 50% of the marks allocated.
The commonest pitfall is the inability of some candidates to present the statement in IFRS
recommended format. Some also lost marks due to wrong approximation of figures and non
indication of appropriate naira symbol.
Candidates are advised to be careful when answering examination questions so as to avoid loss of
marks earned due to failure to follow examiners‟ instructions.
SOLUTION 2

Well - Being PlcStatement of profit or loss and other comprehensive


incomefor the year ended March 31. 2014
N'000 N'000
Revenue 4,296
Cost of sales
Opening inventory 321
Purchases 1.464
1,785
Closing Inventory (357)
1.428 (1,428)
Gross Profit 2,868
Other income:
Rent received 90
Profit on disposal of vehicle 30
Decrease in allowance for receivable 3 123
2,991
Distribution costs (711)
Administrative expenses (1,812)
Finance Cost (27)
Profit before taxation 441
Income tax expense:
Provision for the year 90
Over provision 2013 (18) (72)
Profit for the year 369
Other comprehensive income:
Gain in valuation of land (N900 -N360) 540
Total comprehensive income 909
61.5 Kobo

EPS
Workings
1. Distribution costs N'000
General distribution cost 303
Wages and salaries (1/4) 207
Bad debts written off 24
Depreciation 177
711

2. Administrative Expenses N'000


General administrative expenses (558-27) 531
Wages and salaries (3/4) 621
Directors fees 450
Depreciation 210
1.812
Tot
3. Depreciation for the year: al Distributio Administrat
Building 4% of N750,000 Equipment 40% of N‟0 n ion
(588,000- 228,000) Vehicles: 00 N‟000 N‟000
Disposed 25% of N132,000 Others 30 15 15
25% of N852,000-132,000 144 36 108
33 33
180 126 54
387 177 210

Allowance for receivables: N'000


Opening balance 24
Closing balance 4% of (N549-N24) 21
Decrease in allowance for receivable 3

Profit on Disposal of vehicle: N'000 N'000


Proceed on disposal Carrying amount at 30
disposal:
Cost of purchase 132
Depreciation 2011,2012 &2013(3yrs) At (99)
25% 133
Depreciation in year of disposal 2014 ) 0
30

6. EPS = N369.000 61.5 kobo


600,0 shares
Marking Guide _____________________________________________________ Marks
1/3
- Stating the title of the financial statement
- Determination of Gross Profit GP:
1/3
• Revenue
• Cost of Sales 1 2/3
• GP
2/3 22/3
- Determination of Other Income
1/3
• Rent received
1/3
• Profit on Disposal
• Other Income 1 12/3
- Determination of profit before taxation
1/3
• DistributionCost
1/3
• Admin Exp
1/3
• Finance cost
1/3 11/3
• Profit beforetaxation
- Calculation of profit for the year
• Income Tax expense 1
1/3 11/3
• Profit for the year
- Determination of other comprehensive income
1/3
• Gain on revaluation of land
1/3
• Total Comprehensive Income
1/3
2/3
(6 ticks @ 1/3 mark each) 2
Determination of distribution costs
(5 ticks @ 1/3 mark each) 12/3
Calculation and apportionment of Depreciation (14 Ticks @ 1/3
mark each)
4
2/3

Calculation of allowance for receivables


(3 ticks @ 1/3 mark each) 1
Determination of profit on disposal of vehicle:
(6 Ticks @ 1/3 mark each) 2
• Calculation of EPS
1/3
(1 Tick @ 1/3)
11/3
Total marks for the question 20
preparation of Statement of Profit or Loss and Other Comprehensive Income of an
entity.
More than 70% of the candidates attempted the question and the performance was fair. Only about
40% of the candidates who attempted the question obtained up to 50% of the marks allocated to it.
Candidates‟ commonest pitfall is their inability to properly calculate and apportion depreciation
charged for the year between the relevant subheads of distribution and administrative expenses.
Candidates are advised to prepare adequately before presenting themselves for examination.
» » SKILLS LEVEL EXAMINATION - NOVEMBER 2015 20
SOLUTION 3

OMOSIGHO LTD

(a) Ratios of Interest to Creditors 2014 2013


(i) Current ratio = Current Assets
Current liabilities 621.935 584.065
268,545 241,770
= 2.3:1 = 2.4:1
(ii) Quick ratio/Acid Test
= Current Assets - Inventory 310,465 323,085
Current Liabilities 268,545 241,770
= 1.16:1 = 1.34:1
(iii) Receivable Turnover
= Revenue 3.364.720 2.750.555
Trade Receivables 274,295 250,260
12 times 11 times
(iv) Debt to Equity Ratio
Long term Debt x 100 125,000 x 100 125,000
S /Capital and Reserve 241,87 211,435
51.7% 59.1%
(v) Interest cover
PBIT 117,060 104,410
Interest charged in the year 12,500 12,500
= 9.4:1 = 8.4:1

b. Ratios of Interest to Management

i. Return on Capital employed(ROCE)


PBIT X 100 117,060 104,410
Equity + Long term debt 62,500 +179370 62,500+148,935
+ 125,000 + 125,000
31.9% 31%
ii. Return on Assets
PBIT X 100 117.060 104.410
Total Asset 703,045 618,770
16.7% 16.0%
iii. Profit to sales Ratio
PBT X 100 104560 91910
Revenue 3,364,720 2,750,355
3.1% 3.34%
iv. Assets Turnover Ratio 2.750.335
Revenue X 100 3,364,720 336,435
Equity + Long term debt 366,870 8.2:1
9.2 :1
'OR'
v. Asset Turnover 3,364,720 2750355
Revenue 434,500 377,000
Equity + NCL 7.7 times 7.3 times

Ratios of Interest to Shareholder

2014 2013
i. E arnings per share (EPS) 48175
PAT x100 52030 62,500
No of ord share 62,500 77k/share
83 k/share
P. E arnings ratio (P/E Ratio)
MPS 1.20 1.20
EPS 0.83 0.77
1.4:1 1.6:1
iii. D ividend yield 0.3 0.28
DPS x100 1.20 1.20
MPS 25% 23.33%
iv. D ividend Cover 0.83 0.77
EPS 0.3 0.28
DPS 2.77 Times 2.75 Times
OR
D ividend Pay Out 0.28
DPS 0.3 0.77
EPS 0.83 36.3%
36.1%

N.B: The ratios stated above are not exhaustive for all categories.
COMMENTS
The Company performance appeared to be relatively stable in terms of short term and long term
liquidity. This is because the current ratio and Acid test ratio are relatively stable and higher than
the theoretical average for a manufacturing industry which is 2:1 for current and 1:1 for Acid test
Ratio.
Also the performance of the company in terms of profitability and efficiency is fair and stable as
both the profitability ratio and efficiency ratio appeared to be better than industrial average.
Although the profit to sales Ratio is low but the company utilized its assets efficiently.
For minority shareholders the performance of the company is good as the dividend yield is
relatively high and increased over the period (that is from 2013 to 2014). Similarly the earnings of
the company adequately covers the dividend declared. However for majority shareholders the P/E
ratios may not be good enough because the ratio declined over the period which is an indication
that the future potential of the company might deteriorate.

Marking Guide ____________________________________________________ Marks


a. Two accounting ratios each:
(i) CREDITORS
• Any two ratios of interest to creditors,
• Correctly identified with right calculations and
Answers 5
(ii) MANAGEMENT
• Any two ratios which are of interest to the management
• Correctly identified, calculated and rightly answered 5
(iii) SHAREHOLDERS
• Calculation of two ratios which are of interest to the Shareholders
• Correctly identified, calculated and
rightly answered 5
15
b. Comments
(i) CREDITORS
• Stating the trend of profitability 1
• Comparison with industrial average 1
(ii) Management
• Stating the trend of profitability 1
• Identifying Improvement in efficiency of utilization of assets 1
iii. SHAREHOLDERS
• Identifying Improvement in earnings per share/Dividend yield 1
• Specifying deterioration in Price earnings ratio 1
• Stating Stability of dividend pay out ratio 1
• Any five valid points from b(i)-(biii) 5
Total marks for the question 20
Examiner's Report
The question tests the ability of the candidates to compute relevant accounting ratios which are of
interest to creditors, management and shareholders as well as the interpretation of the ratios
calculated.
About ninety percent (90%) of the Candidates attempted the question and performance was above
average. The commonest pitfalls of the Candidates was their inability to correctly identify the
relevant ratios for each class of the Stakeholders while others could not correctly interprete the
ratios calculated.
Ratio analysis and interpretation of Financial Statements is a regular feature at the skill level of the
Institute's examinations. Therefore candidates are advised to pay more attention to this section of
the syllabus while also emphasizing other sections for better performance in future examinations.
SOLUTION 4

a. INDICATORS OF IMPAIRMENTS
When assessing whether there is an impairment, IAS 36 requires that the following factors
should be considered by an entity

EXTERNAL FACTORS
i. An unexpected decline in the assets market value
ii. Significant changes in Technology, market, economic factor or laws and regulations
that have an adverse effect on the company.
iii. An increase in interest rates, affecting the value in use of the assets.
iv. Whether the carrying amount of the net assets of the entity is more than its market
capitalization.

INTERNAL FACTORS
i. Evidence that the asset is damaged or no longer of use to the entity
ii. There is a reduction in the assets expected remaining useful life
iii. There is plan to discontinue or restrict the operation for which the assets is currently
used
iv. There is evidence from internal reporting indicating that asset is performing worse
than expected.

(b) PHONEX NIG LTD - INTERNAL


Memo From: Mr. XYZ
To: Financial Controller
Subject: Impairment of Plant and Equipment
Date: 1st February, 2015

The Recoverable amount of an asset is defined as the higher of its fair value minus cost of
disposal and its value in use. While Impairment loss is the amount by which the carrying
amount of an asset (or a cash generating unit) exceeds its recoverable amount.

In view of the above, the following submissions are made in respect of our plant and
equipments
1) MAST
The Recoverable amount is N302,500,000 and since the carrying amount (N29,750,000
is lower than the recoverable amount, no impairment loss should be recognised on the
plant and equipment.

2) GENERATORS
The Recoverable amount is N517,500,000 and since the carrying amount (N-
592,500,000) is higher than the recoverable, amount, the impairment loss to be
recognized is N592,500,000 - N517,500,000 =N75,000,000.
According to IAS 36 (paragraph 60) the loss should be treated as a revaluation
decrease.

Therefore N15,000,000 of the loss should be debited to the revaluation surplus in other
comprehensive income and the balance of the loss of N60,000,000 should be
reorganized/charged in the profit or loss

3) COMPUTER EQUIPMENT
The recoverable amount is N307,500,000. And since the carrying amount of
N287,500,000 is lower than the recoverable amount, no impairment should be
recognized.

4) CREDIT CARD MACHINES


The Recoverable amount is N197,500,000. The Asset is impaired since the carrying
amount of N207,500,000 is higher than the recoverable amount of N197,500,000.
The impairment loss is N207,500,000 - N197,500,000 = N 10,000,000. The impairment
loss of N10,000,000 should be debited or expensed in the Statement of profit or loss.

5) MOTOR VEHICLES
The recoverable amount of the bus cannot be determined because the assets value in
use cannot be estimated to be close to its net selling price, and it does not generate cash
inflow from continuing use that are largely independent of those from other assets.

Therefore the management must determine the cash-generating units to which the bus
belongs and state the recoverable amount of this unit as a whole. If this unit consist of
items 1 to 5 of the property plant and equipment. Then the impairment loss arrived at
should be allocated on pro-rata basis to items 1,3,and 5 that is mast, computer
equipment and motor vehicle.

Signed
Mr. XYZ
Marking Guide_____________________________________________________ Marks
a. 1 Mark each for any 5 correct point (External or Internal) 5
b. Calculation of recoverable amount for each item of plant &
Equipment (1 mark x 5) 5
• Comparison of Recoverable & Carrying amount to
determine impairment for each item of plant & Equipment
(1 mark x 5) 5
• Correct treatment of Generators impairment 2
• Correct treatment of Credit card machine impairment 1
• Correct explanation on the derivation of impairment of
Motor vehicle 2
Total marks for the question 20

EXAMINER'S REPORT
The question tests the provisions and applications of IAS 36 -Impairment of Assets.
More than seventy percent (70%) of the Candidates attempted the question and performance was
average. Majority of the candidates were able to correctly highlight internal and external factors
that are indications of assets impairment in part(a) of the question however, only few candidates
could explain the treatment of the impairment loss in the books of the company in part „b'.
Candidates are advised to make proper use of the Institutes Study Pack when preparing for their
examinations because the study packs treated in details all IFRS relevant to this level of the
Institutes examinations.
SOLUTION 5

a. UDO PLC GROUP


CONSOLIDATED STATEMENT OF FINANCIAL POSITIONAS AT
DECEMBER 31. 2014

U N-'m
Property. plant & Equipment 6,260.00
Other investments 1,200.00
Goodwill on consideration 49000
Total non-current assets 7,950.00
Current Assets 1.878.00
Total Assets 9.828.00
Equity & Liabilities:
Equity N-'m
Equity shares of N1 each 400.00
Share premium 1,000.00
Revaluation Surplus 60.00
Retained earnings 5,846.50
7,306.50
Non-controlling interest 175.50
Non-current Liabilities:
Loan notes 1,300.00
Current Liabilities:
Sundry payables 1.046.00
Total equity & Liabilities 9.828.00

Workings N'M N'M


1) Goodwill on Consideration:

Cost of investment /consideration 1,300


FV of NCI at acquisition 160
Less Aloma Plc Net Asset at Acquisition: Share 200 1,460
Capital

Share Premium 170


Retained Earnings 600 (970)
490
2. Consolidated Retained Earnings: N'M N'M
Udo Plc Aloma Plc
Retained earnings per question 5,800 662
Pre-acquisition - (600)
Post acquisition - 62
Group share at 75% 46.5
5.846.5
4. Non- controlling interest N'M
Four value at acquisition 160
Share of post acquisition retained earnings 15.5
25% of N62 million 175.5

Marking Guide ____________________________________________________ Marks


- Determination of the components of non-current assets
• Property Plant & Equipment %
• Goodwill %
• Other Investment %
• Total non-current assets %
2
- Stating the Current & Total Assets figures 1
- Determination of components of Equity & Liabilities
• Share Capital %
• Share Premium %
• Retained Earnings %
• Revaluation surplus %
2
- Stating Loan notes figure %
- Stating sundry payables and total equity & Liabilities 1
- WORKINGS:
• Calculations of goodwill on Consolidation:
(8 ticks @ % mark each) 4
• Calculations of consolidated Retained earnings
(6 ticks @ % mark each) 3
• Calculations of non-controlling interest
(3 ticks @ % mark each) 1/2
Total marks for the question 15
EXAMINER‟S REPORT
The question tests candidates‟ knowledge of group financial statements.
Most candidates attempted the question and the performance was good.
The commonest pitfall is the candidates‟ inability to properly apply the Fair Value when
computing the Non-controlling interest and the wrong calculation of the consolidated retained
earnings. Some candidates also lost marks due to wrong approximation of figures and non-
indication of appropriate naira symbol.
Candidates are advised to note every detail in the question before attempting it, so as to avoid loss
of marks.
SOLUTION 6

GLOBAL PLCSTATEMENT OF CASH FLOWFOR THE YEAR


ENDED SEPTEMBER 30. 2014
METHOD 1- USING INDIRECT METHOD
Cash flow from operating Activities

N'000 N'000
Net Profit before Taxation 170.000
Adjustments: 20,000
Depreciation 1,000
Finance Cost 10.000 31.000
Goodwill 201.000
Increase in Receivables/payables: (20,000)
Increase in Taxes (15.000)
Increase in Trade Receivables (30.000)
Decrease in Trade Payables (20,000) (85,000)
Decrease in other Payables (50,000)
Taxation paid (1.000)
Interest charges paid 65,000
Net cash inflow from operations Nil
Investing activities
Financing activities
Share Capital and Premium 12,000
Loan notes (10%) 10,000
Dividend paid (87,000) (65.000)
Net Increase /decrease in cash and
cash equivalent Nil
Opening cash and cash equivalent
Cash and Bank 5,000
Overdraft (10,000) (5,000)
Closing cash and cash equivalent 5.000
Workings N'000

1. Taxation paid
Opening 30.000
balance Per 40.000
profit or loss 70.000

Less
Closing Balance (20,000)
Tax Paid 50.000

2. Capital/ share capital N'000


Increase in share capital(N10,000-N8,000) 2,000
Share premium received (wk 2.1) 10,000
12.000
2.1 Share premium received
Retained earnings B/F 197,000
Profit for the period 130.000
327,000
Closing retained earnings (250,000)
Retained earnings declared as part of Dividend 77,000
Total Dividend paid 87.000
Share Premium paid as dividend 10.000

a. Comments

- The company has not managed its cash flows properly despite Net cash inflow of
N65,000,000 generated from operations, since the company still went ahead to pay
dividend of N87,000,000.
- As a result of high dividend payment the company depleted all funds generated from
operation, share premium and part of other revenue reserve.
- The company might be profitable but the profit is not reflected in its liquidity position
hence the request for additional long term loan of N400 m.
- Their bankers may not be too willing to extend the additional loan request in view of poor
state of short term liquidity which may jeopardize the long term liquidity position of the
company.
- If the loan is to be granted the bank might impose conditions on the company to moderate
its dividend payment policy.
- There is no cash inflow or outflow from investing activities during the year. This is an
indication that Global Plc may have poor investment culture or that the company has sold
all its assets and money realized is shared to shareholders in form of dividend.

METHOD 2 - „USINGDIRECTMETHOD'
GLOBAL PLC
STATEMENT OF CASH FLOWS FOR THE YEAR ENDED 30 SEPTEMBER 2014

Operating Activities: __________________________________________ Notes N'000


Cash receipts from customers 1 485,000
Cash payments to suppliers 2 (350,000)
Cash payments for operating expenses 3 (19,000)
Company income Tax paid 4 (50,000)
Cash interest paid 5 (1,000)
Net cash inflow from operating activities 6 65.000
Investing activities:
Net cash outflow from investing activities

Financing activities:
Issue of new share(including share premium) 6 12,000
10% loan notes proceeds 10,000
Dividends paid (87.000)
Net cash outflow from financing activities 65.000

Net increase/Decrease cash & cash equipment 0

Cash and cash equivalents: opening balances:


Bank balances 5,000
Bank Overdraft (10,000)
Short term investments
(5,000)
Expected closing balances agrees with actual closing balance of:
Bank balances 20,000
Bank Overdraft (25,000)
Short term investments (5.000)

Workings
1. Cash receipts from customers N'000
Opening balance of accounts receivable 60,000
Add: Revenue for the year 500,000
Expected cash from customers 560,000
Deduct: Closing cash receipts from customers during the year (75.000)
Actual cash receipts from customers during the year 485.000

2. Calculation of purchase during the year N'000

Cost of sales 300,000


Add: Closing inventory 100.000
Cost of goods available for sale 400,000
Deduct: Opening inventory (80,000)
320,000
30,000
Payment in respect of Trade payable 350.000
3 ». Cash interest payments N'000

Opening balance of accrued interest -


Add: Interest incurred for the year 1,000
Expected interest payments 1,000
4. Cash Davments for operating expenses N'000

Cash expenses incurred for the year (29,000-20,000-10,000) (1,000)


Add: Closing balance of prepayments -
Opening balance of other payable 40,000
Expected payments 39,000
Deduct: Opening balance of prepayments -
Closing balance of other expenses payable (20,000)
Actual cash payments for operating expenses during the year 19.000
5. Company income tax paid N‟000
Opening balance of income tax payable 30,000
Add: Income Tax expense for the year 40.000
Expected income tax payments 70,000
Deduct: Closing balance of income tax payable (20,000)
Actual company income tax paid during the year 50.000
6. Cash interest payments N‟000

Opening balance of accrued interest -


Add: Interest incurred for the year 1.000
Expected interest payments 1,000
Deduct: Closing balance of accrued interest -
Actual interest payments during the year 1.000

7. Issues of new shares N‟000 N‟000


Closing share capital 10,000
Less operating share capital 8.000 2,000
Share Premium
Opening reserve 197,000
Add: Profit for the year 130,000
327,000
Deduct: dividend paid (87.000)
Reserve (240,000) 10.000
250.000
12 .000

Marking Guide ____________________________________________________ Marks


Global Plc
Statement of cashflow for the year ended September 30, 2014
(ai) Using Indirect method
Ascertainment of operating activities 31/2
1/4
Determination of Inventory activities
Determination of Financing activities 1
Determination of Cash and Cash equivalent 114
6
Workings relating to:
Presentation of tax paid in the year 1
Financing activities 3
4
Total 10

OR

(aii) Using Direct method


Ascertainment of operating activities 11/2
Determination of Financing activities 1
Computation of Cash and Cash equivalent 11/2
workings relating to:
Determination of cash receipts from customers 1
Computation of purchases 11/2
1/2
Ascertainment of cash interest payments
3/4
Determination of cash payments for operating expenses
Computation of company income tax paid 1
1/2
Ascertainment of the value of new shares issued
1/4
Computation of share premium
Determination of cash interest payments -
Total 10
(b) Comments on the cashflow management strategies:
Any five valid points at 1 mark each 5
Total marks for the question 15

EXAMINER'S REPORT
The question tests preparation of statement of cash flows using either direct or indirect method.
Candidates were also required to use the statement of cash flow prepared as a guide to comment
on cashflow management strategies of the entity.
Almost all the candidates attempted the questions and performance was good. The commonest
pitfall is the Candidates' inability to ascertain that ordinary share capital issued was at a premium
and that the premium generated was also used as part of the dividend payments.
The statement of cash flows is one of the important components of Financial Statements which are
normally included in annual report of companies hence candidates are advised to familiarise
themselves with the Annual report of Companies and relevant International Financial Reporting
Standards(lFRS) in order to improve on their performance in future.
SOLUTION 7

a. DIFFERENCE IN THE TREATMENT OF AN INVESTMENT PROPERTY CARRIED


AT FAIR VALUE AND OWNER-OCCUPIER PROPERTY CARRIED UNDER
REVALUATION MODEL
Generally speaking Revaluation model and fair value sound very similar because both
require properties to be valued at their fair value which is usually a market based
assessment (often by an independent valuer)
- However any gain or loss over the previous valuation is taken to profit or loss if it relates
to an investment property. whereas for an owner-occupied property any gain is taken to a
revaluation reserve (via other comprehensive income and statement of changes in equity).
And a loss on the revaluation of an owner- occupier property is charged to profit or loss
unless it has previous balance in its revaluation reserve which can be used to offset the loss
until it is exhausted.
- A further difference is that an owner-occupier property continues to be depreciated after
revaluation whereas investment properties are not depreciated
-
b. i.TREATMENT OF THE PROPERTY IN THE FINANCIAL STATEMENTS OF KOLA NITDAPLC A
The property should be classified as an Investment property and should be accounted for in
terms of the fair value model in IAS 40

As the portion of the property cannot be sold separately the entire property must be
assessed under IAS 40. The motivation for classifying the whole property as an investment
property is that the portion occupied by the company for administrative purposes (6%) is
insignificant and security services rendered to the lessees are also insignificant.
Furthermore, the majority of the floor space is used to generate rental income.

ii. INVESTMENT PROPERTY TO BE DISCLOSED IN STATEMENT OF FINANCIAL


POSITION

N ' 000
Opening Balance -
Additions (32,000 + 84,000) 116,000
Improvement to building 16,000
Net gain in fair value Adj(WK1) 12.000
Closing balance at fair value 144.000
Workings (1)

Carrying amount of Investment Property


N'000
Land 32,000
Building 84,000
Improvement to building 16,000
Carrying Amount 132,000
Fair value 144.000
Fair value Adjustment per
Profit or loss and other comprehensive 12.000

Marking Guide ______________________________________________ Marks


a. Explanation of measurement models 1
Treatment of differences in gain or loss over the previous valuation
(investment property and owner-occupied property) 4
5
b. (i) Correct identification of
the property type 2
Reasons for classification as an investment
Property 3 5
(ii) Computation of the value of investment property
for disclosure(5 ticks @1/2 mark each) 21/2
Computation of fair value adjustment charged to statement of Profit
or loss and other
Comprehensive Income (5 ticks @1/2 mark each) 2/2 5
Total marks for the question 15
Examiner's Report
The question tests the application of the IAS-40 on investment property with particular emphasis
on the treatment of Investment Property carried at fair value and Owner-Occupier property stated
on revaluation basis.
About twenty percent (20%) of the Candidates attempted the question and performance was poor.
Majority of the Candidates that attempted the question could not correctly differentiate between
Investment Property carried at fair value and Owner-Occupier Property stated on revaluation
basis. Similarly, most of the candidates could not correctly calculate value of Investment Property
that should be disclosed in the Statement of Financial Position while others could not determine
the amount to be charged to Statement of Profit or Loss and Other Comprehensive Income.
The Institute syllabus clearly identified IFRS which candidates must be familiar with at this level
of the examinations; therefore Candidates are advised to pay special attention to them for better
performance in future.
THE INSTITUTE OF CHARTERED ACCOUNTANTS OF NIGERIA
SKILLS LEVEL EXAMINATIONS - MAY
2015 FINANCIAL REPORTING Time
Allowed: 3 hours ATTEMPT FIVE
QUESTIONS IN ALL
SECTION A: COMPULSORYQUESTION (30 Marks)
QUESTION 1
UN1TAR1SATON PLC
Unitarisation Plc is a successful Nigerian Company which recently amended its objects clause in
the Memorandum of Association to include "programmes and activities that will promote
National unity and encourage anti-terrorism compliance" as part of its social responsibilities. The
company, therefore, acquired 60% of the equity share capital of Famous Plc a widely known and
successful advertising company to propagate this mission.
The summarised draft financial statements of the two companies are as follows:
Statement of Profit or Loss and Other Comprehensive 1ncome for the year ended
31 October, 2014.

Unitarisation Famous
Plc. Plc.
N'm N'm
Revenue 51,000 25,200
Cost of sales (37,800) (19,200)
Gross profit 13,200 6,000
Distribution costs (1,200) (1,200)
Administrative expenses (3,600) (1,920)
Finance costs (180) (240)
Profit before tax 8,220 2,640
1ncome tax Expense (2,820) (840)
Profit for the year 5,400 1,800
Statement of Financial Position as at 31 October, 2014
Unitarisation Plc. Famous Plc.
Assets: N'm N'm
Non-current assets:
Property, plant & equipment 24,360 7,560
Current assets 9,600 3,960
Total assets 33.960 11,520
Equity & Liabilities:
Equity shares of N1 each 6,000 2,400
Retained earnings 21,240 3,900
27,240 6,300
Non-current liabilities:
12% loan notes 1,800 2,400
Current liabilities 4,920 2,820
Total equity & liabilities 33,960 11,520

Relevant additional information are presented below:


(i) The shares of Famous Plc. was acquired on 1 May, 2014 and the issue of shares was not
recorded by Unitarisation Plc.
(ii) There is cash in transit of N120,000,000 due from Unitarisation Plc. to Famous Plc.
arising from intra-group trading.
(iii) The non-controlling interests are valued at full fair value by the parent company. The fair
value of the non-controlling interests in Famous Plc. at the date of acquisition was
N3,540,000,000. There is no goodwill impairment at the end of the accounting year.
(iv) The fair value of Famous Plc. assets were equal to the carrying amounts at the acquisition
date except for one equipment with a fair-value of N1,200,000,000 over its carrying
amount which has a five-year remaining life. Straight line depreciation is adopted.
Famous Plc. has not effected the adjustment in the records.
(v) The 60% of share capital of Famous Plc. acquired was settled through share exchange of
two shares in Unitarisation Plc. for three shares in Famous Plc. The market value of
Unitarisation Plc. at the date of acquisition was N6 per share.
(vi) In the post-acquisition period, Unitarisation Plc. bought goods from Famous Plc.
amounting to N4,800,000,000. Famous Plc. had made a mark-up on cost of 40% on the
transaction. As at the year end, Unitarisation Plc. had sold part of these goods worth
N3,120,000,000.
(vii) Famous Pic's trade receivables at 31 October, 2014 include N360,000,000 due from
Unitarisation Plc. However, the amount did not agree with the corresponding balance in
Unitarisation Plc's trade payable ledger.
(viii) Assume that profits or losses accrue evenly over the period except otherwise stated.
Required:
a. Prepare Unitarisation Plc. Consolidated Profit or Loss and Other Comprehensive
Income for the year ended 31 October, 2014. (10 Marks)
b. Unitarisation Plc. Consolidated Statement of Financial Position as at 31 October,
2014. (10 Marks)
c. Consolidated Statement of Changes in Equity for the year ended 31 October, 2014.
(6 Marks)
d. In accordance with IFRS 3 on Business Combinations, what is Gain On Bargain
Purchase? (4 Marks)
(Total 30 Marks)

SECTION B: ATTEMPT TWO OUT OF THREE QUESTIONS IN THIS SECTION(40Marks)


QUESTION 2
a. When a parent Company elects not to prepare consolidated financial statements and
instead prepares separate financial statements; what are the disclosure requirements
stipulated in IAS 27 on Separate Financial Statements?
(6 Marks)
b. Kerewanta Plc. acquired 60% of the equity shares of Orijinmi Plc. by means of share
exchange of three shares in Kerewanta Plc. for four shares in Orijinmi Plc. The market
value of the shares of Kerewanta Plc. at the date of acquisition which is 1 April, 2013
was N10 per share.
Kerewanta Plc. would make a deferred cash payment of 70k per acquired share on 1
April, 2014. Kerewanta Plc. cost of capital is 12% per annum. None of the consideration
has been recorded in the books of Kerewanta Plc. The following information was
extracted from the financial statements of the two companies as at 31 March, 2014.
Kerewanta Pic. Orijinmi Pic.
N'm N'm
Equity shares of N1 each 60,000 20,000
Share premium 15,000 NIL
Retained earnings 1 April, 2013 20,500 11,600
Retained earnings for the year
ended 31 March, 2014 9,800 6,700
Property, plant and equipment 50,400 22,900

The following is the additional relevant information:


(i) An equipment had a fair value of N360,000,000 above its carrying
amount. At the date of acquisition of Orijinmi Plc. the asset had a
remaining life of four years. It is the group's policy to depreciate such
asset using the straight-line method.
(ii) Orijinmi Plc. had deferred tax liability of N10,000,000 as at 31 March,
2014 which had not been recorded. The company's goodwill is not
impaired.
(iii) Non-controlling interests are to be valued at fair value at the date of
acquisition of Orijinmi Plc. The fair value of the shares of Orijinmi Plc.
held by non-controlling interests at the date of acquisition is N6 per
share.
Required:
Calculate the following as at 31 March, 2014:
i. Equity
ii. Non-controlling interests
iii. Consolidated goodwill
iv. Property, plant and equipment

(14 Marks)
(Total 20 Marks)
QUESTION 3

Galadanci Plc. is a telecommunications company operating in Nigeria. The management of the


company presented the following summarised financial statements for the years ended 31
December, 2013 and 2014.
Statements of Profit or Loss and Other Comprehensive Income for the year ended:

2014 2013
^billion N'billion
Revenue 2,430 1,638
Cost of sales (1,701) (983)
Gross profit 729 655
Administrative costs (311) (180)
Distribution costs (207) (117)
Finance costs (36) (6)
Profit before taxation 175 352
Income tax expense (54) (102)
Profit for the year 121 250

tatements of financial position as at 31 December:


2014 2013
N'billion N'billion N'billion N'billion
Assets:
Non-current assets:
Property, plant & equipment 612 369
Intangible assets 270 180
Investment in shares of Papanga Plc. at cost 207 Nil

1,089 549

Current assets:
Inventory 120 100
Trade receivables 150 70
Other receivables Nil 270 110 280
Total assets 1,359 829

Equity & liabilities:


Equity shares of N1 each 390 225
Retained earnings 340 270
730 495
Non-current liabilities:
10% secured loan notes 80 80
12% secured loan notes 260 340 Nil 80

Current liabilities:
Bank short term loan 49 10
Trade payables 190 140
Income tax payable 50 289 104 254
Total equity & liabilities 1,359 829
The following additional information is relevant for the year ended 31 December,
2014:
(i) Galadanci Plc. acquired 60% interests in the equity shares of Papanga Plc. which is into
commercial rice production in order to diversify its business portfolio and take advantage
of the favourable incentives in agriculture recently announced by the Federal
Government of Nigeria.
(ii) Galadanci Plc. increased its mobile telephone subscriber based and average revenue per
user.
(iii) No dividends were received from Papanga Plc. and the value of its shares had not
increased during the year ended 31 December, 2014.
Required:
a. Calculate the following ratios and use them to analyse Galadanci Plc's operating
performance during the year ended 31 December, 2014 and comment on any relevant
qualitative factors that may impact on the company's performance.

i. Gross profit percentage


ii. Return on capital employed (where capital employed current = Total Asset less
liabilities)
iii. Net profit (PB1T) percentage
iv. Asset turnover
v. Gearing ratio
vi. Debt/Equity ratio (16 Marks)
b. Prepare Galadanci Plc's Cash Flows from operating activities using the indirect method in
accordance with 1AS 7 on Statement of Cash Flows. (4 Marks)
(Total 20 Marks)
QUESTION 4

a. The following information is extracted from the financial statements of Kubua Plc. for
the year ended 30 September, 2014.
N'000
Ordinary Share Capital (fully paid at 1.25 kobo each) 20,000
Operating profit before tax 4,000
Other relevant information:
(i) The companies income tax rate is 30%
(ii) The average fair value of one ordinary share during the year was N5.00.
(iii) During the year, the company issued share options of 2.5million ordinary shares
to existing shareholders at an exercise price of N4.00.
Required:
Calculate the basic and diluted Earnings Per Share for the year ended 30 September,
2014. Show all workings (5 Marks)

b. Extract from the Statements of Profit or Loss and Other Comprehensive Income of
Bajulaye Plc. for the years ended:
30/09/2014 30/09/2013
N'000 N'000
Revenue 5.000 2,800
Profit Before Interest and Taxes (PB1T) 2.500 1,200
Extract from the Statements of Financial Position as at
30/9/2014 30/9/2013
1ssued Share Capital: N'000 N'000
Ordinary Shares at 50k each 3,000 3,000
12% Redeemable Preference Shares 1,500 1,500
Total Equity 4,500 4,500
Other relevant information:
- On 1 January, 2013 the entity issued convertible loan notes of N2,000,000 with
effective interest rate of 10% per annum.
- The loan notes are convertible at nominal values of N100 each into the following
number of ordinary shares:
30 September, 2018 130 shares
30 September, 2019 125 shares
30 September, 2020 114 shares
30 September, 2021 105 shares
- Companies Income tax rate is 30%.
Required:
i. Calculate the basic and diluted Earnings Per Share for the year ended 30
September, 2014. (8 Marks)
ii. Write a short memo to the Board of Directors of Bajulaiye Plc. explaining
FOUR advantages and THREE limitations of Earnings Per Share as a
performance indicator to users of financial statements. (7 Marks)
(Total 20 Marks)

SECTION C: ATTEMPT TWO QUESTIONS IN THIS SECTION (30 Marks)


QUESTION 5
1AS 38 - 1ntangible Assets, specifies the criteria that must be met before an intangible asset can
be recognised by an entity in its Financial Statements. 1ntangible assets are identifiable non-
monetary assets without physical substance and include goodwill, brands, copyright and research
and development expenditure. They could be purchased and/or internally generated.
Required:
a. 1dentify any TWO characteristics of goodwill which distinguish it from other
intangible assets? (2 Marks)
b. Explain THREE differences between purchased goodwill and non-purchased
goodwill. (3 Marks)
c. 1dentify any THREE conditions that must be met under 1AS 38 for development
expenditure to be recognised as an intangible asset. (3 Marks)
d. State any FOUR factors to be considered when determining the useful life of an
intangible asset. (4 Marks)
e. Calculate the goodwill on consolidation from the information below:
N'000
Parent's cost of investment in subsidiary 299,700
Net asset at acquisition date (parent) 986,600
Net asset at acquisition date (subsidiary) 345,800
Fair value of non-controlling interest atacquisition date 169,500
Net asset at reporting date (subsidiary) 316,400
1mpairment of goodwill 62,200
Parent has 80% interests in subsidiary (3 Marks)
(Total 15 Marks)

QUEST1ON 6
a. 1AS 28 - 1nvestments in Associates and Joint Ventures permits, the application of equity
method when accounting for investments in associates and joint ventures.
Required:
Explain briefly the Equity Method and state the circumstances under which an entity can
discontinue the use of equity method under IAS 28. (5
Marks)
b. Agbantara Plc. acquired equity shares from Odinma Plc. and Dangari Limited. The
following are the Statements of Profit or Loss and Other Comprehensive 1ncome for the
year ended 31 December, 2014 for the three companies:

Agbantara Plc. Odinma Plc. Dangari Ltd


N'm N'm N'm
Revenue 4,500 1,350 630
Cost of sales (2,430) (720) (270)
Gross profit 2,070 630 360
Admin expenses (1,350) (180) (135)
Finance income 135 90 -
Finance costs (180) - (90)
Profit before tax 675 540 135
1ncome tax expenses (225) (135) (45)
Profit for the year 450 405 90
Other comprehensive income:
Gains on property revaluation,
net of tax 180 90 45
Total comprehensive income for
the year 630 495 135
The following information is also relevant:
(i) Agbantara Plc. acquired 72 million ordinary shares in Odinma Plc. out of its
120,000,000 ordinary shares of N1 each par value for N160,000,000. The shares
were acquired four years ago when it had N15,000,000 credit balance on its
retained earnings. During the year, Odinma Plc. sold goods costing N38,000,000
to Agbantara Plc. for N45,000,000. These goods were yet to be sold as at 31
December, 2014.
(ii) Agbantara Plc. acquired 35,000,000 ordinary shares in Dangari Limited out of
100,000,000 ordinary shares. The shares were acquired three years ago when the
company had a credit balance on its retained earnings of N10,000,000.
(iii) Agbantara Plc's group policy is to measure non-controlling interests (NCI) at fair
value. NCI at acquisition date in Odinma Plc. at fair value was N48,000,000.
Impairment test carried out on the goodwill relating to Odinma Plc. and
investment in Dangari Limited at year end resulted in N10,000,000 and
N15,000,000 losses respectively.
You are required to:
Prepare Agbantara Plc. Consolidated Statement of Profit or Loss and Other
Comprehensive Income for the year ended 31 December, 2014.
(10 Marks)
(Total 15 Marks)
QUESTION 7
a. There is usually a lead time between the end of an entity's accounting year and when the
financial statements are approved and signed off by the directors. In between this period,
there are two types of events according to IAS 10-Events After The Reporting Period,
which may require consideration when preparing financial statements.
You are required to:
Identify and explain these events and state how they are treated in the financial
statements. (4 Marks)
b. Company A is indebted to company B to the tune of N50,000,000. The financial year-end
of company B is 30 June, 2014. On 30 July 2014, company B received a letter from a
liquidator advising it that company A has gone into insolvency. The letter revealed that
company A ceased operations a month ago and that company B is only likely to receive a
liquidation dividend of 20k for every naira owed by company A. It is the normal practice
of company B's board to approve the audited financial statements three months after the
financial year end.
Required:
i. Explain how the above transactions should be treated in the financial
statements of company B in accordance with IAS 10-Events After The Reporting
Period. (2 Marks)
ii. Prepare journal entries that are required to adjust company B's financial
statements to account for the above event. (2 Marks)
iii. State what would have been the treatment in the financial statements
assuming it was fire that destroyed company B's factory building on 30 July,
2014. (3 Marks)
c. The directors of XYZ Plc declared that a dividend of N1 per ordinary share be paid to
shareholders on the company's register as at 15 April, 2014. The financial statements
were approved by the company's board on 30 May, 2014. The shareholders, at the
company's annual general meeting held on 15 June, 2014, approved the payment of the
dividend to eligible shareholders on 1 July, 2014.
Required:
Explain how the dividend proposed by the Directors should be treated in the
financial statements of XYZ Plcin accordance with IAS 10. (4 Marks)
(Total 15 Marks)
SECTION A

SOLUTION 1 (a)
UN1TAR1SAT1ON PLC.
CONSOLIDATED STATEMENT OF PROFIT OR LOSS AND OTHER
COMPREHENSIVE INCOME FOR THE YEAR ENDED 31 OCTOBER, 2014
N'm

58,800
Revenue (51,000 + (25,200 x 6/12) - 4,800 (Wk 3) 4,800
(43,200
Cost of sales (Wk 2) )
Gross profit 15,600
Distribution costs (1,200 + (1,200 x 6/12) (1,800)
(4,560)
Admin. Costs (3,600 + (1,920 x 6/12) (300)
8,940
Finance costs (180 +(240 x 6/12) (3,240)
5.700
Profit before tax __ Nil
5.700
Income tax (2,820 + (840 x 6/12)

Profit for the year


Other comprehensive income
Profit
Total attributable to:
comprehensive income
Owners of the parent 5,580
Non-controlling interest (wk 1) 120
5,700

Workings:
Wk 1 Non-controlling N'm
interests: Statement of profit or
loss
Post acquisition profit (Famous) (1,800 x 6/12) 900
Unrealised profit (480)
Fair value adjustment movement (120)
NCI post acquisition share (40% x 300) 300
120
Wk 2 Cost of sales

N'm
Unitarisation Plc 37,800
9,600
Famous Plc (19,200 x 6/12) 120
(4,800)
Movement of fair value adjustment (Wk 480
1) Intragroup purchases (Wk 3) 43,200
Unrealised profit (Wk 3)

Wk 3 Intragroup Trading:
DR CR
N'm N'm
Revenue 4,80 4,800
Purchases 0
Being removal of intragroup sales and purchases
Retained earnings/cost of sales (4,800 - 3,120) x 40/140
480
1nventories 480
Being elimination of unrealised profit

(b)
UNITARISATION PLC.
CONSOLIDATED STATEMENT OF FINANCIAL POSITION AS AT 31 OCTOBER,
2014
N'm
ASSETS
33,000
Non-current assets
2,700
Property, plant & equipment (24,360 + 7,560 + 1,080) (Wk 5)
35,700
Goodwill (Wk 3)
12,840
Current assets (Wk 8)
48,540
EQUITY AND LIABILITIES
Equity attributable to owners of parent company:
6,960
Share capital (6,000 + 960) (wk 6)
4,800
Share premium (Wk 6)
21,420
Retained earnings (Wk 2)
33,180
Shareholders fund
3,660
Non-controlling interest (Wk 4)
36,840

Non-current liabilities
12% loan notes (1,800 + 2,400) 4,200
Current liabilities
(4,920 + 2,820 - 240) (Wk 7) 7,50
48,540
0
Workings
Wk 1 Group structure
Unitarisation Plc.
60% (1/5/2015)

Famous Plc
The subsidiary was acquired during the year (middle of the accounting Year)

Wk 2 Retained earnings:
Unitarisation Famous
Plc. Plc.
N'm N'm
As in the question 21,240 3,900
Fair value adjustment movement (Wk 5) (120)
Unrealised profit (Wk 3) (480)
Pre-acquisition profit (3,000)
Retained earnings of subsidiary 300
Group share of subsidiary (60% x 300) 180
Consolidated retained earnings 21,420
Wk 3 Goodwill:
N'm N'm

Consideration transferred 5,760


Fair value of non-controlling interest 3,540
Less: Net asset fair value at acquisition:
Share capital 2,400
Retained earnings 3,000
Fair value adjustment 1,200 (6,600)
Goodwill 2,700

Wk 4 Statement of Financial Position:


N'm
NCI at acquisition 3,540
Share of post acquisition retained earnings (40% x 300) 120
3,660
Wk 5 Fair Value Adjustments:

Acquisition Movement Year


1/5/14 (6/12) end
N'm N'm 31/10/
Movement in equipment (120)
1,200 14
N'm
1,080
Wk 6 Share exchange:

DR CR
N'm N'm
No of share issued (2,400 x 60% x 2/3) =
5,760
960 Consideration transferred (960 x N6)
Unitarisation share capital (960 x N1) 960
Unitarisation share premium (960 x N5) 4,800

Wk 7 Cash in transit:

DR CR
N'm N'm
Receivables 360
Payables 240
Group cash 120
Wk 8 Current asset:
N'm
Unitarisation Plc. 9,600
Famous Plc. 3,960
Unrealised profit (480)
1ntercompany receivables (State in the question) (360)
Cash in transit (WK 8) 120
2,840

c)
UNITARISATION PLC.
CONSOLIDATED STATEMENT OF CHANGES IN
EQUITY FOR THE YEAR ENDED 31 OCTOBER, 2014
Share Share Retained Equity NCI Total
Capital Premium Earnings Unitarisation Equity
N'm N'm N'm N'm N'm N'm
As at 1 November, 2013 6,000 15,840 21,840 21,840
Profit for the year 5,580 5,580 120 5,700
Other comprehensive income:
Transactions with owner's
equity:
Addition paid-in capital 960 4,800 5,760 5,760
Addition on acquisition of
Famous Plc. 3,540 3,540
As at 31 October, 2014 6,960 4,800 21,420 33,180 3,660 36,840

d) Gain on Bargain purchase arises when the aggregate of the consideration transferred
(which is usually measured at fair value), the non-controlling interest and the fair value of
the previously held equity interest is less than the acquisition date amount of the
identifiable assets acquired and the liabilities assumed.
EXAMINER'S REPORT
The question tests candidates' knowledge of Preparation of Consolidated Statement of Profit or
Loss and other Comprehensive Income Statement of Financial Position, Statement of Changes in
Equity and explanation of Gain on Bargain Purchase.
About 98% of the candidates attempted the question and performance was below average.
Most of the candidates could not correctly explain the Gain on Bargain Purchase in accordance
with IFRS 3 on Business Combination, while candidates who were unable to correctly prepare
the consolidated final accounts properly lost some marks, because N'm symbol in their solutions.
(For example recording N5m as N51,000). Also majority of the candidates could not correctly
prepare Statement of Changes in Equity.
Candidates are advised to note that future Chartered Accountants should pay special attention to
denominations and should correctly reflect this in their solution to questions.
Similarly, they should pay attention to relevant International Financial Reporting Standards
(1FRS) that affect preparation of consolidated and separate financial statement for better
performance in future examinations of the 1nstitute.

SOLUTION 2

a)
DISCLOSURE REQUIREMENTS STIPULATED 1N 1AS 27 WHERE A COMPANY
ELECTS NOT TO PREPARE CONSOLIDATED FINANCIAL STATEMENTS
When a parent company elects not to prepare consolidated financial statements and
instead prepares separate financial statements, 1AS 27 on Consolidated and Separate
Financial Statements states that it shall disclose in those separate financials.
(i) The fact that financial statements are separate financial statements.
(ii) A list of significant investment in subsidiaries, joint venture and associates
including:
■ The name of those investees
■ The principal place of business (and country of incorporation, if
different) of those investees
■ Its proportion of ownership interest (and its proportion of the voting
rights, if different) held in those investees.
(iii) A description of the method used to account for the investment listed under (ii).

b)
(i) Equity as at 31 March, 2014

Share capital of Kerewanta Plc. 69.00


Share premium 0
Retained earnings (Wk 1) 96.0 3
3,36
0
198,3
60
(ii) Non-controlling interests as at 31 March, 2014
N'm
Added on acquisition at 1 April, 2013 40% post 48,000
acquisition on profit (40% x N6,700) 2,680
Deferred tax expenses (N10million x 40%) (4)
Additional depreciation (1/4 year x N360 x 40%) (36)
50,640
(iii) Consolidated Goodwill as at 31 March, 2014
Purchase consideration of Kerewanta Plc. N'm
Share consideration 90,000
Deferred consideration 7,500
Non- controlling interest 97,500
Cost of business combination 48,000
Fair value of identifiable net asset at 1 April, 2013 145,500
Share capital
Pre-acquisition reserves N'm
Fair value adjustments 20,000
Full goodwill 11,600
(iv) Property, plant and equipment as at 31 March, 2014 360 (31,976)
113,540

N'm
Kerewanta Plc. 50,400
Orijinmi Plc. 22,900
Fair value Adjustment 360
Less: Additional depreciation (1/4 years x N360million) (90)
73,570
Workings:
Wk 1 Retained earnings as at 31 March, 2014
Kerewanta Plc. retained earnings (N20,500 + N9,800)
N'm
60% post-acquisition profit (60% x N6,700)
30,300
Finance cost (12% x N7,500million)
4,020
Deferred tax expense (N10million x 60%)
(900)
Additional depreciation (1/4 years x N360million x 60%)
(6)
(54)
33,360
EXAMINER'S REPORT

The question tests the disclosure requirement necessary for the computation of items
in the Consolidated Statement of Financial Position, such as equity, property, plant and
equipment.
About 50% of the candidates attempted the question and performance was poor.
The commonest pitfalls observed include the following:
• Inadequate knowledge of the disclosure requirement required for calculating some of the
basic Consolidated Statement of Financial Position (CSFP) items highlighted above.
• Failure to show necessary workings as to how the figures were arrived at.
• Omission of N'm configuration in their solution thus, reducing the figures that are expected
in million naira to thousand naira.
• Failure to recognize deferred tax liability in their solution as required by the examiner.
• It is apparent that most candidates lack the requisite knowledge of IFRS, hence they could
not correctly calculate the relevant items of SFP as required by the examiner.
They are therefore advised to pay more attention to this section of the syllabus as it
could feature regularly at this level of the Institute's examinations.

SOLUT1ON 3

(a)
Ratio Analysis (i) 2014 2013
Gross profit % 729
Grossprofit 2,4 x100% 655
x100% x100%
30' 1,638
= = 39.98%
30175 + (40%)
(ii) Return on capital employed (ROCE) 352 + 6
%36 x100% 829 - x100%
-------111[111111[C1111) ------ 1 [100%
1 1 11 1H11111 1111111 Hu 1 1111111 1,359 - 254
289 358
x100% x100%
1,070
211 575
=19.72% =
=20% 62.26%
= 62%
(iii) Net profit (PB1T) percentage 211 358
x100% x100%
----- ----------- x100% 2,43 1,63
Re veue / sales 0 8
=8.68 =21.86
% %
(iv) Asset Turnover =2,430
9% =1,638
22
1,359 829
= 1.79 times = 1.98 times

(v) Gearing Ratio 80 + 80


260 x100% x100%
Debt 495+8
x100%
730+34 0 80
0 x100%
1,070 ---- x100%
340
=31.78% 575
=32% = 13.91%
= 14%
(vi) Debit/Equity Ratio 389 90
x100% x100%
Debit 730 495
x100%
=53.29 =
% 18.18%
ii. ANALYSIS OF OPERATING PERFORMANCE OF GALADANC1
• There is increase in revenue from N1,638bn to N2,430bn (2014) with a rising cost
of sales from N983bn (2013) to N1,701bn (2014) which leads to decrease in gross
profit % from 40% to 30%.
• Profit before interest and tax dropped from N358bn to N211bn, while capital
employed increased from N575bn to N1,070bn as a result of the increase in the
level of production and non-current asset to meet the level of increase in demand.
• The capital expended on non-current asset is long-term in nature and can only be
recouped on a spread over a number of years, hence the reduction in Net Profit
percentage of 22% (2013) to 9% (2014). Therefore, it will take some time for the
profit to rise synergistically.
• The immediate effect of the increase in non-current asset also led to the decrease
in asset turnover of 1.98 times (2013) to 1.79 times (2014).
• The financing effect is reflected in increase in the gearing ratio from 14% (2013)
to 32% (2014) as a result of the increase in the long-term debt
from N80bn (2013) to N340bn (2014) and capital employed from
N575bn to N1,070bn. This is also reflected in increase in the Debt/Equity ratio
from 18.18% (2013) to 53.29% (2014) which is a very high level of
difference/increase.
• The company, in a bid to increase revenue, might have incurred excessive cost
which is higher in proportion to the increase in the revenue generated through
increase in customers' base.
• The company's investment in the subsidiary (Papanga Plc.) might have been
financed by debt which has equally contributed to significant increase in finance
cost and high gearing ratio, without any commensurate return from the same
investment.
• Other inefficiencies that may be attributed to the company's business model and
strategy.
Comment on Qualitative Analysis
(i) The measure in non-current asset and capital employed may lead to synergistic effect
in the next few years because it works like the cyclical movement in the company's
performance at the peak and if the level of activities are not increased by injecting
more capital funds, it may lead to serious problem that may lead the company exiting
the market if it gets to saturated point, because telecommunication is a dynamic and
complex industry.
(ii) 1nvestment in shares of Papanga Plc. is futuristic in terms of yielding profit because
the industry is at the infant stage which will yield high level of profit in the very near
future.
1t could be a very good product in the investment portfolio of the company.
b)
GALADANC1 PLC.
STATEMENT OF CASHFLOWS FOR
THE YEAR ENDED 31 DECEMBER, 2014 (INDIRECTN'bn METHOD)
N'bn
OPERATING ACTIVITIES: 175
Profit before taxes 36 36
Adjustment for non-cash and non-operating i
tems
Depreciation of property, plant & equipment
Amortisation of intangible assets
Finance cost
Movement in working capital:
Increase in inventory N (120 - 100) (20)
Increase in trade receivables N(150 - 70) (80)
Decrease in other receivables N(110 - 0) 110
Increase in trade payables N(190 - 140) 50 __ 60
Cash flows from operating activities 271
1nterest paid (36)
Tax paid (Wk 1) (108)
127
(Wk 1) Working
N'bn
Tax b/f 104
Income tax expenses (P or L) 54
Less payable bal c/f (50)
108

EXAMINER'S REPORT
The question tests the candidates' ability to calculate financial ratios and the use of those ratios to
analyse the operational performance of a company, while preparation of operating activities of
cash flow using indirect method is also examined in part 'b' of the question.
About 95% of the candidates attempted the question and performance was average.
The commonest pitfalls were the inability of candidates to calculate the financial ratios correctly
and failure to give correct interpretations for ratios calculated.
Candidates are advised to pay more attention to the 1nterpretation of financial ratios being a
financial report paper, as failure to do so may leads to loss of marks.
SOLUTION 4

a. (i) Calculation of Basic Earnings Per Share (EPS)


N'000
Operating Profit Before Tax 4,000
Taxation @ 30% of N4,000,000 (1,200)

Profit for the period 2,800


No. of shares = N20,000,000 N1.25k = 16,000,000 shares
N2 00 000
\ Basic EPS = ,8 ,

16,000,000shares
17.5k

a. (ii) Calculation of Diluted EPS

1111111
DilutedlEPS =
liiiiitBiiintBiiii SI 111 111 1 CB1111 11 [01 111111 1[011 11 1 11 !i 1111
2,800,000/(16,000,000 + 278,000)
2,800
= N0.17 kobo or 17 kobo
16,278 ---------
Test for Dilution
Since the average market price of (N4.50 per share) is higher than the
exercise/strike price (of N4.00 per share), the potential shares are D1LUT1VE.
Step 1 - Proceed on Deemed Exercise of Share Option
= N4 share x 2.5 million shares
= N10 million
Step 2 -Deemed Re-purchased own shares for delivery to the Option Holders
Shares Re-Purchased = N10 million N4.5 per share
= 2,222,222 shares
Step 3 - Potential Ordinary Shares
(i.e. The shortfall to be issue as bonus)
= (2,500,000 - 2,222,222 shares)
= 277,778 shares
» 278,000 shares

Step 4 - Weighted average Potential Ordinary Shares


12
= — x 277,778 shares
12
= 277,778 shares

278,0 shares

Average Market Price N(5 + 4) = N9 2 = N4.5

b. (i) BASIC EPS


N'000
Profit Before Interest and Tax (PB1T) Less: 2,500
Interest on convertible loan notes: 10% of (200)
N2million PBT 2,300
Taxation @ 30% (690)
Pref. Dividend (180)
No of shares 1,430
6,000

1,430
111
= -T———r- = 24[l
Diluted EPS Earnings 111
Add: 1nterest on Convertible Loan 6,000 1,430
200

Income Tax on Conv. Loan @ 30% (60)


1,570
No of shares 6,000
2,000,000
Loan notes converted to Ordinary shares = ----------------------------------- —— x 1302 600
8,600

111 [11 11111111111 = 18[kobo 1,111

b. Date: 19 May 2015


From: Mr. XYZ
To: Chairman, Board of Directors
MEMORANDUM TO BOARD OF DIRECTORS ON ADVANTAGES AND
LIMITATIONSOF EARNINGS PER SHARE (EPS)
The above subject matter refers.
The following are some of the advantages:
- It is a medium through which the EPS of different entities are compared.
- It can also be used to evaluate and compare the EPS of the same entity in
different years. Thus, it is useful in trend analysis of an entity's EPS.
- It measures performance from the perspective of investors and potential investors.
- It points out earnings distributable to the ordinary sharesholders
- It is a measure of returns to entity holders.
Limitations of EPS include the following:
- Inflation is not considered
- Historical cost information from the financial statements is used, so the productive
value is low.
- The choice of accounting policies affects EPS of an entity.
- Window dressing of financial statements raises earnings unnecessarily to
deceive shareholders.
Yours faithfully,
Mr.
XYZ
Signed
EXAMINER'S REPORT

The question tests the principles and application of IAS 33 on Earnings Per Share (EPS).
Candidates are required to compute both 'basic' and 'diluted' Earnings Per Share (EPS).
About 60% of the candidates attempted the question and performance was average.
Most candidates could not correctly calculate the number of shares as well as relevant earnings
particularly for the purpose of calculating diluted Earnings Per Share (EPS).
Candidates are advised to pay more attention to the provision of IAS 33 and other relevant
standards for better performance in future examinations of the Institute.

SOLUTION 5
a. Characteristics of Goodwill which distinguished it from other intangible assets
i. It is a balancing figure: Goodwill itself is between the fair value of the
whole business and the fair value of the separatable net assets of the business. It
cannot be valued on its own.
ii. Goodwill cannot be disposed off as a separate asset.
iii. The factors contributing to goodwill cannot be quantified.
iv. The value of goodwill is volatile, i.e. it can only begiven a numerical value at the
time of acquisition of the whole business.
v. Goodwill exists in perpetuity unless impaired.
b. Differences between purchased and non-purchased goodwill

Purchased Non - purchased


i. Arises when one business
It's inherent in the business.
acquires another as a
going concern.
ii. It arises from purchase Has no identifiable value. and
consolidation of a subsidiary.


iii. Will be recognized in the 1t is not recognized in the financial statements
as its financial statements. value at a particular point in time is certain.

c. Conditions for recognizing development expenditure as an intangible asset


exists where the entity demonstrates the following
i. The technical feasibility of completing the intangible asset so that it will be
available for use or for sales;
ii. Its intention is to complete the intangible asset and use or sell it;
iii. Its ability to use or sell the intangible assets;
iv. How the intangible asset will generate probable future economic benefits. Among
other things, the entity can demonstrate the existence of a market for the output of
the intangible cost or the intangible asset itself or, if it is to be used internally, the
usefulness of the intangible asset;
v. The availability of adequate technical, financial and other resources to complete
the development and to use or sell the intangible asset;
vi. Its ability to measure reliably, the expenditure attributable to the intangible asset
during its development.
d. Factors to consider in the determination of the useful life of 1ntangible Asset
include the following
i. The expected usage of the asset by the entity and whether the asset could be
managed efficiently by another management team;
ii. Typical product life cycles for the asset and public information estimates of useful
lives of similar assets that are used in a similar way;
iii. Technical, technological, commercial or other types of obsolescence;
iv. The stability of the industry in which the asset operates and changes in the market
demand for the products of services output from the asset and the entity's ability
and intention to reach such a level;
v. The period of control over the asset and legal or similar limits on the use of the
asset, such as the expiry dates of related leases;
vi. Whether the useful life of the asset is dependent on the useful life of other assets
of the entity.

e. Calculation of goodwill on consolidation

N'000
Cost of investment in Subsidiary 299,70
Fair value of non-controlling interest 0
169,50
469,200
Net Asset at acquisition date (Subsidiary) 0
(345,800
Goodwill on consolidation Impairment )123,400
(62,200
)
61,200

EXAMINER'S REPORT
The question tests the principles and application of IAS 38 on Intangible Assets. Candidates are
required to differentiate between purchase and non-purchase goodwill, state the conditions
required for development cost to be recognized as an intangible asset, factors that must be
considered in determining the useful life of an intangible asset and characteristic of goodwill.
About 80% of the candidates attempted the question and performance was above average.
Some of the pitfall of the candidates was their lack of indept knowledge of the provisions of IAS
38.
Candidates are required to pay attention to the provisions of IFRS for better performance in
future.
SOLUTION 6

a. (i) The Equity Method is a method of accounting whereby the investment is initially
recognized at cost and adjusted thereafter for the post-acquisition change in the investor's
share of the investee's net assets.
The investor's profit or loss includes its share of the investee's profit or loss and the
investor's other comprehensive income includes its share of the investee's other
comprehensive income.
(ii) Discontinuing the Use of Equity Method
An entity shall discontinue the use of Equity Method from the date when its investment
ceases to be an associate or joint venture as follows:
• 1f the investment becomes a subsidiary, the entity shall account for its investment in
accordance with 1FRS 3 on Business combination and 1FRS 10.
• 1f the retained interest in the former associate or joint venture is a financial asset, the
entity shall measure the retained interest at fair value. The fair value of the retained
interest shall be regarded as its fair value on initial recognition as its financial asset in
accordance 1FRS 9. The entity shall recognize in profit or loss any difference
between:
- The fair value of any retained interest and any proceeds from disposing of a part
interest in the associate or joint venture;
- The carrying amount of the investment at the date the equity method was
discontinued.
- When an entity discontinues the use of the equity method, the entity shall account
for all amounts previously recognized in other comprehensive income in relation
to that investment on the same basis as would have been recognized if the
investee had directly disposed of the related assets or liabilities.
b. AGBANTARAGROUP
CONSOLIDATED STATEMENT OF PROFIT OR LOSS AND OTHER COMPREHENSIVE
INCOME FOR THE YEAR ENDED 31 DECEMBER, 2014
N'million
Revenue N(4,500 + 1,350 - 45) 5,805.00
Cost of sales N(2,430 + 720) - 38 (Wk 4) (3,112.00)
Gross profit 2,693.00
Administrative expenses N(1,350 + 180 + 10) 1,540.00
Operating profit 1,153.00
Finance income N(135 + 90) 225.00
Share of Associate Profit [(35% x N90) - N15] 16.50
Finance costs (N180) (180)
Profit before taxes 1,214.50
Income taxes N(225 + 135) 360.00
Profit for the year 854.50

Other comprehensive Income:


Items that may not be reclassified to Profit or Loss;
Gains on property valuation N(180 + 90) 270.00 15.75 285.75
Gains on property valuation of Associate (35% x N45) 1,140.25
Total Comprehensive Income

Profit for the year attributable to:


- Owners'of
Agbantara Plc. 699.30
- Non
Controlling Interest 155.20
854.50
Total Comprehensive Income attributable to:
- Owners'of
Agbantara Plc. Wk 5 949.05
- Non-Controlling Interest Wk 5 191.20
1,140.25
Workings:
Wk 1 - % Holding in Odinma Plc. by Agbantara Plc.

72 million on shares
120 million on shares
=60% (Subsidiary)
Wk 2 - %Holding in Dangari Ltd by Agbantara Plc.
35 million on shares
100 million on shares
=35% (Associate)

Wk 3 - Unrealised Profit on Sales of Goods


= 100% x (N45 million - N38 million)
= N7 million
Wk 4 - Inter Company Cost of Sales
= Inter-company Revenue - Unrealised Profit
= (N45 million - N7 million)
= N38 million
Wk 5 Profit/Total Comprehensive 1ncome Attributable to:

Agbantara NC1
Plc.
N'million N'million
Agbantara Plc for the year 450.00 -
Share of Post-acquisition Profit for the Year N405
million in the ratio 60:40 243.00 162.00
Unrealized Profit on 1nventory
N7 million in the ratio 60:40 (4.20) (2.80)
Share of Dangari Profit (35% x N90) 31.50 -
1mpairment Loss on Goodwill
N10 million in the ratio 60:40 (6.00) (4.00)
1mpairment loss on 1nvestment of Dangari (15.00) -
699.30 155.20

Other Comprehensive 1ncome:

- Gain on Revaluation - Agbantara Plc. 180.00 -


- Gain on Revaluation - Odunma Plc.
=^90 million in the ratio 60:40 54.00 36.00
- Gain on Revaluation - Dangari Ltd
(35% x N45 million) 15.75 -
949.05 191.20

Alternative Solution to Working 5

Calculation of NC1 N' m

Total comprehensive income 1,140.25

Attributable to:
Owners of the parent (balancing figure) 949.05
Non-controlling interests 4 191.20
1,140.25
Working 5

Calculation of NC1 OD1NMA N'm

On profit for the period N405m x 40% 162


On gains on revaluation of property N405 x N90 36
Unrealised profit on intercompany sales N7 x 40% (2.8
1mpairment of goodwill N10 x 40% )
(4.0
)
EXAM1NER'S REPORT 191.2
The question tests the provisions and application of 1AS 28 on 1nvestment in Associates.
Candidates are required to explain Equity Method of Accounting for Associates and joint
venture.
They are also required to prepare a Consolidated Statement of Profit or Loss and Other
Comprehensive 1ncome disclosing the treatment of Associates in the statement.
About 70% of the candidates attempted the question and performance was poor.
Although majority of the candidates were able to explain Equity Method of accounting correctly,
some could not prepare Statement of Profit or Loss and Other Comprehensive 1ncome correctly,
while others that prepared it correctly were unable to disclose the transactions relating to
Associates properly in the statement.
Similarly, most candidates failed to show the N'm sign in the solution and this led to loss of
marks.
Candidates are advised to pay special attention to all relevant 1FRS at this level of this 1nstitute
examination as Section C of the Financial Reporting paper is meant to test candidate's
knowledge and application of 1nternational Financial Reporting Standard (1FRS).
SOLUTION 7

Events after the reporting period are those events favourable and unfavourable, that occur
between the end of the reporting period and the date when the financial statements are
authorized for use.

Types of events
i. Adjusting events
These are events that provide evidence of conditions that existed at the end of
the reporting period.
ii. Non-Adjusting events
These are events that indicate conditions that arose after the reporting period.

Treatment of Events
i. Adjusting events - require adjustment in the financial statement. Examples are:
• Evidence of permanent diminution of property before the year-end;
• Insolvency of a customer with balance owing at the year end;
• Amount received or paid in respect of legal or insurance.

ii. Non-Adjusting events - do not require adjustment in the financial statement.


Examples are:
• Acquisition or disposal of a subsidiary after the year-end;
• Announcement of a plan to discontinue an operation;
• Major purchase and disposal of assets.

b. (i) IAS 10 provision confirms that the receipt of information after the reporting period
indicating that an asset was impaired at the end of the reporting period or that the amount
of a previously recognized impairment loss for that asset needs to be adjusted. In this
scenario, the bankruptcy of a customer that occurs after the reporting period usually
confirms that the customer was credit - impaired at the end of the reporting period.
Hence, company should adjust its financial statements for the year ended 30 June, 2014
for the probable loss (bad debts) of N40 million (i.e. 80% of N50 million) suffered as a
result of Company A insolvency as confirmed by the appointed liquidation on 30 July,
2014.
(ii) General Journal
DR CR
N'000 N'000
Bad debt (written off) 40,000
Trade receivable (Company A) 40,000
Being bad debt written off of N40 million
(i.e. 80% of N50 million) suffered as a result of Company A
insolvency as confirmed by the liquidator on 30 July, 2014

(iii) A fire incident occurring on 30 July, 2014 is a "Non-Adjusting Event" as the event
occurred subsequent to (or after) the reporting date (30 June, 2014).
The requirement of IAS 10 requires only a disclosure of the incident with respect to its
occurrence (i.e. the nature), the circumstances and the financial effects, unless if it is so
material that it may affect the going concern status of Company B.
c. If an entity declares dividend to equity shareholders (as defined in IAS 32 - Financial
Instruments Presentation) after the reporting period, the entity shall not recognize those
dividends as a liability at the end of the reporting period. If dividends are declared after
the reporting period but before the financial statements are authorized for issue, the
dividends are not recognized as a liability at the end of the reporting period because no
obligation exists at that time. Such dividends are disclosed in the notes in accordance
with IAS 1 - Presentation of Financial Statements.

EXAMINER'S REPORT
The question tests the principles and application of provisions of IAS 10 on Event After
Reporting Date, IAS 37 on Provision and IAS 1 on Presentation of Financial Statement.
Only about 40% of the candidates attempted the question and performance was below average.
The common pitfalls include:
• Difficulty in differentiating between adjusting and non-adjusting events
• Inability to raise current journal entries to reflect the appropriate adjusting entries.
• Failure to realize that dividend not yet approved at the Annual General Meeting should
be disclosed by way of notes and should not be recognized as liability in the Statement of
Financial Position as required by 1AS 37 on Provisions.
Candidates are required to be more familiar with all relevant 1FRS at this level of the 1nstitute's
examinations for better performance in future examinations.
THE INSTITUTE OF CHARTERED ACCOUNTANTS OF NIGERIA
SKILLS LEVEL EXAMINATION - MAY 2016 FINANCIAL
REPORTING
Time Allowed: 3hours
ANSWER FIVE OUT OF SEVEN QUESTIONS IN All
SECTIONA: COMPULSORY
QUESTION 1
GBENGANIG Pic. Trial balance as at December 31, 2015 is shown below:

(30MARKS)

N N
Revenue 2,290,125
Administrative expenses 237,150
Selling and distribution expenses 175,200
Legal and professional expenses 81,150
Allowance for receivables - 31/12/15
8,625
Inventories -finished goods - 31/12/14 276,750
Work-in-progress - 31/12/14 49,125
Inventories - raw materials at cost-31/12/14 162,600
Purchases - raw materials 1,125,900
Carriage inwards - raw materials 15,750
Manufacturing wages 375,000
Manufacturing overheads 187,500

Authorised and issued 900,000 ordinary shares of N0.50 each


fuiiy paid 450.000
150,0 8.4% cumulative preference shares of Ni each 150.000
fuiiy paid 65.000
Revaluation surplus Share 150.000
premium Generai reserve 85.0
Retained earnings-31/12/14 425,250
Patents and trademarks 323,250
Freehold property at cost
375,000
Leasehold property at cost
112,500

Amortization of leasehold • property - 31/12/14 Plant and 22,500


equipment at cost
225, 000
Accumulated depreciation - plant and equipment-
31/12/14 102,750
Furniture and fittings at cost 75,000
Accumulated depreciation - furniture and fittings - 23,625
31/12/14
Motor vehicles at cost 112,500
Accumulated depreciation - motor vehicles 31/l2/14 37,500
10% loan notes 150,000
Trade payables 146,250
Trade receivables 266,445
Bank overdraft 76,875
Cash 7,680 ______
4.183.500 4,183500
(i) A gain of N20,000 made on the revaluation of old freehold property during the year is
yet to be accounted for.
(ii) Inventories at December 31, 2015 were:

N
Raw 168,900
materials 413,025
Finished 56.700
goods Work-
in-progress
(iii) Legal and professional expenses include:
Solicitor‟s fees for purchase of new freehold land during the year N7,500.
(iv) Provision is to be made for the full year's interest on the loan notes.
(v) The leasehold land and buildings - are held on 50years lease, with 40 years
unexpired life left as at the end of December 31, 2014.
(vi) Depreciation for the year is to be charged as follows:
- Plant and equipment 8% on cost - charged to production
- Furniture and fittings 10% on cost - charged to administration
- Motor vehicles 20% on carrying amount - charged -25% to administration and 75% to
selling and distribution.
(vii) Income tax on the profit for the year is estimated at N68,900 and is due for
payment on February 28, 2016.
(viii) Dividend of N2.25k per ordinary share capital is recommended for payment by the
directors. No dividend was paid in the previous financial year.
Required:

a. Prepare the statement of profit or loss and other comprehensive income for the year
ended December 31, 2015.
b. A statement of financial position as at December 31, 2015 in accordance with International
Financial Reporting Standards.
(Total 30 Marks)

SECTION B: ANSWER ANY TWO OUT OF THREE QUESTIONS IN THIS SECTION (40
MARKS)
QUESTION 2
a. A Parent Company acquired 60% equity interest in a subsidiary company for N440million.
The market value of the net assets of the subsidiary on acquisition date was N400million.
The parent company estimates that the full 100% interest in the subsidiary company would
have cost N640million.
Required:
a. Calculate the goodwill at acquisition date where non-controlling interest is
measured:
i. As a proportionate share of the net assets of the subsidiary company.
ii. At fair value (the full good will method). (5 Marks)

b. The statement of financial position of PAPA Pie and MAMA Pie as at December
31, 2015 were as follows:-

PAPA PLC MAMA


N'OOO N'OOO
PLC
Property Plant & Equipment 9,000 5,000
Investment in MAMA Pie. 5,000
Other Assets 2,000 1,500
16.000 6500

Share Capital 500 500

Retained Earnings 14,500 5,000


Other liabilities 1,000 1,000
16,000 6,500

PAPA Plc acquired 80% equity interest in MAMA Plc two (2) years ago.
At the date of acquisition MAMA's retained earnings stood at N3million and the fair value
of its net assets was N5million. This was Nl.5million above the carrying amount of the net
assets at this date. The fair value adjustment related to an asset that had a remaining useful
economic life of 10 years as at the date of acquisition.
The goodwill arising on consolidation has not suffered any impairment.

Required:

Prepare consolidated statement of financial position of PAPA Pie Group as at December


31, 2015, on the assumption that non-controlling interest is valued at
fair value (the full goodwill method). (15 Marks)
N.B: Show all workings. (Total 20
Marks)

QUESTION 3
On October 1, 2015 Panda purchased 75% of the equity shares in Sanda. The acquisition was
through a share exchange of two shares in Panda for every three shares in Sanda.
The stock market price of Panda's shares at October1, 2015 was N6 per share.
The summarized statements of comprehensive income of the two companies for the year ended
March 31, 2016 are:

Panda Sanda
N'OOO N'OOO
Revenue 675.0 3
Cost of sales 60,000
Gross profit (390,000) (165,000)
Distribution costs 285.0 1
Administrative expenses 95,000
Finance costs (35,400) (18,000)
Profit before tax (40,500) (34,500)
Income tax expense (2,250) (1,800)
Profit for the year 206.850 1
40,700
(72,000) (41,7OO)
Panda Sanda
134.850 9
N'OOO N'OOO
9,900
Other comprehensive income Gain on
revaluation of land(note(i)) 3.750 1.500
Loss on fair value of equity financial asset (1,050) (600)
Investment 2,700 900
Total comprehensive income 137,550 99,900
The following information is relevant:
(i) The equity of the companies at October l, 2015 (i.e. just before the share exchange took
place) is available:

Panda Sanda
N'OOO N'OOO
Equity shares of Nl Each 375,000 240,000
Share premium 150,000 Nil
Revaluation reserve (land) 12,600 Nil
Retained earnings 135,000 220,500

(ii) Immediately after the acquisition of Sanda on October 1, 2015, Panda transferred an item
of plant with a carrying amount of N4million to Sanda at an agreed value of N7.5million.
At this date the plant had a remaining life of two and half years. Panda had included the
profit on this transfer as a reduction in its depreciation costs. All depreciation is charged to
cost of sales.
(iii) After the acquisition Sanda sold goods to Panda for N60million. These goods
had cost Sand a N45million. N18million of the goods sold remained in Panda's closing
inventory.
(iv) Panda's policy is to value the non-controlling interest of Sanda at the date of acquisition at
its fair value which the directors determined to be N150million.
(v) The goodwill of Sanda has not suffered any impairment.
(vi) All items in the above statements of comprehensive income aredeemed to accrue
evenly over the year unless otherwise indicated.
Required:
a. Show your computation of the amount paid by Panda and the goodwill arising
on the acquisition of Sanda on October 1, 2015. (6 Marks)
b. Prepare the consolidated statement of comprehensive income of Panda for the
year ended March 31, 2016. (14 Marks)
(Total 20 Marks)
QUESTION 4
Quintet Plc sells provisions through its stores located in various retail shopping centres in the
major cities in Nigeria. It has of recent been experiencing declining profitability and the board is
wondering if this development is peculiar to the company or related to the sector as a whole. It is
also not clear as regards the company's solvency. As such it has engaged a consulting firm that
specializes in analyzing corporate reports producing average ratios across many businesses to rate
performance. Below are the ratios that have been provided by the consulting firm for Quintet Plc.'s
business sector based on the year end of June 30, 2015.

Debt to equity 38%


Gross profit margin 35%
Operating profit margin 12%
Return on year -end capital employed (ROCE) 16-8%
Net asset turnover 14 times
Current ratio 125:1
Average inventory turnover 3 times
Trade payables' payment period 64 days

The financial statements of Quintet Plc for the year ended September 30, 2015 are:

Income statement
N'OOO N'OOO
Revenue 224,000
Opening inventory 33,200
Purchases 175,600
208,800
Closing inventory (40.800) (168,000)
Gross profit 56,000
Operating costs (39,200)
Finance costs (3,200)
Profit before tax 13,600
Income tax expense (4,000)
Profit for the year 9,000
Statement of financial position
Assets
Non-current assets N‟OO N‟OOO
Property and shop fittings O
Deferred development expenditure
Current assets 102,400
Inventory 20,000 122,400
Bank 40,800
Total assets 4,000
Equity and liabilities 44,800
Equity 167,200
Equity shares of N1each Property
revaluation reserve Retained 60,000
earnings 12,000
Non-curentliabes10% loan notes Curentliabes 34,400
Trade payables Current tax payable 106,400
Total equity an liabilities 32,000

21,600
7,200 28,800
167,200

Note:
(i) Net asset is defined by the consulting firm as total assets less current liabilities.
(ii) The deferred development expenditure relates to one off payment for a franchise as a sole
distributor of a particular product under negotiation but not yet concluded as at September
30, 2015 although payment has already been made.
Required:
a. Compute the equivalent ratios for Quintet Plc that have been provided by the
consulting firm for the business sector. (9 Marks)
b. Write a report to the board which assesses the profitability and solvency performance of
Quintet Plc in comparison to its business sector averages. For clarity solvency is a
measure of liquidity and gearing. (11 Marks)
N.B: Ignore Income Tax (Total 20 Marks)
SECTIONC: (30 MARKS)
ANSWER ANY TWO OUT OF THREE QUESTIONS IN THIS SECTION

QUESTION 5
a. As one of the accountants of Oiuwaseun Pic, a company which has migrated to IFRS, you
are aware that IAS 8 Accounting Poiicies, Changes in Accounting Estimates and Errors
contains guidance on the use of accounting poiicies and accounting estimates.
Requird:
Expiain the basis on which the management of an entity such as Oiuwaseun Pic must
seiect its accounting poiicies and distinguish, with an exampie, between changes in
accounting poiicies and changes in accounting estimates.
(6 Marks)
b. The directors of Oiuwaseun Pie. Are disappointed by the draft profit for the year ended
September 30, 2015. One of your staff, who is an assistant accountant, has suggested
one area where he beiieves the reported profit may be improved, if it is acceptabie to the
company's management.
Inciuded in the financiai statement of Oiuwaseun Pic is an item of piant which had cost
N80miiiion to purchase and instaiied three years ago on October 1, 2012. It is the poiicy
of Oiuwaseun Pic to depreciate this piant on a straight-iine basis over a five-year period,
assuming anii residuai vaiue. The depreciation of the piant has progressed as envisaged,
but at the start of the current year (October 1, 2014) the production manager estimated
that the piant was iikeiy to iast eight years in totai from the date of its purchase as
against the originai five year period upon which current depreciation is based.
The assistant accountant has calculated that, based on an eight-year life (with no residual value)
the accumulated depreciation of the plant at September 30, 2015 would be N30million
(N80million/8 years x 3). In the financial statements for the year ended September 30, 2014, the
accumulated depreciation was 32million (N80million/5 year x 2). Therefore, by adopting an
eight-year life, Oluwaseun can avoid making a depreciation charge in the current year and
instead N2million (N32million-N30million) from the accumulated depreciation account will be
credited to the income statement in the current year to improve the reported profit.
Required:
i. Comment on
the acceptability of the assistant accountant's suggestions.
(6 Marks)
ii. Illustrate how the suggestions will affect the financial statements of
Oluwaseun Plc based on the correct application of the relevant IFRS.
(9 Marks)
(Total 15 Marks)
QUESTION 6

The statement of financial position extract of Apapta Limited is given as follows:

2015 2016
N'000 N'000
Inventories 3,950 3,250
Receivables 2,151 2,675
Investments (Marketable Securities) 375
4
3
0

Cash 565 -
7,460 6,300
Payables amounts due within one year (3,865) (3,755)
3,595 2,545

Payables are analysed as follows: Trade payables 2,600 2,215

Company Income Tax 695


8
2
0

Dividend payable 570


5
4
0

Bank overdraft - 180


3,685 3,755
Its profit or loss account extract is as follows:
Sales 17,795 16,715
Cost of sales (12,100) (11,200)
Gross profit 5,695 5,515
Cost of sales is analysed as follows:
Opening inventory 3,250 3,150
Add: Purchase 12,800 11,300
Less: Closing inventory (3,950) (3,250)
12,100 11,200

In 2014 and 2015, credit sales were 83% of total sales.


Required:
a. Calculate the working capital cycle for 2015 and 2014. (9 Marks)
b. Compute the ratios listed below and comment on the company's
liquidity over the two years.
i. Cash ratio
ii. Current ratio
iii. Quick ratio (6 Marks)
(Total 15 Marks)
Question 7
a. After initial recognition in the Financial Statements, Financial Assets are measured,
either at fair value or amortised cost according to the provisions of IAS 39-financial
Instruments; Recognition & Measurement.
Required:
Briefly explain how fair value and amortised costs of financial assets are determined
and give one example each of the class of financial assets that can be measured
using the methods. (6 Marks)
b. Anifowose Plc issued a debt instrument at it fair value of N100million on January 1,
2013. The debt instrument is to mature in 2017. It has a principal amount of
N125million and carries fixed interest rate of 4.72%
that is paid annually. The effective interest rate is 10% and on December
31, 2015 had a fair value of 105 for every N10 nominal value.
The company makes up its accounts to December 31, every year.
Required:
i. Show your computation schedule for the amortised cost of the financial
liability up to December 31, 2015 on the assumption that the financial liability
is valued at amortised cost.
ii. What is the value of the financial liability as a December 31, 2015, if the fair
value option is adopted by Anifowose Plc? (9 Marks)
(Total 15Marks)
THE INSTITUTE OF CHARTERED ACCOUNTANTS OF NIGERIA
PROFESSIONAL LEVEL EXAMINATIONS - MAY 2016 FINANCIAL
REPORTING

SOLUTION 1
Gbenga Nig. Plc.
Statement of Profit or Loss and other comprehensive income for the year
ended December 31, 2015

Note N N
Revenue Cost of Sales Gross Profit Less 2,290,125
Expenses: 1 (1,572,000)
Administrative expenses 718,125
Selling and distribution expenses 2 324,300
Finance cost 3 186,450
Profit before taxation 4 27,600 538,350
Income tax expense 179,775
Profit for the year (68,900)
Other comprehensive income 110,875
Items that cannot be reclassified to
profit or loss
Gain on revaluation of freehold property Total
comprehensive income EPS = N0.12 i.e 12
kobo
20,000
130,875
Gbenga Nig. Plc.
Statement of Financial Position as at December 31, 2015
Assets Note N
N
Non-Current Assets:
698,375
Property, Plant and equipment 5
323,250
Patents and trademarks
1,021,625
Current assets:
Inventories
Trade receivables (N266,445 - 8,625)
Cash and cash equivalents
Equity and liabilities: 6 638,625
Equity 257,820
Authorised and issued 900,000 ordinary shares of 7,680
904.125 1
50k each ,925,750
Share premium N
Revaluation surplus (N65,000 + 20,000) 450.000
General Reserve 150.000
Retained earnings (N425,250 + 130,875) 85.000
Non-current liabilities 85.000
10% loan notes 536.125
8.4% cumulative preference shares of N1 each Current 1,306,12
liabilities: 5
Trade payables Income tax expense 150.000
Bank overdraft Interest on loan 150.000
Preference shares dividend

146,250
68,900
76,875
15,000
12,600

319,625
1,925,750
Notes/Workings:
1. Cost of Sales: N
Opening Inventory- Raw materials 162,600
- Work in progress 49,125
- Finished goods 276,750
Purchases of raw materials 1,125,900
Carriage inwards - raw materials 15,750
Manufacturing wages 375,000
Manufacturing overhead 187,500
Depreciation - Plant and equipment (wk 7) 18,000
Closing Inventory - Raw materials 168,900
- Work in progress 56,700
- Finished goods 413,025 (638,625)
1,572,000
2. Administrative expenses: N
Per trial balance 237,150
Legal and professional expenses (N81,150 - 7,500) 73,650
Depreciation (wk 7) 11,250
Authorisation of leasehold property (wk 8) 2,250
324,300
3. Selling and distribution expenses: N
Per trial balance 175,200
Depreciation (wk2) 11,250
186,450
4. Finance Cost: N
Interest on loan notes at 10% 15,000
Preference shares dividend at 8.4% 12,600
27,600
5. Property, plant and equipment (PPE)
Freehold Leasehold Plant & Furniture Motor Total
property Property Equipment & Fittings Vehide N
N N N N N 900,000
Cost Jan 1, 375,000 112,50 225,000 75,000 112,500 7,500
2015 7,500 0 20.000
Solicitors fees 20,000 927,500
Revaluation 402,500 (186,375)
gain
(42.750)
698.375
Production
Accumulated - (22,500) (102,750) (23,625) (37,500)
N
Dep. Jan 1
18,000
Depreciation/
amount for the 18.000
- (2,250) (18,000) (7,500) (15,000)
year
Carrying amount
Dec 31 2015
, 402.500 87.750 104.250 43.875 60.000

N
6. Inventories:
Raw materials 168,900
Work-in- 56,700
progress Finished 413.025
goods 638,625

7. Depreciation for the year: Admin Selling &


Dist.
N N
Plant and Equipment @ 8% on - -
cost
Furniture & Fitting " 10% on cost 7,500 -
Motor vehicle @ 20% on C. A 3.750 11.250
11.250 11.250
8. Amortisation of ieasehoid property:
N112.500 - N22.500
40 years
= N2.250
9. Earnings per share EPS:
N 110,875
900,0 shares
= 0.12 i.e 12kobo
10. An ordinary dividend of N2.25 per share amounting to N2,025,000 was
proposed during the year.
MARKING GUIDE
a) STATEMENT OF PROFIT OR LOSS AND OTHER COMPREHENSIVE
INCOME FOR Marks
THE YEAR ENDED DECEMBER 31, 2015:
- Caicuiation of Gross Profit:
Titie of statement
Revenue
Cost of Saies
Gross profit
- Determination of profit before taxation:
Administrative expenses
Seiiing and distribution expenses
Finance Cost
Profit before taxation Vs
- Determination of profit for the year: Vs
Income tax expense Vs
Profit for the year Vs ^Vs
- Ascertainment of Totai Comprehensive income Vs
1
Gain on revaiuation of freehoid property /s
1
Totai comprehensive income /s
EPS presentation V_
- Workings/caicuiation of figures for: s
1
/s ^Vs
Cost of saies
Vs
Administrative expenses 1
/s
1 2
/s /s
1
/s

4
12/
s
Selling and distribution expenses 1
Finance cost 1 72/3
b) STATEMENT OF FINANCIAL POSITION AS AT DECEMBER 31, 2015:
- Presentation of non-current assets:
Title of statement 1/3
1
Property, plant and equipment (PPE) /3
1
Patents and trademarks /3
1
Total non-current assets /3 1 1 /3
- Determination of total current assets:
1
Inventories /3
1
Trade receivables /3
1
Cash and cash equivalent /3
1
Total current assets /3
1
Total assets /3 12/3
- Presentation of equity:
1
Authorised and issued share capital /3
1
Share premium /3
1
Revaluation surplus /3
1
General reserve /3
1
Retained earnings /3
1
Total equity /3 2
- Presentation of non-current liabilities
1
10% loan notes /3
1 2
8.4% cumulative preference shares /3 ^
- Presentation current liabilities:
1
Trade payables /3
1
Income tax /3
1
Bank overdraft /3
1
Interest on loan /3
1
Preference shares dividend /3
1
Total current liabilities /3
1
Total equity and liabilities /3 2 1 /3
- Workings/Calculation of figures for:
Property, plant and equipmentschedule 6
Inventories at year end 1
Depreciation/amortization 2 1 /3
1
Earnings per share /3
1
Note on proposed dividend /3 10
TOTAL MARKS FOR THE QUESTION ^30
Examiners' Report
The question tests candidates' knowledge of the presentation of financial statements. Candidates
are required to prepare a statement of profit or loss and other comprehensive income and a
statement of financial position of a company. Majority of the candidates attempted the question
as it is compulsory and the performance was fair. About 55% of the candidates who attempted
the question obtained 50% of the mark allocated.
The commonest pitfall is the inability of some candidates to present the financial statements in
an appropriate format. Some also lost valuable marks due to wrong calculation of the amount for
cost of sales.
Candidates are advised to study and practice past examination questions on the preparation and
presentation of financial statements.

SOLUTION 2
a) i. Calculation of Goodwill at acquisition:

Nmillion
Consideration transferred 440
NCI share of subsidiary net
asset (40% of N400 million)
160
60
Less Net asset of subsidiary at acquisition 0
Goodwill (40
0)
20
ii. Calculation of Goodwill at acquisition with fair value of NCI:
0
Nmillion
Consideration transferred 440
Fair value of NCI at acquisition (40% 200
- 640) 64
Less Net asset of subsidiary at 0
acquisition Goodwill (40
0)
24
0
b) Papa PlcGroup
Consolidated Statement of Financial position as at Dec. 31, 2015

Assets: N'000
Property, plant and equipment (PPE) 15,20
Goodwill on consolidation 0
Other assets 1,000
Equity and liabilities: 3,500
Share capital 19,70
Retained earnings 0
Non-controlling interest 500
Other liabilities 15,86
0
16,36
0
1,340
19,700
2,000

Workings
1) Calculation of Net asset of subsidiary - Mama
At
Consolidation
At Post
N'000
Acquisition Acquisition
500
N'000 500 3,000 N'000

Share capital
Retained earnings 5000
Extra depreciation
on FV adjustment
N1,500 x 2/10 (300)
years 4,700 3,000 1,700
Fair Value Reserve
1,500 1,500
6,700 5,000
1,700
2) Goodwill on consolidation

N'000
Consideration transferred 5,000
Fair value of NCI at acquisition (20% of 1,000
N5,000)
6,000
Less Net asset of Mama at acquisition (5,000)
1,000

3) Non-controlling interest
N'000
Fair value at acquisition (20% of N5,000) 1,000
NCI share of post acquisition reserve (20% of 340
N1,700)
1,340

4) Consolidated Retained earnings:


Papa Group N'000 Mama
Retained earnings per question 14,500 N'000
Less: FV extra depreciation adjustment - 5,000
Pre-acquisition reserve - (300)
Group share of post acquisition reserve at 80% 1,360 (3,000)
of N1,700 1,700
15,860
5) Property, Plant and Equipment (PPE)
Group N'000 Mama N'000

Per question 9,000 5,000


FV adjustment 1,500
Extra depreciation on FV adjustment (300)
6,200 6,200

15,200
TUTORIAL NOTE
IFRS 3 Business Combination allows permits Non-controlling interest to be valued at
fair or dull goodwill method. This holds usually where the market value of non-
controlling interest can be independently determined example for a listed subsidiary.
The question gives the fair value of the 60% subsidiary's shares acquired by the parent
at N440million. But the cost of the full 100% interest in the subsidiary would have been
N640million. This is an indication that the fair value of the entire 100% is N640million.
Applying the fair or full goodwill method means that fair of non-controlling interest will
be (640 - 440) i.e. N200million which is the value of the 40% non-controlling interest.
Almost all the student correctly identified the N640million as the fair value of the
subsidiary. However, some incorrectly tried to calculate the fair value of non-
controlling interest as 40% x N640million i.e. N256million. It should be noted that fair
value is not necessarily a market value. To this recall that according to IFRS 13, Fair
Value Measurement, Fair value is the price that would be received to sell an asset or
paid to transfer a liability in an orderly transaction between market participants at the
measurement date (i.e. it is an exit price). Thus the N440million paid is the fair value
of the parent and N200million to non-controlling interest. This explains why there can
be a bargain purchase in acquisition of subsidiary.
MARKING GUIDE
(a) CALCULATION OF GOODWILL AT ACQUISITION
(i) Consideration transferred i/2
Determination of NCI share of net assets 1
Net asset of subsidiary at acquisition i/2 Marks
Determination of Goodwill i/2 2l
/2

(ii) Consideration transferred l


/
Determination of fair value of NCI at acquisition
2
1
Net assets of subsidiary at acquisition i/ 2
Determination of goodwill i/ 2 2V2
5
(b) PREPARATION OF CONSOLIDATED
STATEMENT OF FINANCIAL POSITION
Determination of Property, Plant & Equipment (PPE) 22/B
Determination of Goodwill on Consolidation 31/3
1
Calculation of other Assets /B
1
Calculation of Share Capital /B
2
Determination of Retained Earnings 5 /B
Determination of Non-Controlling Interest (NCI) 1 2^
1
Calculation of other Liabilities /B
2
Calculation of Total Asset and Total Liabilities ^ 15
TOTAL MARKS FOR THE QUESTION 20

Examiners' Report
The question tests candidates' ability to calculate goodwill on the basis of two different
assumptions that is where non-controlling interest is measured as a proportionate share of the net
asset of the subsidiary or at fair value. Candidates are also required to prepare consolidated
statement of financial position on the assumption that noncontrolling interest is valued at fair
value.
Majority of the candidates attempted the question but performance was below average. Only
30% of the candidates that attempted the question scored 50% of the marks allocated. The
commonest pitfall is the inability of some candidates to calculate goodwill using the two
different assumptions while others could not correctly reflect the effect of fair value adjustments
on the assets. Some candidates also lost valuable marks due to non indication of appropriate
naira symbols (N'000) despite advice given in past examiners reports.
Candidates are advised to cover all aspects of the syllabus and prepare adequately before
presenting themselves for examinations.
SOLUTION 3
a)
i) No. of shares acquired in Sanda = 75% of 240 million shares
= 180 million
shares No of shares issued by Panda in exchange
180 millionshares _
=------------------- x 2
3
= 120 million
shares :. Amount paid = 120 million shares x N6
per share
= N 720 million
ii) Calculation of Goodwill on acquisition:
N'000

870,000
Less: Net asset of Sanda at acquisition:

409,500

Consolidated Statement of Comprehensive income for the year ended March 31, 2016
Note N'000 N'000
Revenue (N675,000 + 180,000 - 60,000) 795,000
Cost of sales 1 (419,800)
Gross profit 375,200
Distribution costs (35,400 + 9,000) (44,400)
Administrative expenses (40,500 + 17,250) (57,750)
Finance Costs (N2,250 + 900) (3,150)
Profit before taxation 269,900
Income tax expense (N72,000 + 20,850) (92,850)
Profit for the year 177,050
Other comprehensive income:
Gain on revaluation of land (N3,750 + 1,500) 5,250
Loss on fair value of equity financial asset investment (N1,050 + 600) (1,650)
Total comprehensive income 180,650
Profit attributable to:
Owners of the parent Non- 165,80
controiiing interest at 25% 0
Totai comprehensive income 11,250
attributabie to: Owners of the parent 177,05
Non-controiiing interest at 25% 0169,17
5
11,475
180,65
0
b) Aiternate solution
Panda Group
Consoiidated Statement of Comprehensive income for the year ended March 31, 2016

Sanda Dr Consoiidated
Panda Cr
N'000 N'000 N'000
N'000 N'000
(60,000)
Revenue Cost of Saies 675.0 ( 180,000 60,700 795,000
(82,500 (8,000) (419,800)
Gross profit 390,00
) 375,200
Distribution cost 0)
97.500
Administrative expenses 285.0 (44,400)
(35,40 (9,000 (68,000) 60,700 (57,750)
Finance costs
) (3,150)
Profit before taxation 0)
(17,250 269,900
Income tax expense (40,50
0) ) (92,850)
Profit for the year
(2,250) (900) 177,050
Other comprehensive income:
Gain on revaiuation 206.850 ( 70,350 (68,000) 60,700 5,250
(20,850
Loss on fair vaiue of equity 72,000
) )
instrument (68,000) (60,700)
134.850 49.500
Totai Comprehensive income
1,500
Profit attributabie to: 3,750
(1,050) (600)
Owners of the Parent Non-
controiiing interest at 25% 137,550
(1,650)
50,400 (68,000) 60,700 180,650
165,800
11,250
177,050
Total comprehensive income
attributable to:
Owners of the parent Non- 169,17
controlling interest at 25% 5
11,47
5
180,65
0
Workings:
1) Cost of sales: N'000
390,0
Per question - Panda 00
- Sanda (165,000 x 6/12 ) 82,50
Adjustment for inter group sales/purchase 0
(60,000)
Adjustment for unrealized profit in inventory
Adjustment for unrealized profit on transfer of 4.5
plant 00
Adjustment for excess depreciation charge 3.500
1
419,800
2
0
2) Unrealised profit on transfer of plant: 0
Proceed N7.5m less carrying amount N4m 1 = N3.5million
3) Unrealised profit in inventory
Total profit on sale = Sales N60m less cost of sales N45m = N15million
Unrealised Profit = N15m x
N60m
= N4.5million
4) Excess Depreciation charge

x 1 = N700,000

5) Non-controlling interest at 25%


On profit for the year = 25% of N49,500,000 less N1,125,000 share of
unrealized = N11,250,000

a) On Total comprehensive income:


25% of N50, 400,000 less N1,125,000 share of unrealized
= N11,475,000
Tutorial:

Panda's acquisition of the Sanda on October 1, 2015 happened at the middle of the current
accounting year. Therefore, the post acquisition period is 6 months (October 1, 2015 to
March 31, 2016).
For consolidation purpose, only the post acquisition proportion /part of the incomes and
expenses of Sanda is relevant. That is Panda's figure plus V^of Sanda's figure.
However, for gain on revaluation and loss on fair value reported under other
comprehensive income, the amounts from Sanda are not apportioned because the
transactions occurred after Panda's acquisition of Sanda.
MARKING GUIDE
(a) DETERMINATION OF CONSIDERATION TRANSFERRED Marks
AND GOODWILL ON CONSOLIDATION:
- Calculation of consideration transferred 2
- Calculation of Goodwill on consolidation 4 6
(b) CONSOLIDATED STATEMENT
OFCOMPREHENSIVE INCOME FOR
THE YEARENDED MARCH 31, 2016:
- Determination of Gross profit:

Title of the statement i/2


Revenue i/2
Cost of Sales i/2
Gross Profit l/2 2
Determination of profit before taxation:
Distribution costs l/2
Administrative expenses 1/2
Finance costs 1/2
-
Profit before taxation /2 2
Determination of profit for the year:
1
Income tax expense /2
1
Profit for the year /2
1
Profit attributable to owners of parent /2
1
Profit attributable to non-controlling interest /2 2
Determination of Total Comprehensive
Income:
Gain on revaluation of land 1/2
Loss on fair value of equity financial asset i/2
Attributable to owners of parent ^
Attributable to non-controlling interest ^ 2
Workings/Calculation of figures for:
Cost of sales 3V2
Unrealise profits 1
Excess Depreciation ^2
Non-Controlling Interest 1 _6
TOTAL MARKS FOR THE QUESTION 20

Examiners' Report
The question tests candidates' knowledge of group accounts. Candidates are required in part (a)
of the question to compute the consideration transferred by the parent and also to determine the
goodwill arising on acquisition. The part (b) required candidates to prepare a consolidated
statement of comprehensive income.
Many candidates attempted the question and the performance was good as over 65% of the
candidates who attempted the question obtained above 50% of the marks allocated. The
commonest pitfall is the wrong calculation of the consideration transferred by the parent for the
acquisition of shares in the subsidiary.
Candidates are advised to study in-depth all aspects of group accounts in the syllabus for this
paper.
SOLUTION 4
a) Calculation of equivalent ratios
i) Debt to Equity:
Debt v 100 _ N32,000 v 100
x— x
Equity 1 N106,400 1
— 30%

ii) Gross Profit Margin


Gross Profit v 100 _ N56,000 v 100 x
x — x)
SaJes 1 N224,000 1
— 25%
iii) Operating profit margin
PBIT or Operating profit v100 N16,800
v100
x— x
SaJes 1 N224,000 1
— 7.5%

iv) Return on Capital Employed (ROCE)

PBIT v100 _ N16,800 v 100


x— x
Capita? EmpJoyed 1 N138,400 1
— 12%
v) Net Asset Turnover
Sales — N224,000
Net ^sset N138,400
— 1.6 Times

vi) Current Ratio


Current Assets N44,800 v 100
— x
Current JiabiJities N28,800 1
— 1.55 : 1

vii) Average Inventory Turnover


Cost of Sales —N168,000 x100
— x
^rera^e /nrentory N37,000 1
— 4.54 Times
viii) Trade Payable Payment Period

Trade Payable N21,600


Credit Purchasesx 365 days = ------------- x 365 days
s N175,6
AH n c. cnn s
00
= 44 days
b) Report to the Board
From: Chief Accountant
To: Board of Directors
Date: 17th May, 2016
Subject: Report on Profitability and Solvency Performance of Quinet
Plc as at June 30, 2015
This Report is an assessment of the profitability and solvency situation of Quinet
Plc in order to ascertain what is responsible for the recent declining performance.
The assessment is based on a comparison of the relevant performance indices
of our company with those of the sector in which we operate as summarized
below:

Ratio Quinet Industry


Plc
Debt to equity 30% 38%
Gross profit margin 25% 35%
Operating profit margin 7.5% 12%
ROCE 12% 16.8%
Net Asset Turnover 1.6 Times 1.4
Times
Current ratio 1.56:1 1.25:1
Average inventory turnover 4.5 Times 3 Times
Trade payable payment 45 days 64 days
period
Profitability
All the profitability ratios of our company namely - gross profit margin, operating
profit margin and ROCE, are lower than the corresponding industry ratios. This is
an indication that there is a room for significant improvement in Quinet's
profitability to catch up with its competitors in the sector. This is particularly
striking considering the fact that Quinet turns its net assets and inventory more
often than the industry does. Thus, an indication that the lower profitability of
Quinet Plc is very likely to be due to under-pricing of its products/services.
Solvency
The short term and long term solvency of Quinet Plc seem to be better than what
obtains in the industry as indicated by the company's current ratio, trade payable
payment period and debt to equity ratios which are better than the industry's.
In conclusion, Quinet Plc's solvency seem to be in good shape despite the lower
profitability which can be remedied with appropriate upward adjustment of its
selling prices.
MARKING GUIDE
(a) COMPUTATION OF EQUIVALENT RATIOS Marks
- Computation of each Equivalent Ratios
(8 Ratios @ ^mark each) 8
- Presentation of solution 1 9

(b) WRITING OF REPORTS


- Comparison of Ratios calculated with Industrial Ratios 1
- Correct Interpretation of profitability position with 3
relevant reasons
- Correct interpretation of solvency situation with relevant 3
reasons
- Appropriate opening paragraph 1
- Correct closing remarks 1
- Presentation in Report format 2
(Memo for or letter form) 11
TOTAL MARKS FOR THE QUESTION
20
Examiners' Report
The question tests the abiiity of the candidates' to compute equivaient industriai ratios and to aiso
use the ratios computed to carry out financiai anaiysis in terms of profitabiiity and soivency.
About 95% of candidates attempted the question and performance was average. The commonest
pitfall of the candidates was their inability to use the ratios computed to carry out correct
interpretation of the financial statements.
Ratio analysis and interpretation of financial statement is a regular feature at the skill level of the
Institute's examination. Candidates are therefore advised to pay more attention to this section of
the syllabus in order to improve on their performance in future examinations.
SOLUTION 5
a) According to the provisions of IAS 8 - Accounting policies, changes in accounting
estimates and errors:
Accounting policies are the specific principles, bases, conventions, rules and
practices applied by an entity in preparing and presenting financial statements.
The basis on which management of an entity must select its accounting policies are
as follows:
i) When an IFRS specifically applies to a transaction, other event or condition,
the accounting policy or policies applied to that item shall be determined by
applying the specific IFRS.
ii) In the absence of an IFRS that specifically applies to a transaction, other event
or condition, management shall use its judgment to develop and apply an
accounting policy that results in the financial statements providing reliable
and more relevant information about the effects of such transaction, other
event or condition.
A change in accounting policy involves a change in principle, bases, rule or
practice applied while a change in accounting estimate involves an
adjustment of the carrying amount of an asset or liability, or the amount of the
periodic consumption of an asset resulting from new information or new
development. An example of change in
accounting policies include; adopting the revaluation model of IAS 16 where
the cost model was previously in use. Also example of changes
in accounting estimates include, change in the useful lives of
Property, Plant and Equipment (PPE).
b) i. In accordance with IAS 8, Accounting policies, changes in accounting
estimate and correction of prior period errors, a change in accounting
estimate involves a change in the periodic consumption of assets which is similar to
the case of Oluwaseun Plc. In line with IAS 8, a change in accounting estimate
should be applied prospectively i.e from the year of change and in future periods. In
addition, IAS 16 - Property, Plant and Equipment states that when an entity changes
its depreciation method and useful live such should be treated as a change in
accounting estimate and the current and future years depreciation should be based on
the carrying amount of the asset at the date of the change.
Drawing from the above, the assistant accountants' suggestion of crediting the
income statement with an amount debited from accumulated depreciation is not
acceptable and not in line with IAS 8 and 16.
ii. By applying IASs 8 and 16, the carrying amount in the date of the change i.e 1 st
October 2014 is derived as follows:
N'million
Annual depreciation before change (N80m 16
5years)
Total depreciation to date (N16m x 2years) 32
Therefore carrying amount at 1/10/2014 = N80m - N32m = N48m
So, for the year ended 30 September 2015, the depreciation charge will be N48m
divided by 6 (i.e the revised useful life of 8 less the already used up life of 2 years).
This depreciation charge should be a charge against profit or loss for the current
period and in future periods until a more reliable useful life is determined.
In conclusion, the effect of this on the financial statements is to reduce the annual
depreciation charge from N16million to N8million. At the end of the year the
carrying amount of the asset would be (N48m - N8m) N40million. All other things
being equal a higher profit would be reported in the current
year but would be much lower than it would have been if the assistant accountant's
suggestion was followed.

MARKING GUIDE
(a) SELECTION BASIS OF ACCOUNTING POLICIES & DIFFERENCES Marks
BETWEEN ACCOUNTING POLICIES AND CHANGES IN ACCOUNTING
ESTIMATES
- Definition and Explanation of Accounting Policies
- Explanation of basis of selection of Accounting Policies 1
by an entity 2
- Explanation of differences between Accounting 1
Policies and changes in accounting estimates 1
- Giving one (1) examples of changes in Accounting 16
Policies
- Giving one (1) example of changes in Accounting
estimates
COMMENTS ON ACCEPTABILITY OF THE
ASSISTANT
ACCOUNTANT'S SUGGESTION
(bi) Comment that the Assistant Accountants suggestion is
not acceptable
- Giving at least three (3) correct reasons why the
Assistant Accountants suggestion is not
acceptable 1
- Correct application of IAS 8 and IAS 16 to 3
determine the carrying amount as at 1 Oct. 2014 3
- Correct illustration of how the Assistant 2
Accountants suggestion will affect the Financial _
9
statement
1
5
1
5
TOTAL MARKS FOR THE QUESTION

Examiners' Report
The question tests candidates' knowledge of IAS 8 - Accounting Policies, Changes in Accounting
Estimates and Errors. The part (a) of the question required candidates to state the basis of
selecting accounting policies by an entity and to distinguish with example, changes in accounting
policies from changes in accounting estimates. The part (b) required candidates to apply the
provisions of IAS 8 on a given scenario.
34
Few candidates attempted the question and the performance was poor as only 20% of the
candidates who attempted the question obtained 50% of the mark allocated. The commonest
pitfall is the inability of the candidates to distinguish between changes in accounting policies and
change in accounting estimates.
Candidates are advised to pay special attention to all relevant accounting standards listed in the
syllabus for this paper while preparing for the examination.

SOLUTION 6
a) WORKING CAPITAL CYCLE

Year 2015 Year 2014


(Days) (Days)
119 106
Inventory Turnover Period (W1)
Add:
Account Receivable Collection
Period (W2) 62 70
Less:
Account Payable Payment Period
(W3)
174) 172)
WORKING CAPITAL CYCLE

107 104

(w1) Year 2015 Year 2014


Inventory Turnover Period:

3,250
365
—— x
= 119 days = 106 days

11,2001

(w2) Accounts Receivable Collection Period: Year 2015 Year 2014

2,515 2,675
365 365
=——
62 daysx —— days
= 70 x
— —
14,7701 13,8731
(W3) Accounts Payable Payment Period:Year 2015 Year 2014
Trade Trade
Payables x Payables x
365
2,600 365
2,215
=Purchases
74 365 1 Purchases
= 72365 1
-— x —
daysYear -— x-----
daysYear
12,800 1
2015 11,300
2014 1
bi)
Cash & Cash Cash & Cash
Cash Ratio Equivalent Equivalent
Current Current
liabilities
3,865 liabilities
3,755
565+430 375-180
= 0.26:1 = 0.05:1
Current Current
Current Ratio
Assets Assets
Current
7,4 Current
6,3
liabilities
60 liabilities
00
1.93:1
3,8 1.68:1
3,7
65
Current Assets— 55
Current Assets—
Inventory Current Inventory Current
Quick Ratio
liabilities liabilities
7,460—3,950 = 6,300—3,250
3,755
= 0.91:1 = 0.81:1

bii) Comments
- Apata Limited's Liquidity position has improved slightly over the
period as evidenced by the improvement in current and quick ratios.
- The improvement noted above can be easily attributable to reduction in trade
receivables period indicating that customers are beginning to settle their
debt in time.
- This was also reflected in the company's cash position, as the
overdraft or deficit cash position in year 2014 improved to a positive cash
flow in year 2015. In fact the other cash and cash equivalent of the company
improved substantially.
- The significant improvement in the cash ratio was as a result of the
increase in cash and cash equivalent of the company in addition to the
decrease in receivable collection period.
However, the company's working capital cycle increased due to the fact
that it takes a longer time for the company to convert its inventory to sales.

MARKING GUIDE
(a) CALCULATION OF WORKING CAPITAL CYCLE: Marks
Determination of working capital cycle for 2014 2
Determination of working capital cycle for 2015 2
Workings/Calculation of figures for:
Inventory turnover period 2
Accounts receivable collection period 2
Accounts payable payment period 1 9
(b) i. CALCULATION OF LIQUIDITY RATIOS:
Cash Ratio 2014 and 2015 1
Current Ratio 2014 and 2015 1
Quick Ratio 2014 and 2015 1 3
ii. Comments on liquidity ratio:
Any three appropriate comments on the company's
liquidity position at 1 mark each 3
TOTAL MARKS FOR THE QUESTION

Examiners' Report
The question tests candidates' Knowledge of Financial Accounting ratios as well as the
computation of working capital cycle and the applications of those ratios in interpreting financial
statements.
Majority of the candidates attempted the question and performance was above average.
However, some candidates could not calculate the working capital cycles correctly while others
were unable to use the ratios computed to interpret the liquidity position of the company.

The computation and application of financial ratios questions is a regular feature of Financial
Reporting paper at Skill level. Candidates are advised to familiarize themselves with various
financial ratios while also making reference to Annual Reports of Companies with the aim of
carrying out analysis and interpretation of the financial
statements in some of the reports. This would assist candidates to perform better in future
examination of the Institute.

SOLUTION 7
a) FAIR VALUE
- Is the price that would be received to sell an asset or paid to transfer a
liability in an orderly transaction between market participants at the
measurement date.
- Fair value measurement look at financial assets from the point of view of a
market participant. The fair value must take into account all factors that in
market participant view would be considered relevant to value.
- A quoted price in an active market provides a reliable evidence of a fair
value.
EXAMPLES OF FINANCIAL ASSETS THAT CAN BE MEASURED USING FAIR
VALUE
i) Available for Sale Financial Assets.
ii) Financial Assets Fair Valued through profit or loss.
AMORTISED COSTS
Amortised costs is calculated as follows:
- Amount initially recognised (Initial cost of Inv.)
Plus:
- Interest Income recognised (using effective rate)
Less:
- Interest actually received (cash received)
EXAMPLES OF FINANCIAL ASSETS THAT CAN BE MEASURED AT AMORTISED
COST
i) Held - to - maturity Investment
ii) Loans and Receivables
i) AMORTISATION SCHEDULE
Years 1 Jan Effective
Coupon 31 Dec.
int. @
Rate
10%
4.72%
N'm
N'm N'm N'm
10.0
100.0 (5.9) 104.1
2013 10.4
104.1 (5.9) 108.6
2014 10.9
108.6 (5.9) 113.6
2015

ii) FAIR VALUE AS AT 31 DECEMBER 2015


125,000,000X
105 100
= N131,250,000

MARKING GUIDE
(a) EXPLANATION OF HOW FAIR VALUE
ANDAMORTISED COST ARE
DETERMINED Marks
- Explanation of how fair values are determined 2
- Explanation of how Amortised Costs are determined 2
- One example of financial asset that can be measured at fair 1
value
- One example of financial asset that can be measured at
amortised cost 6

(b)i. AMORTISATION SCHEDULE OF FINANCIAL


LIABILITIES 2
- Determination of opening balance of the financial liability 2
- Calculation of effective interest rate for each of the relevant years 2
- Calculation of Fixed interest rate for each of the relevant years
- Determination of the closing balance of the financial
liability
(ii) Calculation of the fair value of the financial liability
TOTAL MARKS FOR THE QUESTION
2
1 _
9
1
5
Examiners' Report
The question tests the knowledge of measurement of Financial Instruments at fair value and at
amortized cost as well as computation of amortized cost of financial liabilities. Less than 25% of
the candidates attempted the question and performance was poor. The commonest pitfall of the
few candidates that attempted was their inability to explain fair value and amortized cost. Also
majority of them could not prepare the amortization schedule of the Financial Instrument.
Candidates are advised to pay more attention to all aspect of the syllabus especially the relevant
International Financial Reporting Standards (IFRS) stated in the syllabus.
THE INSTITUTE OF CHARTERED ACCOUNTANTS OF NIGERIA
SKILLS LEVEL EXAMINATION - NOVEMBER 2016
F1NAN1CAL REPORTING
Time Allowed: 3 hours
INSTRUCTION: YOU ARE REQUIRED TO ANSWER FIVE OUT OF SEVEN
QUESTIONS
IN THIS PAPER
SECTION A: COMPULSORYQUESTION (30 MARKS)
QUESTION 1
a. IFRS 3 on Business Combination permits a non-controlling interest at the date of
acquisition to be valued by one of two methods.
i. At its proportionate share of the subsidiary's identifiable Net Assets or
ii. At its Fair Value (usually determined by the directors of the parent
Company).
Required:
Explain the difference that the accounting treatment of these alternative methods could
have on the Consolidated Financial Statements, including where Consolidated Goodwill
may be impaired. (5 Marks)
b. Harmony Limited acquired 70% interest in the equity shares of Foremost Limited for
N3,000,000 on January 1, 2015. The abridged Statement of Financial Position of both
companies at the date of acquisition were as follows:

HARMONY FOREMOST
LIMITED LIMITED
N'000 N'000
Identifiable Assets 32,800 8,000
Investment in Foremost Limited 3,000 —
35.800 8,000
N'000 N'000
Equity 24,000 4,800
Identifiable Liabilities 11,800 3,200
35.800 8.000
The fair value of the identifiable assets of Foremost Limited amounts to
N11,200,000 and the fair value of its liabilities is N3,200,000. The Non-
Controlling Interest will be measured as a percentage of the Net Asset of the
acquiree.
Required:
Calculate the Gain on Bargain Purchase arising from the acquisition.
(3 Marks)
c. On January 1, 2016 Kehinde Plc acquired 45million of the Equity shares of
Taiwo Plc in a share exchange in which Kehinde Plc issued two (2) new shares
for every three (3) shares it acquired in Taiwo Plc. This gave Kehinde Plc a
holding of 90% additionally on 31 December, 2016, Kehinde Plc will pay
shareholders of Taiwo Plc N1.76 per share acquired. Kehinde Plc cost of capital
is 10% per annum.
At the date of acquisition, the shares in Kehinde Plc and Taiwo Plc had a market
price of N6.50 and N2.50 respectively.
STATEMENT OF PROFIT OR LOSS AND OTHER COMPREHENSIVE INCOME
FOR THE YEAR ENDED SEPTEMBER 30. 2016
KEHINDE TAIWO PLC
PLC
N'000 N'000
323,0 190,000
(256,000) (130.000)
Revenue Cost of Sales 67.0 60.000
Gross Profit Distribution (8.000) (9,000)
Cost Administrative (19.0) (12,000)
Expenses Investment 2,500 -
Income Finance Cost (2.100) -
Profit before Tax 40.400 39,000
Income Tax Expenses (14.0) (8,000)
Profit for the year 26.400 31.000

Equity as at October 1. 2015


Share Capital(N1 per share) 300,000 75,000
Retained Earnings 270.000 175.000
The following additional Information Is also relevant:
(i) At the date of acquisition the Fair Value of Taiwo Plc's assets and liabilities
were equal to their carrying amount with the exception of two items:
> An item of plant had a fair value of N9million above the carrying amount. The
remaining life of the plant at the date of acquisition was three (3) years.
Depreciation is charged to cost of sales.
> Taiwo Plc had a contingent liability which Kehinde Plc estimated to have a fair
value of N2.25million. This has not changed as at September 30, 2016.
> Taiwo Plc has not incorporated this fair value changes into its financial
statements.
(ii) It is Kehinde Plc's policy to value non-controlling interest at fair value at the date of
acquisition. For this purpose, Taiwo Plc share price at the date can be deemed to be
representative of the fair value of the shares held by the noncontrolling interest.
(iii) Sales from Kehinde Plc to Taiwo Plc throughout the year ended September 30, 2016
had consistently been N4million per month. Kehinde Plc made a mark-up of 25% on
these sales. Taiwo Plc had N7.5million of these goods in inventory as at September
30, 2016.
(iv) Kehinde Plc's investment income is a dividend received from its investment in a 40%
owned associates which it has held for several years. The underlying earnings of the
associate for the year ended September 30, 2016 were N10million.
(v) Although Taiwo Plc has been profitable since its acquisition by Kehinde Plc, the
market for Taiwo Plc's product has been badly hit in recent months and Kehinde Plc
has calculated that the goodwill has been impaired by N10million as at September 30,
2016.
Required:
(i) Calculate the goodwill on acquisition of Taiwo Plc. (7 Marks)
(ii) Prepare the Consolidated Statement of Profit or Loss and Other Comprehensive Income
for Kehinde Plc group for the year ended September 30, 2016.
(15 Marks) (Total
30 Marks)
SECTION B: YOU AREREQUIRED TO ANSWER ANY TWO OUT OF THREE
QUESTION IN THIS SECTION (40MARKS)

QUESTION 2
a. Conceptual framework for reporting emphasises the importance that
transactions should be recorded on the basis of their substance over their form. Explain
the importance of substance over form and why financial statements should show the
substance of the underlying transaction different from its legal form. 4 Marks)
b. Evaluate the conceptual issues involved in product development costs and the definition
of an asset that may be applied in determining whether development expenditure should
be treated as an expense or an asset.
(5 Marks)
c. i. The conceptual framework states that there are two concepts of
capital. Explain these two concepts. (4 Marks)
ii. Perfect World Limited commenced business on January 1, 2015 with a
single item of inventory which costs N120,000. During the year it sold the item
for N180,000 in cash. Also, during the year, general inflation was 10% but the
inflation specific to the item was 12%. Calculate the profit under each concept
of capital maintenance and show the effect on the Equity of the Company.
(7 Marks)
(Total 20 Marks)

QUESTION 3
Magifera Plc had been trading in merchandise for several years in Garden City. The
information below relates to extracts from the Financial Statements for the past two
(2) years.
Statement of Profit or Loss and Other Comprehensive Income for the year ended September
30:
2016 2015
N‟ Million N‟ Million
Revenue 100.000 160.000
Gross Profit 45,000 70,000
Administrative Expenses 22,500 27,500
Finance Cost:
10% Loan Note Interest 1.250 1.250
23,750 28,750
Operating Profit Before Tax 21,250 41,250
Less: Taxation Expense 8.000 16.000
Operating Profit for the year 13.250 25.250
Dividends Paid to Equity holders 6,050 8,550
Extract of Statement of Financial Position as at September 30

2016 2015
N'Million N'Million
Assets:
Non - Current Assets at Cost 50,000 70,000
Less: Accumulated Depreciation 10.000 12.500
Carrying Amount 40,000 57,500
Current Assets:
Inventory 32,500 7,500
Trade Receivables 20,000 5,000
Bank Balance 4,000 37,500
56.500 50.000
Total Assets 96.500 107.500
Equity and Liabilities:
Ordinary Share Capital @ 50k each 23,000 23,000
Retained Earnings 17,200 10,000
10% Loan notes 12,500 12,500
10% Redeemable Preference Shares — 2,000
52.700 47.500
Current Liabilities:
Trade Payables 7,500 10,750
Taxation 24,000 16,000
Bank Overdrafts 12.300 33.250
43.800 60.000
Total Equity and Liabilities 96.500 107.500

The Board of Directors were worried over the dwindling financial performance and precarious
financial position of the company. The products are ageing; the economic depression is biting
as a result of the worsening exchange rate of $1 to N400. The company imports 60% of the
goods sold in Garden City. The worsening exchange rate had affected the company's
importation, consequently the revenue of the company dropped significantly. The unsafe
financial performance has also affected the market

price of the company's share which dropped from 12kobo/share in the year ended September
30, 2015 to 8kobo/share in 2016.

You are required to:


a. Calculate the following ratios for the year ended September 30, 2015 and 2016 in
columnar form:

i. Return on Capital Employed


ii. Total Assets Turnover
iii. Quick Ratio
iv. Debt- Equity Ratio
v. Fixed Interest Cover
vi. Earnings Yield
vii. Price Earnings Ratio
viii. Dividend Yield (12 Marks)

b. Write a brief and formal technical report to the Board of Directors to assess the
performance, liquidity and stability of the Company using only:
i. Return on Capital Employed
ii. Total Assets Turnover
iii. Quick Ratio
iv. Fixed Interest Cover
v. D
ebt Equity Ratio (8
Marks)
(Total 20 Marks)
Hint: Brevity, technical and formal report presentations are essential and will be rewarded.
QUESTION 4
The summarised Financial Statements for the year ended March 31, 2016 of Perfect World Plc
are as follows:
STATEMENT OF PROFIT OR LOSS FOR THE YEAR ENDED MARCH 31. 2016

N'm
Revenue 19,350
Cost of Sales (9.000)
Gross Profit 10,350
Operating Expenses (4,500)
Finance Costs (1,125)
Profit Before Tax 4,725
Income Tax Expense (2r025)
Profit for the year 2.700

STATEMENT OF FINANCIAL POSITION AS AT MARCH 31


2016 2015
Non-Current Assets: N'm N'm
Property, Plant & Equipment Current 18,900 16,650
Assets
Inventories 6,750 7,200
Trade Receivables 9,900 8,100
16.650 15.300
Total Assets Equity 35.550 31.950

Share Capital 5,400 5,400


Retained Earnings 9.900 8.550
Non-Current Liabilities 15,300 13,950

Deferred Tax 4,815 3,825


Financial Lease Liabilities 5,850 5,400
Current Liabilities 10.665 9r225

Trade Payables 5,625 4,905


Current Tax 1,013 923
Finance Lease Obligation 2,250 2,025
Bank Overdraft 697 922
9,585 8,775
Total Equity & Liabilities 35.550 31.950
Additional Information include:
(i) Dividend paid during the year amounted to N1,350million.
(ii) Perfect World Plc finances a number (but not all) of its property plant and
equipment purchased using finance lease. During the period, property, plant and
equipment which would have cost N2,700million to purchase outright was acquired
under finance lease.
(iii) There was no accrual of interest at the beginning or at the end of the year.
(iv) Depreciation charged for the year totalled N4,365million. There were no disposals of
property, plant and equipment during the year.
Required:
a. Prepare the statement of cashflows of Perfect World Plc for the year ended
March 31, 2016 using indirect method. (14 Marks)
b. Draft a Memo to the Director of Perfect World Plc summarising themajor
benefits that users receive from a published statement of cashflows.
(6 Marks) (Total
20 Marks)

SECTION C: YOU ARE REQUIRED TO ANSWER ANY TWO OUT OF THREE


QUESTIONS IN THIS SECTION (30 MARKS)
QUESTION 5
a. In many Countries of the world, Government provides financial assistance to industry
in the form of grants. In accordance with IAS 20 - Accounting for Government Grants
and Disclosure of Government Assistance.
Explain the term:
i. Grant related to Assets (2 Marks)
ii. Grant related to Income (2 Marks)
b. Prospect Nigeria Plc obtained a grant of N100million from the Federal Government of
Nigeria (FGN) for an investment project to construct a plant costing N880million.
The principal terms of the grant are as follows:
• Grant payment will be made subject to attaining the minimum level of the plant
expenditure.
• The secondary intention of the grant is to safeguard 500 jobs.
• The grant will have to be repaid pro-rata if there is an under spending on capital.
• Twenty percent (20%) of the grant will have to be paid if the jobs are not
safeguarded until 18 months after the date of the cost of capital expenditure.
Prospect Nigeria Plc completed the construction of plant on January 1, 2013 at a total
cost of N900million. The plant has an expected useful life of 20 years and is
depreciated on a straight line basis with no residual value.
Required:
i. State the type of grant that Prospect Nigeria Plc has obtained giving
reasons for your answer. (3 Marks)
ii. Show how the Asset and the grant would be reflected in the Statement of
Financial Position and Statement of Profit or Loss for years ended December 31,
2013; 2014 and 2015 under both methods of Accounting for Grants allowed by
IAS 20. (8 Marks)
(Total 15 Marks)

QUESTION 6
a. Identify the TWO kinds of leases stipulated in IAS 17 and compare in tabular
form with at least FIVE differences. (7 Marks)
c. Ijaw Oil Plc has entered into agreement to lease a plant and equipment from
Ogoni Leasing Company Limited. The lease period of the plant and equipment is six
(6) years. The agreement provides that Ogoni Leasing Company Limited will incur
upkeep expenses. The cost of the plant and equipment is N900,000,000,000. The
economic useful life is 20 years. Ijaw Oil Plc is to pay annual lease rentals of
N150,000,000 in advance over 6 years after which the plant and equipment revert to
the lessor. The implicit interest rate is 22% per annum which is stated below:

Year 0 1 2 3 4 5 6
PV(N1) 1.0000 0.8197 0.6719 0.5507 0.4514 0.3700 0.3033
You are required to:
i. Calculate the present value of the lease rental of the equipment.
(4 Marks)
ii. Identify with justification the kind of lease involved (3 Marks)
iii. Advise on how to treat the lease rentals paid by Ijaw Oil Plc. in the
financial statements (1 Mark)
(Total 15 Marks)

QUESTION 7
Alpha Plc started a 4-year contract to build a dam. Activities commenced on February 1,
2015. The total contract price amounted to N30billion, and it was estimated that work would
be completed at a total cost of N23.75billion. In the construction agreement, the customer
agreed to accept increase in wages tariffs in addition to the contract price.
The following information relates to contract activities for the financial year ended December
31, 2015.
(1) Cost for the year:
N'million

Material 3,500
Labour 2,000
Operating Overheads 375
Subcontractors 450

(2) Current estimate of total contract costs indicate the following:


i. Material will be N450million higher than expected.
ii. Total labour cost will be N750million higher than expected. Of this
amount only N600million will be the result of increase in wage tariffs. The
remainder will be caused by inefficiencies.
iii. A savings of N75million is expected on operating overheads.
(3) During the year ended December 31 2015 the customer requested a variation to the
original contract and it was agreed that the contract price would be increased by
N2.250billion. The total estimated cost of this extra work is N1.875billion.
(4) By the end of year 2015, certificate issued by the quantity surveyors indicated a 25%
stage of completion.
Required:
a. Calculate the profit to date based on:
i. Option A - Contract cost in proportion to estimated contract costs.
(6 Marks)
ii. Option B - Percentage of work certified. (6 Marks)
b. Briefly explain TWO methods recognized by IAS 11 which can be used to
determine the stage of completion of any contract. (3 Marks)
(Total 15 Marks)
SOLUTION 1

a) DIFFERENCES BETWEEN THE ACCOUNTING TREATMENT OF THE


ALTERNATIVEMETHODS
THE FIRST METHOD:
■ IFRS 3 allows as an option a non-controlling interest to be valued at its
proportionate share of the acquired subsidiary's identifiable net assets.
■ Its effects on the statement of financial position is that the resulting carrying
value of purchased goodwill only relates to the parent's elements of such
goodwill and as a consequence the non-controlling interest does not reflect its
value of the subsidiary's goodwill.
■ Any impairment of the goodwill under this method would only be charged
against the parent interest as the non-controlling interest' share of goodwill is
not included in the consolidated financial statement.
The second method of valuing non-controlling interest at its fair value would
(normally) increase the value of the goodwill calculated on acquisition.
■ This increase reflects the non- controlling interest ownership of the subsidiary's
goodwill and has the effect of grossing up the goodwill and the non-controlling
interest in the statement of financial position, by the same amount.
■ Under this method any impairment of the subsidiary's goodwill is charged to
both the controlling (parent share) and non-controlling interest in the
proportion of their holding of share in the subsidiary.

b) GAINONBARGAINPURCHASE
N'000

Consideration 3,000
Add:
Non-Controlling Interest (8,000 x 30%) 2,400
5,400
Less:
Net Identifiable Assets of Foremost Ltd (11,200 - 3,200) 8.000
Gain on Bargain Purchase 2.600
ALTERNATIVE SOLUTION

N’000
fair value of identifiable net assets acquired by Harmony {70% x (11200-
3200) 5,600
fair value of consideration transferred 3,000
Gain on Bargain Purchase 2,600

c) (i) CALCULATION OFGOODWILL

N'000
Consideration Transferred:
45x 2
Share Capital ( -------- = 30m shares x N6.50) ,. c .. .
3 195,000
Deferred Consideration:
(45m shares x N1 76 x
' 1l) 72.000
267,000
Fair Value of NCI
(5m shares x N2.50k) 12,500
279.500
Fair Value of Assets N'000
Share Capital 75,000
Retained Earnings 175,o o o + ( 31,000*^) 182,750
Fair Value Adjustment to Plant 9,000
Contingent Liabilities (2.250)
264.500
Goodwill (279,500 - 264,500) 15.000
c) (ii) KEHINDE PLC GROUP - CONSOLIDATED
STATEMENT OF PROFIT OR LOSSFOR
THE YEAR ENDED SEPTEMBER 30. 2016
Revenue 323,000 + (190,000 - 36,000 (w2) N'000
Cost of Sales (256,000 + 130,000- 36,000 + 1,500 (w2) + 2,250 (w3) Gross 429,500
Profit (321.250)
Distribution Cost ^s,ooo + 9,000 x(^)) 108,250
Administrative Expenses ^19,000 + (12,000 +10,000(Goodlmpairment) (14,750)
Finance Cost (w4) (38.000)
Share of Profit of Associates (10,000 x 40%) (7,500)
Profit Before Tax 4,000
Less: 52.000
Income Tax Expenses^ 14,000 + (s,ooo (20.000)
Profit for the year 32.000
Profit Attributable to: 30,900
Owners of the Parent Non- 1,100
Controlling Interest (w5) 32.000

Working Notes
W1 Group Structure Total Control
Kehinde plc interest Taiwo plc 100%
interest Acquisition of Subsidiary 90%
during 10%
the year 9 months before year-end.

W2 Intergroup Trading N'000 N'000 N'000


(i) Debit Revenue (4,000 x 9 months) 36,000
Credit Cost of Sales 36,000
(ii) unrealised Profit ^7,500 x(^r))
Cost of Sales 1,500
Group Inventory (SFP) ==== 1.500
W3 Fair Value Adjustments
N'00 N'000 N'000
Plant ^9,000 -h years 0 2,250 6,750
9,000

W4 Finance Cost N'000


Kehinde per Profit or Loss 2,100
(£)) 5,400
Discount on Deferred Consideration ^72, 000x1094
7.500

W5 NCI N'000
Profit for the year (31,000 x(^)J 23,250
Depreciation on Fair Value Adjustment (2,250)
Goodwill Impairment (10,000)
11.000
1.100
NCI Share @ 10%

MARKING GUIDE
Marks Marks
a. Explanation of five (5) differences @ 1mark each 5
b. Determination of consideration 2
Determination of fair value of net assets 1 3
ci. Determination of consideration 4
Ascertainment of fair value of net assets 3 7
cii. Determination of Revenue 2
Calculation of cost of sales 372
Determination of Gross Profit ¥2
Determination of Distribution cost 1
Administrative Expenses 1
Finance Cost %
Share of Associate's profit 1
Profit before tax %
Income tax expenses 1
Profit for the year %
Profit attributable to owners of parent %
Profit attributable to NCI 1
Calculation of unrealised profit 2 15
Total Marks 30
EXAMINER'S REPORT

The question tests the following:


- Difference in the accounting treatments of non-controlling interest in accordance with
IFRS 3 on business combination.
- Calculation of Gain on bargain purchase and preparation of consolidated profit or loss
and other comprehensive income including calculation of Goodwill.
As a compulsory question all the candidates attempted the question, but performance was very
poor. The commonest pitfalls of the candidates include:
• I
nability of the candidates to differentiate between the different methods of accounting.
• I
nability of some of the candidates to correctly calculate the goodwill in part 'c' of the
question and failure to reflect necessary adjustments while preparing the Consolidated
Statement of profit or loss and other comprehensive income.
Candidates are advised to pay more attention to all sections of the syllabus, Institute study text
and pathfinder while preparing for the examinations for better performance in future.
SOLUTION 2

a. Importance of substance over form


Users desire information in general purpose financial statements for making economic
decisions. Therefore, financial statements are expected to be of value to their users and as
such they must possess certain important characteristics like reliability and faithful
representation.
For financial statements to be reliable, they must faithfully represent an entity's
underlying transactions and other events. To achieve faithful representation, transactions
must be accounted for and presented in accordance with their substance and economic
reality where this differs from their legal form.
For example, where an entity 'sold' an asset to a third party, but continued to enjoy the
future benefits embodied in that asset, then this transaction would not be represented
faithfully by recording it as a sale. In all probability this would be a financing transaction.
b. Conceptual issues in project development cost
An asset is defined by the Conceptual Framework as a resource controlled by an entity as
a result of past transactions or events from which future economic benefits (normally net
cash inflows) are expected to flow to the entity. However assets can only be recognised
(on the statement of financial position) when these expected benefits are probable and
can be measured reliably.
The Conceptual Framework recognises that there is a close relationship between
incurring expenditure and creating an asset, though they do not necessarily happen at the
same time. Development expenditure is an example of the relationship between
expenditure and creating an asset.
Research and development expenditure is incurred with the hope that it will lead to a
profitable product, process or service, but at the time that the expenditure is being
incurred, entities cannot be certain or it may not even be probable that the project will be
successful.
In line with accounting concepts, if there is doubt that a project will be successful the
application of prudence would imply that the expenditure is expensed in the statement of
profit or loss.
When management becomes confident that the project will be successful, it then meets
the definition of an asset and the accruals/matching concept would mean that it should be
capitalised (treated as an asset) and amortised over the period of the expected benefits.
IAS 38 Intangible Assets interprets this as write off of all research expenditure and
only capitalising development costs from the point in time where they meet definition
of an asset.

c. (i) Capital Concepts


A financial concept of capital is that the capital of an entity is measured as its net assets,
which is also its equity. Where a financial concept of capital is used, the main concern of
users of the financial statements is with the maintenance of the nominal financial capital
of the entity.
With financial concept of capital maintenance, a profit is not earned during a period
unless (excluding new equity capital raised during the period and adding back any
distribution of dividends to shareholders) the financial value of equity (net assets) at the
end of the period exceeds the financial value of equity at the beginning of the period.
A physical concept of capital is that the capital of an entity is represented by its
productive capacity or operating capability. Where a physical concept of capital is used,
the main concern of users of the financial statements is with the maintenance of the
operating capability of the entity.
With a physical concept of capital maintenance, a profit is not earned during a period
unless (excluding new equity capital raised during the period and adding back any
distribution of dividends to shareholders) the operating capability of the business is
greater at the end of the period than at the beginning of the period. Current value
accounting reflects the concept of physical capital maintenance.
In a period of inflation, profits reported using a financial concept of capital will normally
be higher than profits reported using a physical concept of capital, such as current value
accounting.
Financial capital maintenance is likely to be the most relevant to investors as they are
interested in maximising the return on their investment and therefore its purchasing
power while Physical capital maintenance is likely to be most relevant to management
and employees as they are interested in assessing an entity's ability to maintain its
operating capacity.
Profit is calculated under each concept in the following ways:-
Financial Financial (real Physical
(Money terms)
terms)
Statement of profit or loss:- N N N

Revenue 180,000 180,000 180,000


Cost of sales (120,000) (120,000) (120,000)
Inflation adjustment (inflation rate
applied to opening equity:-
10% X 120,000 (120.000) (14.400)
12 % X 120,000 60.000 48.000 45.600

Statement of financial position:- N N N

Net Asset Equity:- 180.000 180.000 180.000


Opening equity:

Before adjustment 120,000 120,000 120,000


Inflation reserve - 12,000 14,400
After Adjustment 120,000 132,000 134,400
Retained profit for the year 60.000 48.000 45.600
180.000 180.000 180.000
MARKING GUIDE
Marks Marks
(a) - Explanation - importance of financial statement 1
- Differentiating between substance over form and legal form of
transactions 2
- Giving relevant examples 1 4
(b) - Giving definition of an Asset in accordance with conceptual
frame work 1
- Explaining relationships between incurring expenditure and
creating an Asset 1
- Giving relevant examples to justify explanation of development
expenditure treatment 1
- Making reference to IAS 38 on the treatment of Development
Expenditure 1
- Appropriate calculation 1 5
(ci) - Definition/explanation of financial concept of capital
maintenance 2
- Definition/explanation of physical concept of capital
maintenance 2 4
(cii) - Determination of Net Asset
Using financial Concept(Money terms) 2
- Determination of Net Assets
- Using financial concept(Real terms) 21/2
Determination of Net Assets using physical concept
2/2 7
Total marks 20

EXAMINER'S REPORT
The question tests candidates knowledge on conceptual framework of 1ASB concerning the
issue of “substance" over "form" of financial transactions and the concept of capital
maintenance.
About 30% of the candidates attempted the question and their performance was very poor.
Candidates' displayed lack of knowledge of the content of the conceptual framework of 1ASB
and they could not distinguish between the financial concept and physical concept of capital.
Candidates are advised to study all sections of the syllabus while preparing for future
examinations.
SOLUTION 3
RATIO FORMULAE 2016 2015

(a) Calculation of Ratios


(i) Return on PBIT Capital 21250 + 1250 52,700 41250 + 1250
capital employed = employed 42.69% 47,500 89.47%%

Or PAT 13250 40200 = 25250


Equity 32.96%% 33000
= 76.52%%

(ii) Total asset turnover Revenue 100,000 96,500 = 100,000 107,500 =


TA 1.04 times 1.49 times

(iii) Quick Ratio = CA - Inventory CL 56.500 -32.500 50.000 - 7.500


43,800 = 0.55:1 60,000 = 0.71:1

(iv) Debt equity Ratio = Debt 12.500 23000 12500 + 2000


Equity +17,200 = 31.09%% 23000+10,000 =
43.94%%

(v) Fixed Interest Cover = PBIT 21250 + 1250 1250 = 41250 + 1250 1250
Fixed Interest 18 times = 34 times

(vi) Earnings yield = EPS 13250 x 50k 23000 25250 x 50k 23000
Market Price per share 8k = 28.8k 8k = 12k = 54.89k 12k
360%% 457%%
8k 12k

(vii) Price earning ratio = MPS 13250 X 50K 23000 25250 X50 23000
EPS 8K = 28.8K 12K =
54.89K
RATIO FORMULAE 2016 2015

OR Vey 0.28 0.22


1 1
% %
360 457
0.28 0.22
(Viii) Dividend yield = DPS 6050 x 50k 8550 x 50k
MPS 23000 23000
8k 12k
= 13.15k 18.59k
8k 12k
164 % 155%

b.
REPORT
To: The Board of Directors
Magifera Plc.,
Garden City
From: Financial Accountant
REPORT ON PERFORMANCE. LIQUIDITY AND STABILITY OF MAGIFERA PLC.
The last two years have been challenging for the company. A review of the
financial ratios of the company reveals the following:
Performance
The company's performance is on the decline as evidenced by the two ratios
below.
(i) Return on Capital Employed
This is the Primary ratio indicating the efficiency or otherwise of management
in employing or utilising the resources available. The more the efficiency of
utilisation, the higher the ROCE and vice versa. The company recorded a
decrease in return on capital employed from 89.47% in 2015 to 42.69% in
2016. The decline in the return on capital employed can be attributed to the
declining revenue which may not be specific to the company but to all
companies in Garden City as the city is currently experiencing economic
depression and worsening exchange rate coupled with its ageing products of
the company.
(ii) Total Assets Turnover
The ratio indicates the efficiency in utilisation of assets to generate revenue.
The company recorded a total assets turnover of 1.49 times in
2015 but decreased to 1.04 times in 2016. This is an indication that in
2016 the company did not utilize its assets as efficiently as it did in
2015. This decrease in the efficiency of the utilization of assets is also due to
the challenges stated in return on capital employed above.
Liquidity
The company's liquidity worsened between 2015 and 2016.
(iii) Quick Ratio
The ratio indicates the liquidity of an entity by measuring the relative amount
of cash and other near liquid assets available to meet current liabilities. Thus, it
is a relatively strong indicator of ILiquidity. The quick ratio of the company
which is below the universal norm of 1 : 1 in 2015 and 2016 is deteriorating. It
decreased from 0.71: 1 in 2015 to 0.55: 1 in
2016.
(iv) Fixed Interest Cover
This ratio is an indicator of the long term health of an entity in that it measures
the number of times fixed interest is covered by the profit before deducting the
fixed interest
The higher the fixed interest cover, the higher the long term solvency of an
entity.
The decline by half of this ratio from 34 times in the preceding year to 18 times
in the current year (2016) is an indication of the precarious nature of Magifera
long term solvency.
Stability
(v) Debt Equity Ratio
This ratio also indicates an entity's long term solvency by measuring the extent
of cover for external liabilities. The company‟s ratio of debt to equity
decreased from 43.94% in 2015 to 31.09% in 2016. The ratio for both years
indicates that the company has a reasonable gearing.
In conclusion the performance, liquidity and stability of the company deteriorated from 2015
to 2016.
Thank you Yours
faithfully Financial
Accountant
MARKING GUIDE
Marks Marks
a) Calculation of the following ratios:
- Return on Capital Employed 1V2
- Total Asset Turnover 1V2
- Quick Ratio 1V2
- Debt/Equity Ratio 1V2
- Fixed Interest Cover 1V2
- Earnings Yield 1V2
- Price Earnings Ratio 1V2
- Dividend Yield 1^
b) Report12to the Board of Directors
- Presentation in report format and opening comment
- Stating the correct meaning and comment on the trend of
the following ratios in the report on the performance,
liquidity and stability of the company:
- Return on capital employed
- Total Asset Turnover
- Quick Ratio
- Fixed interest cover
- Debt Equity Ratio
- Conclusion of the report 8
- Closing/signature 20
- Total marks
EXAMINER'S REPORT
The question tests candidate's knowledge of ratio analysis and interpretation of financial
statements. Candidates are required to calculate eight (8) different ratios and to write a the
formal technical report to the Board of Directors concerning performance, liquidity and
stability of the company based on some computed ratios.
Over 90% of the candidates attempted the question and their performance was very good.
Candidates are advised to keep up the performance.
SOLUTION 4
a. PERFECTWORLDPLC
STATEMENT OF CASHFLOWS FOR THE YEAR ENDED 31 MARCH.
2016
CASHFLOWS FROM OPERATING ACTIVITIES N'm Nm
Net Profit before Tax 4,725
Adjustment for:
Depreciation 4,365
Interest Expenses 1,125
10,215
Changes in Working Capital:
Decrease in Inventories (6,750 - 7,200) 450
Increase in Trade Receivables (9,900-8,100) (1,800)
Increase in Trade Payables (5,625 - 4,905) 720
9,585
Interest Paid (1,125)
Income Tax Paid (w2) (945)
Net Cashflow from Operating Activities 7,515
CASHFLOWS FROM INVESTING ACTIVITIES
Purchase of Property, Plant & Equipment (w1) (3,915)
CASHFLOWS FROM FINANCING ACTIVITIES
Payment of Finance Lease Liabilities (w3) (2,025)
Dividend Paid (1.350)(3.375)
Net Increase in Cash and Cash Equivalents 225
Cash and Cash Equivalents at beginning of the year (922)
Cash and Cash Equivalents at the end of the year (697)

WORKINGS
W1 PROPERTY,PLANTANDEQUIPMENT(PPE)

Nm Nm
B/d - Balance 16,650 Depreciation 4,365

Additions 2,700
Cash/Bank * 3,915 23.265 c/d Balance 18.900
23.265


W2 INCOME TAX PAYABLE

N'm N'm
B/d - DT 3,825
Cash/Bank (Paid) * 945 - CT 923
P&L 2,025
C/d - DT 4,815
- CT 1,013
6.773 6.773

W3 FINANCELEASELIABILITIES
N'm N'm
Cash/Bank (Paid) * 2,025 B/d > 1 year 5,400
< 1 year 2,025
< 1 year 2,250 PPE 2,700
C/d > 1 year 5,850
10.125 10.125

b.
MEMO
To: Director of Perfect World Pic
From: Accountant
Date: November, 2016
Subject: Major Benefits to the Users of Financial Statement from the Publication
of Statement of Cashflows
The Users of financial statements can be basically divided into the following groups:
(i) Shareholders
(ii) Management
(iii) Creditors and Lenders
(iv) Employer
The need of the groups are not identical and hence not all benefits listed below will
be applicable to all users.
(i) Statement of Cashflows direct attention to the survival of the entity which depends on
its ability to generate cash.
(ii) Statement of Cashflows indicate the ability of an entity to repay its debts when due.
(iii) Statement of Cashflow give information which can be used in decision making and
stewardship process
(iv) They are more easily understood than statement of profit or loss that depends on
accounting conventions and concepts.
(v) 1t is useful in assessing the ability of the entity to generate cash and cash equivalents.
(vi) A statement of Cashflows when used in conjunction with the rest of the financial
statements provides information about liquidity and insolvency of an entity.
(vii) A statement of cashflows provides information that enables users to evaluate the
changes in net assets of an entity, its financial structure and its ability to effect the
amount, timing and certainty of cashflow.
(viii) 1t enhances the comparability of the reporting of operating performance by different
entities because it eliminates the effect of using different accounting treatment for the
same transaction and event.
(ix) 1t is also useful in checking the accuracy of past assessment of future cashflows and in
examining the relationship between profitability and net cashflows
(x) A statement of cashflows provides information that helps users to evaluate changes in
the net assets of an entity and its financial structure (including its liquidity and
solvency)
Marking Guide
A Preparation of statement of cashflows: Marks Marks
- Determination of cashflows from operating activities 6
- Determination of cashflows from investing activities 3
- Determination of cashflows from financing activities 4
- Stating the cash and cash equivalents 1 14
b. Memo to the Director of Perfect World Plc:
- Presentation in memo format 1
- Correctly stating any five benefits to users of financial
statement associated with the publication of statement of
cashflows at 1 mark each 5 6
Total Marks 20

EXAMINER'S REPORT
The question tests candidates' ability to prepare statement of cashflows using indirect method
and the benefits that users of published financial statements would derive from the inclusion
of statement of cashflows in the published financial statements.
About 85% of the candidates attempted the question and performance was above average.
The commonest pitfall was the inability of the candidates to clearly state the benefits to be
derived from the inclusion of the statement of cashflows into published financial statements.
Also some candidates could not present the cashflow strictly in accordance with format
stipulated by IAS 7 on statement of cashflow
Candidates are advised to familiarise themselves with the presentation of statement of
cashflows in Annual Reports of Companies while also ensuring that all relevant accounting
standards are applied in the presentation of their solution to the Institute's examination
questions.
SOLUTION 5

(a) GRANT RELATED TO ASSETS


Grant related to assets is for purchase or construction of long-term assets. Conditions
may be attached to the grant, specifying the type of asset that should be purchased
with the grant or the location of the assets or the period in which they are to be
acquired or held. For grant related to assets, IAS 20 allow two methods for accounting
for it as follows:
Method 1 -Deduct grant from the cost of the related assets. The asset is included in the
statement of financial position as cost minus the grant while the Net amount is
depreciated over the useful life of the assets.
Method 2 - Treat the grant as deferred income and recognize it as income on a
systematic basis over the useful life of the assets.
GRANT RELATED TO INCOME
This is any other government grants, other than grant related to assets. For grant
related to income. IAS 20 states that the 'INCOME APPROACH' should be used and
the grant should be taken to income over the periods necessary to match the grant with
the cost that the grant is intended to compensate. IAS 20 allows two (2) methods as
follows:
Method 1 -Include the grant for the period as other income for inclusion in profit or
loss for the period.
Method 2 - Deduct the grant for the period from the related expenses.
(b) TYPE OF GRANT
The grant that Prospect Nig. Plc obtained is 'AN ASSET RELATED GRANT'.
• This is because the grant obtained is used for Investment project to construct a
plant.
• The employment condition is just an additional condition to prevent
replacement of labour by capital.
Method 1 STATEMENT OF FINANCIAL POSITION
2015 2014 2013
N^000 N'000 N'000
Plant (900,000 - 100,000) 800,000 800,000 800,000
Accumulated Depreciation (800,000 + 20) (120.000) (80.000) (40.000)
680.0 720.000 760.000
STATEMENT OF PROFIT OR LOSS
Depreciation charged (800,000 + 20) 40.000 40.000
40.000

(ii) USING METHOD 2 - STATEMENT OF FINANCIAL POSITION

2015 2014 2013


Property. Plant and Equipment N'000 N'000 N'000
Plant (Cost) 900,000 900,000 900.0 (
Depreciation (Accum.) (135.000) (90.000) 45.000
765.0 810.000 )
855.000
Current Liabilities
Deferred Income 5,000 5,000 5,000
Non-Current Liabilities 90.000
Deferred Income 80.000 85.000

Statement of Profit or
LossExpenses
Depreciation (900,000 + 20) 45.000 45.000 45.000
Income (5.000)
Government Grant Income (100,000 + 20) (5.000) (5.000)
Marking Guide Marks Marks
ai) Explanation of grant related to Asset 1
Treatment of Grant related to Asset 1
ii. Explanation of Grant related to Income 1
Treatment of Grant related to Income 1 4
bi. Stating the correct type of Grant 1
Giving two reasons for the type of Grant 2
3
ii. Disclosure of Asset and grant in the financial
statements Method - two
Statement of financial position 2V 2
Statement of profit or loss 2
Method - one
Statement of financial position 2V 2
Statement of profit or loss 1 8
Total marks 15

EXAMINER'S REPORT
The question tests candidates' knowledge of the provisions of IAS 20 on Accounting for
Government Grants and Disclosure of Government Assistance.
About 30% of the candidates attempted the question and performance was poor.
Most of the candidates that attempted the question displayed lack of understanding of the
provision of IAS 20, as they could not explain the term grant related to Assets and Grant
related to Income.
Similarly, only about 10% of the candidates that attempted the question could reflect the
effect of the grants in the Statement of Financial Position and Profit or loss for all the relevant
years.
Candidates are therefore advised to ensure that they pay adequate attention to all accounting
standards relevant at this level of the Institute's examination for better performance in future.
SOLUTION 6
(ai) The two types of Leases under 1AS 17 are:
- Operating lease
- Finance or capital lease
( aii) DIFFERENCE BETWEEN OPERATING LEASE AND FINANCE LEASE

OPERATING LEASE FINANCE LEASE


(1) Ownership is not transferred at the end of the Ownership is transferred at the end of the
lease period. lease period

(2) No bargain purchase option in the contract Bargain purchase option exist in the contract

(3) The present value of the minimum lease The present value of the minimum lease
payment does not equal the asset fair value payment substantially equal to the asset fair
value
(4) The asset can be leased out again after the The assets cannot be leased out again after
expiration of the initial lease period. the expiration of the initial lease period
(The assets would have been used for
substantial part of the useful life)

(5) Lessor maintains and is responsible for the The Lessee maintains and upkeeps the assets
upkeep of the assets

(6) Lease rentals is expensed Lease rental is used to reduce Lease


obligation while some portion is expensed
as finance cost.
(7) No value of asset is recognised in the books Value of asset is recognised as an asset and
of the lease and no depreciation nor depreciation/ impairment of the asset are
impairment of the asset is charged. charged.
b.(i)

NET PRESENT VALUE


Year Cashflow Discount favour @22% Present Value
N000 N000
0 (150,000) 1 (150,000)
1 (150,000) 0.8197 (122,955)
2 (150,000) 0.6719 (100,785)
3 (150,000) 0.5507 (82,605)
4 (150,000) 0.4514 (67,710)
5 (150,000) 0.3700 (55,500)
Net Present Value (579.555)

OR
Year Cashflow Discount favour @22% Present Value
N000 N000
0-5 (150,000) 3.8637 (579.555)

(ii) This is an operating lease because of the following reasons:


- Under the lease arrangement, Ogoni leasing Company Ltd, the lessor, will bear the
upkeep (maintenance) expenses on the plant and equipment Ijaw oil Plc will lease the
plant and equipment for six years which is not a substantial part of the asset life of 20
years.
- Under the lease arrangement, the present value of the lease rental of N579,555,000 is
less than and not equal to the fair value of N900,000,000,000 of the plant and
equipment.
- The asset can be leased out again after the expiration of the initial six years
lease period.

(iii) The lease rental paid by Ijaw Oil Plc shall be expensed in the income statement of Ijaw
Oil Plc as operating expenses while the outstanding lease rentals will be reported as
current liability in the statement of financial position.
MARKING GUIDE Marks Marks
a. - Identification of 2 types of lease at 1 mark each 2
- Stating five differences between operating and
finance lease at 1 mark each 5 7
bi. Calculation of present value of lease rentals:
- Cash flows 1
- Correct discount factor 1
- Correct present value 1
- Correct total lease rentals 1

Ii - Correct identification of the kind of lease 1


- Stating of two(2) correct justifications of the
lease identified at 1 mark each 2

Iii - Correct advice of the treatment of the lease rental


in profit or loss account %
- Correct treatment in statement of financial
position % 1
Total Marks 15

EXAMINER'S REPORT
The question tests candidates' knowledge and understanding of accounting for leases - IAS 17.
The candidates are required in Part (a) to identify and differentiate between the two kinds of
leases and in Part (b) calculate the present value of lease rental of equipment.
About 85% of the candidates attempted the question and their performance was good as over
70% of them obtained more than 50% of the marks allocated to the question.
Candidates are advised to study all the accounting standards listed in the syllabus for this
examination for better performance in future.
SOLUTION 7

CALCULATION OF PROFIT AS AT 31 DECEMBER,


2015 Option A Option B
N‟m N‟m
Contract Revenue (w4) 7,767 8,213
Contract Cost to Date (w1) (6r325) (6r325)
Profit recognised to date 1,442 1,888
WORKINGS
W1 Contract Cost N‟m
Material 3,500
Labour 2,000
Operating Overhead 375
Sub-Contractors 450
6,325
W2 Contract Cost (Revised estimated Total Cost) N‟m
Original Estimate 23,750
Materials 450
Labour 750
Operating Overheads (75)
Variation 1r875
26.750
W3 Contract Revenue (Revised Estimate) N‟m
Original Estimate 30,000
Labour (Wage increase added to Contract Price) 600
Variation 2r250
32.850
W4 Option A Option
N‟m N‟
Based on Contract Cost in proportion estimated
Total Cost: (6,3254- 26,750 x N32,850) 7,767

Based on Work Certified: (25% x N32,850) 8.213


b. 1AS 11: Construction Contract - recognises these methods of determining stage of
completion:
i. Contract cost to date in proportion to estimated total contract costs (The cost
basis)
This estimates the stage of completion by comparing the contract costs
incurred to date to the total contract costs expected to be incurred over the life
of the contract. This involves estimating the future costs that will be necessary
to complete the contract.
ii. Percentage of work (Sales basis)
This estimates the stage of completion by comparing the sales value of the
work performed to date to the total contract revenue. As a contract progresses,
at periodic intervals, an independent expert such as a surveyor may inspect the
work, and issue a certificate stating the amount or value of the work done so far. Th
issued by the independent expert provides a basis for judging the
proportion of the contract work that has been completed.
iii. Use the physical proportion of the contract work that has been
completed.

MARK1NG GU1DE
Marks Marks
a. Computation of Revenue under option A 1V2
- Computation of Revenue under option B 1V2
- Determination of Contract cost 3
- Determination of Revised Estimate of Contract cost 3
- Determination of Revised estimate of Contract Revenue 2
- Determination
of profit recognised to date 1
12
b. - Explanation of two (2)
methods recognised by 1AS 11
- 1dentification of the method 1
- Explanation of the method 2 3
Total marks 15
EXAMINER'S REPORT
The question tests candidates' ability to calculate profit on construction contracts using different
methods recognised by 1AS 11.
Less than 30% of the candidates attempted the question and performance was below average.
The commonest pitfall was the inability of the candidates to correctly calculate the Revenue of
the contract, hence they could not determine the profit using the two options specified by the
question.
Candidates are advised to cover all sections of the syllabus for better performance in future
examinations.
• ♦
THE INSTITUTE OF CHARTERED ACCOUNTANTS OF NIGERIA
SKILLS LEVEL EXAMINATION - MAY
2017 FINANCIAL REPORTING
Time Allowed: 3 hours
INSTRUCTION: YOU AREREQUIRED TO ANSWER FIVE OUT OFSEVEN
QUESTIONS IN THIS PAPER
SECTION A: COMPULSORYQUESTION (30 MARKS)
QUESTION 1
1. The following information relates to financial statements included in the
annual report of Bello Professional Nigeria Limited.
Summarised Statement of Profit or Loss for the year ended March 31
2015 2014
N'000 N'000
Revenue 412,500 300,000
Cost of sales (328.500) (187.500)
Gross profit 84,000 112,500
Operating expenses (90,000) (45,000)
Finance expenses (7,500) (4,500)
Profit/(loss)before tax (13,500) 63,000
Income tax credit/(expense) 5.250 (21.000)
Profit/(loss) for the year (8.250) 42.000

Summarised Statement of Financial Position as at March 31


2015 2014
N'000 N'000
ASSETS:
Non-current assets
Property, plant & equipment 142,500 191,250
Current assets:
Inventories 93,750 34,500
Trade receivables 33,750 15,000
Sundry receivables 3,750 -
Bank balance - 11.250
Total assets 273.750 252.000
2015 2014
N'000 N'000
EQUITY & LIABILITIES
Equity
Ordinary shares of N1 each 75,000 60,000
Share premium 24,000 30,000
Retained earnings 33.750 47.250
132,750 .137,250
Non-current liabilities:
10% loan notes - 37,500
Finance lease obligations 36,000 15,000
Deferred tax liabilities 9r000 6r000
45.000 58.500
Current liabilities:
10% loan notes 37,500 -
Current income tax - 18,750
Bank overdraft 10,500 -
Finance lease obligation 12,750 6,000
Trade payables 35.250 31.500
96,000 56,250
Total equity and liabilities 273.750 252.000

The following additional information is also available:


(i) The finance expenses are made up of the following items:
2015 2014
N'000 N'000
Loan notes interest 3,750 3,750
Finance lease charges 2,250 750
Interest on overdraft 1r500 -
7,500 4,500

(ii) The property, plant and equipment schedule included in the notes to the financial
statements contained in the report are as follows:

2015 2014
N'000 N'000
Leased plant 48,750 18,750
Leasehold plant - 66,000
Owned plant 93.750 106.500
142.500 191.250
During the year Bello Professional Nigeria Limited sold its leasehold plant
for N63.75million and entered into an agreement to rent it back from the
purchaser. There were no additions to or disposals of owned Plant during
the year. The depreciation charges which are included in the cost of
sales for the year ended 31 March 2015 were as follows:
N'000
Leased plant 13,500
Leasehold 1,500
plant Owned 12.750
plant 27.750
(iii) On August 1 2014 there was a bonus issue of shares from share premium of
one new share for every 10 held on May 1 2014. There was a fully
subscribed cash issue of shares at par as at March 31. 2015
(iv) The 10% loan notes is due for repayment on June 30, 2016. Bello
Professional Nigeria Limited is in negotiation with the loan providers,
Accrual Bank Plc.
Required:
a. Prepare a statement of cashflow for Bello Professional Nigeria Limited
for the year ended March 31, 2015 in accordance with IAS 7 using indirect
method. (18 Marks)
b. One of the directors present at the annual general meeting of Bello
Professional Nigeria limited where the financial statements were laid before
members was of the view that direct method of preparing cashflow is more
useful and provides better information to users than the indirect method.
Comment on the director's view stating whether you agree or not, giving
reasons for your opinion. (7 Marks)
c. IAS 7 - Statement of cashflow allows some variation in the ways that the
cashflow for interests and dividends are presented in the statement.
Explain the various ways of classifying the following items in a Statement of
cashflow as permitted by IAS 7.
i. Interests paid
ii. Dividends received (5 Marks)
(Total 30 Marks)
SECTION B: YOU ARE REQUIRED TO ANSWER ANY TWO OUT OF THREE
QUESTIONS IN THIS SECTION (40 MARKS)

QUESTION 2
Abuja Limited acquired 80% of Abaji Limited's ordinary shares on January 1 2015. The
company paid an immediate N5.00 per share and a further payment of N19,440,000 in
cash. The company only recorded the cash consideration of N5 per share. The two
statements of financial position as at December 31 2015 are stated below:
Abuja Abaji
Limited Limited N'000N'000
Non- current assets:
Property, plant and equipment 75,600 57,600
Development costs - 7,200
Investment 68,400 3,600
Current assets 23.940 16.380
Total assets 167.940 84.780

Equity and liabilities:


Ordinary shares of N1 each 48,600 14,400
Share premium 14,400 7,200
Revaluation reserve 8,100 -
Retained earnings:
January 1 2015 28,800 24,120
Year to December 31 2015 34,200 13,680

Non-current liability
8% intercompany loan - 10,800
Current liability Total 33,840 14,580
equity & liabilities 167.940 84.780

Additional information provided are:


(i) The parent company, Abuja Limited, value non-controlling interests (NCI) using
the fair value at the acquisition date. The fair value of NCI at the acquisition date
was N14,940,000. There is an impairment as at December 31 2015 resulting in the
reduction of NCI to N14,220,000.
(ii) Abaji Limited revalued land and building using fair value which resulted in an
increase of N3,600,000 at the acquisition date and a further N720,000 at December
31, 2015.
(iii) Abaji Limited have line of products with a brand name valued at N7,200,000 with
an estimated life of 10 years as at 1 January 2015. The brand is not included in
Abaji statement of financial position on this date.
(iv) A loan of N10,800,000 from Abuja Limited granted to Abaji Limited at acquisition
date was included in Abuja Limited investment. Loan interest is payable annually in
arrears. Abuja Limited did not receive the interest due and paid by Abaji for the
year ended December 31, 2015 until after the year end. Therefore Abuja Limited
has not accounted for the Abaji accrued interest.
(v) The development project of Abaji Limited was completed on June 30, 2015 at a cost
of N9,000,000. As at December 31, 2015, N1,800,000 had been amortised. Abaji
Limited had capitalised N3,240,000 at the acquisition date. However, the directors
of Abuja Limited are of the opinion that Abaji Limited development costs cannot be
recognised as an asset because it does not meet the requirement in IAS 38.
(vi) Abuja Limited bought goods from Abaji Limited. One third of the goods were still
in the inventory of Abuja Limited at December 31, 2015. The goods was sold to
Abuja Limited at a profit of N1,080,000.
Required:
Provide figures to be included in the consolidated statement of financial position as at
December 31 2015 in respect of the following:
a. Non- controlling interest (7 Marks)
b. Goodwill (7 Marks)
(show your calculations of net assets as at date of acquisition and date
of consolidation)
c. Consolidated reserves:
i. Share premium
ii. Retained earnings
iii. Revaluation reserve (6 Marks)
(Show your workings) (Total 20 Marks)
NB. You are not required to prepare a consolidated statement of financial
position as at December 31 2015
QUESTION 3
You are a financial reporting consultant. The management of Bode Limited a well-
diversified company with branches in all states of the federation has some
transactions for which it requires advice from you. Bode Limited has a financial
accountant who is not yet a qualified accountant.
These transactions are as listed below:
(a) The company recognised a cash generating unit during the year ended
December 31, 2015 which is made up of the following assets.
C
osts
N'm
4,050
Property, plant and equipment
450
Goodwill
2.700
Other assets
7.200
The management of Bode Limited estimated that the recoverable amount of the cash
generating unit as at the end of the year will be N6.30billion.
The financial accountant of Bode Limited is aware of some of the provisions of IAS
36 on impairment of assets but he is confused as to how impairment (if any) on these
assets should be allocated among the assets that make up the cash generating unit of
the company.
(b) Also, on January 1, 2015 Bode Limited borrowed N300million to finance the
production of two assets both of which were expected to take one year to build.
The work started on January 1, 2015. The loan facility was drawn down on the same
day and was utilised as follows with the remaining funds invested temporarily.

Asset X Asset Y
N'000 N'000

January 1, 2015 50,000 100,000

July 1, 2015 50,000 100,000

The loan interest rate is 9% per annum and Bode Limited can invest surplus funds at
7% per annum.
The financial accountant is not certain as to how these assets (X and Y) should be
accounted for in the financial statement of Bode Limited as at December 31, 2015.
(c) The company owns a building which it has been using as head office in Abuja. In order
to reduce cost, the company's management on June 30, 2015 decided to move the
head office to the branch office at Abuja and has now let out its head office building.
The company's accounting policy is to use fair value model for Investment Property.
The head office building had an original cost on January 1, 2006 of N37.5million and
was being depreciated over 50 years. As at December 31,
2015 the fair value of the head office building was assessed by an independent
valuer to be N52.5million.
The financial accountant is confused as to how these transactions should be treated in
the financial statements of the company.
Required:
Write a memo to the management of Bode Limited explaining how these transactions
should be accounted for in their financial statements. Provide relevant calculations where
necessary. (Total 20 Marks)

QUESTION 4
a. Explain the following, stating their importance to investors in the evaluation of
financial performance:
i. Earnings per share (EPS)
ii. Price earnings ratio (PE-ratio) (6 Marks)
b. The issued and fully paid share capital of Almond Nigeria Limited which has
remained unchanged since the date of incorporation until the financial year ended
March 31, 2015 include the following:
(i) 2,400,000,000 ordinary shares
(ii) 600,000,000 6% participating preference share of N1 each.
The company has been operating at a profit for a number of years. As a result of a
very conservative dividend policy in the previous years, there is a large accumulated
profit balance on the statement of financial position.
On July 1, 2015, the directors decided to issue to all ordinary shareholders two bonus
shares for every one previously held.
The following is the extract of group statement of profit or loss and other
comprehensive income for the year ended March 31, 2016.
Almond Nigeria Limited Extract of Group Statement of Profit or Loss and other
Comprehensive Income for the year ended March 31, 2016
2016 2015
N'000 N'000
Profit for the year 740,000 540,000
Other comprehensive income -- (20.000)
Total comprehensive income 740.000 520.000
Total comprehensive income attributable to:
Owners of parent 680,000 480,000
Non-controlling interest 60,000 40,000
740.0 520.000
The following dividend have been paid or declared at the end of the period.
2016 2015
N'000 N'000
Ordinary 330,000 240,000
Preference 69.000 60.000
The participating preference shareholders are entitled to share profit in the same
ratio in which they share dividends after payment of fixed preference dividend. The
preference shareholders will share the same benefit as the ordinary shareholders of
the company should the company be liquidated.
Required:
i. Calculate the earnings per share (EPS) in accordance with 1AS 33 and
the dividend per share (DPS) for the year ended March 31, 2015 and 2016.
(10 Marks)
ii. What are the limitations of earnings per share (EPS) as a measure of a
company's performance? (4 Marks)
(Total 20 Marks)
SECTION C: YOU ARE REQUIRED TO ANSWER ANY TWO OUT OF THREE
QUESTIONS IN THIS SECTION (30 MARKS)

QUESTION 5
The difference between debt and equity in an entity's statement of financial position is not
easily distinguishable for preparers of financial statements. Debts and equity financial
instruments may have similar characteristics, which may lead to inconsistency of reporting.
Required:
a. Discuss the main distinguishing features in the presentation of debt and
equity under International Financial Reporting Standards (IFRS) with clear
examples. (10 Marks)
b. Explain why it is important for entities to understand the impact of the
classification of a financial instrument as debt or equity in the financial statement.(5 Marks)
(Total 15 Marks)

QUESTION 6
a. IFRS 5 - Non-Current Assets Held For Sale and Discontinued Operations set out
requirements that specify the accounting treatment for assets held for sale and the
presentation and disclosure of discontinued operations.
Required
Explain the conditions that must apply at the reporting date for an asset (or disposal
group) to be classified as held for sale and how the assets can be measured. (5 Marks)
b. i. Explain how impairment of asset should be identified and
accounted for at the end of a reporting period. (4 Marks)
ii. A company has decided to dispose off a group of its assets. The carrying
amounts of the assets immediately before the classification as held for sale
were as follows:-
N
Goodwill 800,000
Property, plant andequipment (at revalued amounts) 3,050,000
Property, plant and equipment (at cost ) 3,200,000
Inventory 840,000
Other current assets 700.000
Total 8,590,000
The company estimates that the "fair value less cost to sell" of the disposal
group is N6,400,000.
Required:
Calculate the impairment loss and its allocation to the non-current assets in
the disposal group. (6 Marks)
(Total 15 Marks)

QUESTION 7
a. IAS - 37 applies to all provisions and contingencies apart from those covered by the
specific requirement of other standards.
Therefore provisions differ from other liabilities because there is uncertainty about
timing or amount of the future cashflow required to settle the liability.
Required:
Explain the criteria for recognition of provisions in the financial statements and
distinguish between provisions and contingent liabilities. (6 Marks)
b. The following activities took place in THREE different companies:
(i) Otapiapia Plc a Rat Trap Company based in Nigeria has just secured exportation
of rat killers to South Africa. The advertising slogan of the rat killers is "KILL
the BLACKS". A South African anti-racist movement with representative in
Nigeria is claiming N-15,000,000 from the company as damages because the
advertising slogan allegedly compromises the dignity of black people. The
company's legal representative believes that the success of the claim will
depend on the judge who presides over the case. They estimate however, that
there is a 70 percent probability that the claim will be thrown out and a 30
percent probability that it will succeed.
(ii) Ire-Akari Motors Plc is a Nigerian company that specialises in the manufacture
of "made-in-Nigeria cars".
During the current financial year 100 cars have been completed and sold.
During the testing of the cars a defect was found in their steering mechanism.
All the 100 customers that bought the cars were duly informed of the defect and
were told to bring their cars back to have the defects repaired at no cost. All the
customers have indicated that this is the only remedy they require. The
estimated cost of the recall is N10.5m.
The manufacturer of the steering mechanism, a quoted company with sufficient
fund has accepted responsibility for the defect and has undertaken to reimburse
Ire-Akari Motors Plc all cost that it might incur in this regard.
(iii) Abeokuta Electricity Company Plc sold a number of electricity transformers
with a warranty in the year ended December 31, 2015. At the beginning of the
year the provisions for warranty stood at N5,625,000.
A number of claims have been settled during the period for N3,000,000.
As at the year-end there were unsettled claims for 300 customers. Experience is
that 40% of the claims submitted do not fulfil warranty conditions and can be
defended at no cost. The average cost of settling other claims will be N52,500
each.
Required:
Explain how the matters in b(i) to b(iii) above should be accounted for in the
financial statements of the three companies using figures to illustrate your
points where appropriate. (9 Marks)
(Total 15 Marks)
SUGGESTED SOLUTION
SOLUTION 1
a. Bello Professional Nigeria Limited
Statement of cashflow for the year ended March 31, 2015
N'000
Cashflow from operating activities
Loss Before Tax (13,500)
Adjustments:
Depreciation 27,750
Loss on disposal of leasehold plant (w1) 750
Finance cost 7,500
22,500
Operating profit before working capital changes
Increase in Inventory (59,250)
Increase in trade receivables (18,750)
Increase in sundry receivables (3,750)
Increase in trade payables 3.750
Cash generated from operating activities (55,500)
Income Tax Paid (w2) (10,500)
(66,000 )
Net Cash used in operating activities
Cashflow from investing activities
Proceeds from disposal of leased hold plant 63,750
Net Cash generated from investing activities 63,750
Cashflow used financing activities
Issue of Shares (w4) 9,000
Dividend paid (w5) (5,250)
Finance cost paid (5,250)
Finance Lease Repayment (w6) (18.000)
Net Cash generated from financing activities (19.500)
Net Increase in cash and cash equivalent (21,750)
Cash and cash equivalents b/f 11,250
Cash and cash equivalents c/f (10,500)
Analysis of Cash and Cash Equivalents
At March Change in At March 31, 2014the year
31, 2015
N'000 N'000 N'000
Cash at bank 11,250 (11,250) -
Bank overdraft ( 10,500) (10.500)

11. 250 (21.750) (10.500)


Working (Notes)
w1. Leasedhold Plant N'000
Balance B/F 66,000
Depreciation (1.500)
Carrying amount at disposal 64,500
Disposal proceeds (63,750)
Loss on disposal Z50
w2. Taxation Account
Balance B/F - Current Tax 18,750
- Deferred Tax 6,000
Income statement (5,250)
Balance C/f - Current Tax -
- Deferred Tax 9.000
Tax Paid 10.500

w3. Leased Plant


Balance B/f (18,750)
Depreciation 13,500
(5,250)
Balance C/F 48.750
Leased Plant(w6) 43.500

w4. Ordinary Shares


Balance as at March 31, 2014 60,000
Bonus Issue: 1 for 10 or (30,000 - 24,000) 6.000
66,000
Balance as at March 31, 2015 75.000
Cash issue at Par 9.000

w5. Retained Earnings


Balance as at March 31, 2014 47,250
Loss for the year (8.250)
39,000
Balance as at March 31, 2015 (33,750)
Dividend paid 5.250
w6. Finance Lease on Leasehold plant
Balance as at March 31, 2014 (6,000 +
15,000) 21,000
Leased Plant(w3) 43,500
Finance lease charges 2,25
0
Balance as at March 31, 2015 66,750
(36,000 + 12,750) 48.750
Finance Lease repayment 18.000

(b) Direct method of statement of cash flow compared with indirect method
In accordance with IAS 7, there are two methods of presenting the statement
of cash flows with respect to operating activities. These methods are "Direct
method" and "Indirect method".
i. Direct method of preparing cash flow disclose major class of gross
receipts and gross cash payments.
ii. Direct method shows the items that affected cash flow and the size of
those cash flows, cash received from and cash paid to specific sources such
as customers and suppliers.
iii. Another observed advantage of direct method is that users see and
understand the actual cashflows and how they relate to items of income and
expenses.
iv. From the view point of the users, the direct method is preferable
because it discloses information not available elsewhere in the financial
statements, which could be used in estimating future cashflow while indirect
method involves adjusting the net profit or loss for changes in non-cash
expenditure and movement in working capital.
v. Direct method tells the reader whether cash collections from
customers are increasing or decreasing.
vi. Direct method shows ability to compare similar types of cash receipts and
payments across companies at least annually.
vii. Direct method gives better representation of an entity's cash cycle for
credit guarantors and more user - friendly format for managers.
viii. Direct method highlights the differences between net income and net cash
from operating activities whereas the indirect method is most useful in
extracting the lead and lay between cashflows and income information.
ix. Direct method highlights the operating changes in non-cash working capital
accounts.
x. Direct method assists the users in determining the reasons for the differences
between net income and associated cash receipts and payments to provide a
basis for evaluation.
xi. Indirect method reconciles operating profit to net operating cashflow.
xiii. Indirect method is easier to prepare.
xiv. It is simple for users to analyse.
Conclusion
Therefore, from the above analysis, l agree with the opinion of the directors that
direct method would be more preferable and more useful to users of financial
statements, although, the IFRS i.e. IAS 7 on statement of cashflow permits the use
of both methods.
(c) Ways of treating interest paid and dividend received in statements of cashflow as
permitted by IAS 7
i. Interest payments
Interest payment may be classified as either:
• An operating cashflow, because they are deducted when
calculating operating profit before taxation or
• A financing cashflow, because they are cost of obtaining finance.
ii. Dividend received
• An operating cashflow,because they are added when
calculating operating profit before taxation.
• An investing cashflow, because they represent return on
investment.

EXAMINER'S REPORT
The question tests candidates knowledge of IAS 7 on statement of cashflow. Part (a)
requires candidates to prepare a statement of cashflow using the indirect method. Part (b)
involves an appraisal of the direct and indirect methods of presenting statements of
cashflow and to give reasons why one method is more useful than the other. Part (c)
requires candidates to explain the various methods of classifying cashflows relating to
interest paid and dividend received.
All the candidates attempted the question but performance was very poor. Only about 20%
of the candidates obtained up to 50% of the 30 marks allocated to the question.
Candidates' commonest pitfall was their inability to correctly classify cashflows in the
statement of cashflow. Candidates did not address the question asked in part (b) and (c)
which suggests inadequate knowledge of the provisions of IAS - 7 on statement of
cashflows.
Candidates are advised to cover adequately all sections of the syllabus for this paper when
preparing for the examination in future.
MARKING GUIDE
Marks Marks

Statement of cashflow
- Determination of cashflows from operating activities 7
- Determination of cashflows from investing activities 1
- Determination of cashflows from financing activities 7
- Analysis of cash and cash equivalent 318
Assessment of Director's comment:
- Arguments in favour of Direct or Indirect method 3
- Arguments against Direct or Indirect method 2
- Conclusion 2 7
Ways of treating interest paid and dividend received in the
statement of cashflow:
- Classification of interest paid 2%
- Classification of Dividend received 2% 5
Total marks 30
SOLUTION 2

(a)
Non controlling interest N'000
NCI value of acquisition 14,940
NCI shares of post acquisition reserves
20% x (62,640 - 53,280) 1,872
16,812
Impairment (Balancing figure) (2.592)
NCI at reporting date 14.220

(bi)
Goodwill N'000
Abuja Limited parent
Investment at fair value 57,600
Cash (80% x 14,400 x N5)
Deferred consideration 19,440
77,040
NCI value at acquisition 14.940
91,980
Net assets at acquisition(bii) (53,280)
38,700

,2,590.8)
Less: Impairment (2591 x (10,368)
2
1
0.2
Good will 28.332
(bii) Net assets Abaji Limited
At At
Acquisition Reporting
1/1/15 31/12/15
N'000 N'000
Share capital Share 14,400 14.400
premium Retained 7,200 7,200
Earnings 24,120 37,800
45,720 59.400

Fair value adjustments:


Brand 7,200 7,200
Amortisation of brand (720)
Research and
Development (3,240) (7,200)
unrealised Unrealised
profit (1/3 x 1,080)
13601
Land and building - revaluation " " - further revaluation
49,680 58,320
3,600 3,600
53.280 720
62.640

(c) consolidated reserves N'000


i. Consolidated share premium (parent company only) 14.400
N'000
ii. Consolidated retained earnings - parent 63,000
Add interest receivable (10,800 x 8%) 864
63,864
Group share of post acquisition in subsidiary excluding revaluation
on land and building taken to revaluation reserve 80% x (58,320 -
49,680) 6,912
Less impairment of investment & others 80% x(57,600 -
54,0) +12,960 15.840
54.984
iii. consolidated revaluation reserve
Parent 8,100

Group share of fair value of land and building 80% (3,600 +


720) 3,406
11.556
N.B: (i) Cost of investment less Loan to subsidiary = N54.000
(ii) All figures are stated in N'000

EXAMINER'S REPORT
This question tests candidates knowledge of group or consolidated financial statements. It
requires candidates to calculate the non-controlling interest, goodwill and consolidated
reserve figures to be included in the consolidated statement of financial position.
Most of the candidates attempted the question and their performance was below average.
Only about 30% of the candidates obtained 50% of the 20 marks allocated to the question.
The candidates' commonest pitfall was their inability to adjust correctly for the fair value
adjustments of some assets at the acquisition and reporting dates. Some candidates mixed-
up the parameters for calculating goodwill with that of calculating non-controlling interest.
Candidates are advised to cover all aspects of group accounts when preparing for
examination in this paper.

MARKING GUIDE

Marks

(a) Calculation of non-controlling interest 7


(b) Calculation of goodwill 7

(c) i. Calculation of consolidated share premium 1

ii. Calculation of consolidated earnings 4

iii. Calculation of consolidated reserve 1

Total marks 20
SOLUTION 3

From: XYZ (Financial Reporting Consultant)


To: Management of Bode Limited
Date: 21/11/2016
Subject: Explanation of Accounting Treatment of some issues
Please, find below details of our explanation on three main issues raised by you for the
treatment of some accounting issues.
The issues required the applications of some International Financial Reporting Standards
(IFRS) which l have taken time to explain below.
(a) The impairment loss should be allocated across the assets of the cash generating unit
according to IAS 36 on impairments.
- The financial accountant should determine the total impairment loss which is
(N7.2billion less N6.3billion) i.e. N900million.
- Out of this impairment loss N450million should be allocated to Goodwill to
write it down to Nil balance.
- The balance of N450million should be allocated to Property, Plant and
Equipment and other assets as follows:

i. Property, Plant & Equipment N450million x 4,050m = N270m


6,750m
ii. Other Assets 450million x 2,700m = 180million
6. 750m
450million

The carrying amount of the assets that should be shown in the financial statement of
BODE Limited as at December 31, 2015 should be shown as follows:

Cost Impairment Carrying Amount


N'm N'm N'm
Property, Plant & Equipment 4,050 (270) 3,780
Goodwill 450 (450) Nil
Other assets 2,700 (180) 2,520
7.200 900 6.300
(b) In a situation where the company, BODE Limited, borrowed money to finance the
production of two assets. IAS 23 states that the borrowing costs that are directly
attributable to acquisition, construction or production of qualifying assets must be
identified and capitalised.
The borrowing cost that should be capitalised with cost of the asset X and Y as at
December 31, 2015 are as follows:

Asset X Asset Y
Borrowing cost N'000 N'000
X = (N100m x 9%) 9,000
Y = (N200 x 9%) Less: Investment Income 18,000

X = (N50m x 7% x 6/12) (1,750)


Y = (N100m x 7% x 6/12) (3r500)
Net borrowing cost 7,250 14.500

Therefore, the cost of assets X and Y of BODE Limited that should be shown
the statement of financial position as at December 31 is:
X Y
N,000 N,000
Cost of assets 100,000 200,000
Borrowing cost to be capitalised 7r250 14.500
107.250 214.500

(c) Reclassification of owner occupied property to investment property (on fair value
model).
IAS 40 requires the following to be done when there is a change in use which
necessitates that an owner occupied property be reclassified as investment property
to be accounted for using the fair value model.
i. Ascertain the fair value of the property as at the date of change in use.
ii. The difference between the fair value ascertained in (i) and its carrying
amount at the same date should be accounted for in accordance with IAS 16.
This implies that a gain (excess of the fair value over the carrying amount)
should be recognised in other comprehensive income from where it would be
included in equity via revaluation surplus reserve. Conversely, a loss (excess
of carrying amount over the fair value) should be recognised in profit or loss.
iii. From the date of reclassification, the property is no longer depreciable
i. e. depreciation would no longer be charged thereon.
iv. Every reporting date therefore, the fair value of the property should be
re-measured and gain or loss recognised in other comprehensive income
from where it would be included in equity via revaluation surplus reserve.
Conversely, a loss (excess of carrying amount over fair value) should be
recognised in profit or loss.
The building was initially a non-current asset under IAS 16: Property, Plant
and Equipment up to June 30 2015 when there was a change in use as
investment property from this date, IAS 40 becomes applicable.
In view of the above, the company's building, investment property will be
accounted for as shown below:
N'm N'm
Fair value as at 31/12/2015 52.5
Less:
Fair value as at 30/06/2015 assumed equals
to carrying value as at that date

Cost 37.5
Depreciation 1/1/2006 to 30/06/2015
(375x 10% years ) i79 296

Fair value gain on investment property 22.9


Extract of statement of Profit or Loss and other Comprehensive Income
N'm
Depreciation (0.4)
Fair value gain on investment property 22.9
Extract of Statement of financial position
Non-current assets N'm
Investment property 52.5
Conclusion
Please do not hesitate to contact us should you require further explanations on the above.

Thank you.

Mr. XYZ
Financial Reporting Consultant
EXAMINER'S REPORT

The question tests candidates' knowledge and understanding of the applications of three
accounting standards: IAS 36 on impairment, IAS 23 on borrowing costs and IAS 40 on
investment properties. The question requires candidates to apply the provisions of the
different standards to three scenarios.
About 50% of the candidates attempted the question, candidates' showed lack of
understanding of the requirements of the question as their overall performance was very
poor.
Candidates' commonest pitfall was their inability to present their solutions in MEMO form
as demanded by the question. Some candidates also lost vital marks due to non-
denomination of the relevant amounts in million or thousand naira.
Candidates are advised to ensure adequate preparation by paying special attention to
relevant financial reporting standards examinable at this level of the examination.

MARKING GUIDE
Marks Marks
(a) Impairment of assets - IAS 36
- Determination of
the impairment loss 1
- Allocation of
impairment loss to goodwill 1
- Calculation and allocation of impairment loss to PPE
and other assets 2
- Determination of carrying amount of assets which will
appear in the financial statement 2 6
(b) Borrowing cost - IAS 23
- Determination of borrowing cost to be capitalised 3
- Determination of cost of assets to be shown in the
statement of financial position 3 6
Investment property - IAS 40
(c) Reclassification of owner occupied property to
investment property (on fair value model):
- Ascertain the fair value of the property at the date of
change in use 1
- Determine the gain or loss in fair value i.e. excess of
carrying amount over the fair value 3
- Extract of statement of profit or loss and other
comprehensive income 1
- Extract of statement of financial position 1 6
c Memo format conclusion 2
Total marks 20
SOLUTION 4

(a) i. Earnings Per Share


- Earnings are profits available for equity holder. Earnings per share is a measure
of the amount of earnings in a financial period for each equity share.
EPS = Net profit attributable to ordinary shareholders
Weighted average no of ordinary shares outstanding during the period
importance
- EPS is used by investors as a measure of the performance of companies in
which they invest or wish to invest.
- it reveals a lot about the financial health of the company.
- it helps in the choice of shares or stocks to acquire.
- it serves as input in the calculation of Price Earnings Ratio.
- it gives a more accurate picture of the actual returns to investors than reported
profits.
- EPS can have a significant effect on a company's share price.
- EPS serves as a means of assessing the stewardship and management of the
company.
ii. Price Earnings Ratio (P/E) Ratio
- Price Earnings Ratio is a measure of the company's current share price (market
price) in relation to the EPS.
P/E Ratio = Market Price of Share
EPS
importance
- it can be used by investors to determine whether the share is expensive or cheap.
- It is an indication of the future strength of the company's performance.
- A high P/E Ratio gives the investors confidence in the stock, and they are
prepared to pay high price for the stock.
- Higher P/E Ratio is an indication of a strong corporate governance in a
company.
- it indicates the ability of the company to pay high and stable dividends to
shareholders.
b. i. EARNINGS PER SHARE (EPS)
March, March 31,
31 2015
2016 N'000
N'000 440.000
Profit attributable to ordinary shareholders (wk 2) 640.000 3,000,000
Weighted No of shares (w3) 7,800,000 N0.147
Earnings per share (EPS) N0.082

Dividend Per Share (DPS)


Ordinary Shares:
Dividend (w1) 330.000 240.000
No of Ordinary Shares 7,200,000 2,400,000
(w3) DPS N0.046 N0.1

Preference shares:
Dividend (w1) 69.000 60.000
No. Or ordinary shares 600,00 600,000
(w3) DPS 0 N0.10
N0.115

WORKINGS
W1 Percentage of Profit attributable to class of equity shares
2016 2015
N'000 N'000
Total pref. Dividend 69,000 60,000
Fixed portion (6% of 600,000) (36,000) (36,000)
33.000 24.000
Dividend paid to ordinary shareholders 330,000 240,000
Therefore, the participating preference shareholders will share profit in the
ratios of 1:10 (33,000 : 330,000) or (24,000 : 240,000) with ordinary
shareholders after payment of the fixed preference dividend out of the profit.
W2
Earning per each class of share
2016 2015
N'000 N'000
Net profit for the year 740,000 520,000
Fixed Pref. Dividend (36.000) (36.000)
704.0 484.000
2016 2015
N'000 N'000
Net profit attributable to ordinary 640.000 (10/11 440.000
shareholders (10/n of 704,000) of
Net profit attributable to participating 484,00
shareholders (1/11 of 704,000) 0)
Fixed dividend (Vnof
64.000 484,000 44.000
36.0 1 ) 36.000
00.000 80.000

W3
Weighted number of shares in issue
Ordinary shares: 2016 2015
Balance April 1, 2014 '000 „000
Capitalization (Bonus) 2.400.000 2,400,000
Participating Preference shares 4.800.000
7.200.0 2,400,000
600,000 600.000
7.800.00 300.000

ii. Limitations of Earnings Per Share (EPS)


• Not all entities use the same accounting policies. It may not always be
possible to make meaningful comparison between EPS of different
companies.
• EPS does not take account of inflation, so that, growth in EPS over time
might be misleading.
• EPS measures an entity's profitability, but this is only part of an entity's
overall performance.
• Diluted EPS is based on current and not on forecast earnings, therefore not a
reliable predictor of future EPS.
• EPS is relied on by investors as the main measure of an entity's performance.
Hence, management try to make EPS appear as high as possible by
attempting to manipulate the figures using "creative accounting" as well as
making decisions which will increase EPS in the short term, but which
damage the entity in the longer term.
EXAMINER'S REPORT
The question tests candidates' knowledge of the performance ratios: Earnings Per Share
(EPS), Price Earnings Ratio (P/E) and Dividend Per Share (DPS). Part (a) requires
candidates to explain and state the importance of EPS and PE ratios to the investor. Part (b)
requires the calculation of EPS and outlining its limitations as a measure of company's
performance.
About 30 percent of the candidates attempted the question and demonstrated fair
understanding of the requirements of the question and performance was fairThe
commonest pitfall was candidates inability to determine correctly the ratio of participation
in the residual profit by the participating preference shareholders. Some other candidates
could neither explain the ratios as required nor correctly state the limitations of the ratios.
Candidates are advised not to undermine any aspect of the syllabus or focus on some
particular ratios. Adequate coverage of the syllabus is therefore recommended.

MARKING GUIDE
Marks
(a) Explanation and importance of ratios
- Explanation of Earnings Per Share 1
- Explanation of Price Earnings Ratio 1
- importance of EPS in performance evaluation
(any 4 points at % mark each) 2
- importance of PE Ratio in performance evaluation
(any 4 points at % mark each) 2 6
(b) i. Calculation of Earnings Per Share (EPS) 1%
- Calculation of Dividend Per Share - Ordinary 1%
- Calculation of Dividend Per Share - Preference 1%
- Workings for Dividend per share 2
- Workings for Earnings per share 3% 10
(c) Limitations of Earnings Per Share
(any 4 points at 1 mark each) 4
Total marks 20
SOLUTION 5

(a)
- IAS 32 Financial Instruments Presentation establishes principles for presenting
financial instruments as liabilities or equity. To determine whether a financial
instrument should be classified as debt or equity, IAS 32 uses principles-based
definitions of a financial liability and of equity. In contrast to the requirements of
generally accepted accounting practice in many jurisdictions around the world, IAS
32 does not classify a financial instrument as equity or financial liability on the basis
of its legal form. The key feature of debt is that the issuer is obliged to deliver either
cash or another financial asset to the holder. The contractual obligation may arise
from a requirement to repay principal or interest or dividends.
- Such a contractual obligation may be established explicitly or indirectly through the
terms of the agreement. For example, a bond which requires the issuer to make
interest payments and redeem the bond for cash is classified as debt. In contrast,
equity is any contract which evidences a residual interest in the entity's assets after
deducting all of its liabilities. A financial instrument is an equity instrument only if
the instrument includes no contractual obligation to deliver cash or another financial
asset to another entity and if the instrument will or may be settled in the issuer's own
equity instruments. For example, ordinary shares, where all the payments are at the
discretion of the issuer, are classified as equity of the issuer. The classification is not
quite as simple as it seems. For example, preference shares required to be converted
into a fixed number of ordinary shares on a fixed date or on the occurrence of an
event which is certain to occur, should be classified as equity.
- A contract is not an equity instrument solely because it may result in the receipt or
delivery of the entity's own equity instruments. The classification of this type of
contract is dependent on whether there is variability in either the number of equity
shares delivered or variability in the amount of cash or financial assets received. A
contract which will be settled by the entity receiving or delivering a fixed number of
its own equity instruments in exchange for a fixed amount of cash or another
financial asset is an equity instrument. However, if there is any variability in the
amount of cash or own equity instruments which will be delivered or received, then
such a contract is a financial asset or liability as applicable.
- For example, where a contract requires the entity to deliver as many of the entity's
own equity instruments as are equal in value to a certain amount of cash, the holder
of the contract would be indifferent whether it received cash or shares to the value
of that amount. Thus this contract would be treated as debt.
Other factors, which may result in an instrument being classified as debt, are:
i. redemption is at the option of the instrument holder;
ii. there is a limited life to the instrument;
iii. redemption is triggered by a future uncertain event which is beyond
the control of both the holder and issuer of the instrument; and
iv. dividends are non-discretionary;
Similarly, other factors, which may result in the instrument being classified as
equity, are whether the shares are non-redeemable, whether there is no liquidation
date or where the dividends are discretionary.
(b) The importance of entities understanding the impact of the classification of a
financial instrument as debt or equity in the financial statements.
When an entity issues a financial instrument, it must determine its classification
either as a liability (debt) or as equity. That determination has an immediate and
significant effect on the entity's reported results and financial position.
i. Liability classification affects an entity's gearing ratios and typically results
in any payments being treated as interest and charged to earnings.
ii. Equity classification avoids these impacts but may be perceived negatively
by investors if it is seen as diluting their existing equity interests.
iii. Understanding the classification process and its effects is therefore a critical
issue for management and must be kept in mind when evaluating alternative
financing options.
iv. This may in turn affect the entity's ability to pay dividends on its equity
shares (depending upon the requirements of local law).
v. Equity classification avoids the negative impact that liability classification
has on reported earnings and gearing ratios. It also results in the instrument
falling outside the scope of IAS 39 Financial Instruments: Recognition and
measurement, thereby avoiding the complicated ongoing measurement
requirements of that standard.
EXAMINER'S REPORT

The question tests the Provisions of IAS 32 - Financial Instrument presentation. Candidates
are required to discuss the main distinguishing features in the presentation of debt and
equity as well as state the importance of the impact of classification of debt or equity in the
financial statements.
Less than forty percent (40%) of the candidates attempted the question and performance
was poor.
Candidates' commonest pitfalls include the following:
• Explaining the differences between equity holders and debenture holders instead of
stating the distinguishing feature in the presentation of debt and equity as stated
in International Financial Reporting Standards (IFRS).
• Inability of candidates to explain the importance of understanding the impact of
classification of financial instruments as debt or equity in the financial statements.
The failure to make use of 1CAN Study texts as well as familiarising themselves with all
relevant accounting standards at this level of the Institute examinations, led to the poor
performance, hence candidates are advised to pay more attention to recommended study
texts and ensure that they cover the syllabus for better performance in future examinations.

MARKING GUIDE
Marks
(a) Discussion on the main distinguishing features in the
presentation of debt and equity under 1FRS (any 10
points at 1 mark each)

10
(b) Explanation of the importance for entities to understand
the impact of the classification of financial instrument as
debt or equity. (any 5 points at 1 mark each)
5
Total marks
1
5
SOLUTION 6

(a) The following conditions must apply at the reporting date for an asset (or disposal
group) to be classified as held for sale:
(i) It must be available for immediate sale in its present condition subject only
to terms that are usual and customary for sales of such assets (or disposal
group);
(ii) The sale must be highly probable i.e:
• The appropriate level of management must be committed to a plan to sell
the asset (or disposal group);
• An active programme to locate a buyer and complete the plan must have
been initiated; and
• The asset (or disposal group) must be actively marketed for sale at a
price that is reasonable in relation to its current fair value.
(iii) The sale must be expected to be completed within one year from the date of
classification (except in limited circumstances) and actions required to
complete the plan should indicate that it is unlikely that significant changes
to the plan will be made or that the plan will be withdrawn.
If the criteria are met for a non-current asset (or disposal group) after the reporting
date but before the authorization of the financial statements, that asset must not be
classified as held for sale as at the reporting date. However, the entity is required to
make certain disclosures in respect of the non-current asset (or disposal group).

(b) How impairment of assets should be identified and accounted for


(i) At the end of each reporting period, the entity should assess whether there
are any indications that an asset may be impaired.
(ii) if there are such indications, the entity should estimate the assets recoverable
amounts.
(iii) When the recoverable amount is less than the carrying value of the assets the
entity should reduce assets' carrying value to its recoverable amount. The
amount by which the value of the assets is written down is an impairment
loss.
(iv) The impairment loss is recognized as loss for a period.
(v) However, if the impairment loss relates to an asset that has previously
been re-valued upward, it is first off-set against any remaining revaluation
surplus for that asset, when this happens, any difference
is charged to other comprehensive income for the period.
(vi) Depreciation charges for the impaired assets in future periods should be
adjusted to allocate the assets revised carrying amount minus any residual
value, over its remaining useful life (revised if necessary).

b(ii) Impairment loss


Carrying Impairment Carrying
Amount loss amount
before N after
Allocation 800,000 allocation
N 678,320 N
Goodwill 800,000 711,680
Property, plant and
equipment (at revalued
amount) - w 2 Property, plant
3.050.000 2,371,68
and equipment (at cost) - w3
3.200.000 0
Inventory
840.000 2,488,32
Other current assets Total
700.000 0
WORKINGS
8.590.000 840.000
(W 1) Impairment Loss:
700.0 6.4
N8,590,000 less N6,400,000 2.190.000
00.000

N2.190.000

(W 2) Allocations N
- Goodwill = 800,000
- Property, plant & equipment (revalued asset)
3,050,000x (2,190,000- 800,000) _
6,250,0 "" 678,320
- Property, plant and equipment (at cost)

.... , 3,200,000x (2,190,000- 800,000)


711,680
(W 3) -----------------------------------------------
2.190.000
6,250,000
EXAMINER'S REPORT

The question is made up of two parts. Part (a) tests candidate's knowledge of IFRS 5 - Non
Current Assets held for sales and discontinued operations, while Part (b) of the question
required the explanation of how impairment of assets should be identified and accounted
for as well as computation of impairment loss and its allocation to non- current assets in
the disposal group.
About 70% of the candidates attempted the question and performance was above average.
Majority of the candidates were able to calculate and allocate the impairment loss, but only
few of them could explain how impairment of assets are identified and accounted for.
Also, some candidates could not explain the conditions that must apply at the reporting
date, for an asset or disposal group to be classified as held for sale as required by IFRS 5.
Candidates are advised to study the 1CAN Study text very well while preparing for the
Institute's examination as question b(ii) was adopted from 1CAN Financial Reporting
Study text.

MARKING GUIDE
Marks Marks
(a) Explanation of the conditions that must apply at the
reporting date for an asset (or disposal group) to be
classified as held for sale: (any 5 points at 1 mark each)
5

(b) (i) Explanation of how impairment of assets should be


identified and accounted for:
(any 4 points at 1 mark each) 4
(ii) Calculation of impairment loss and its allocation to non-
current assets:
- Determination of impairment loss
- Allocation of impairment loss
- Determination of carrying amount of assets

Total marks 15
SOLUTION 7

(a) i Criteria For Recognition Of Provisions


A provision should be recognised when:
• A company has a present obligation (legal or constructive) as a result
of past events.
• It is probable that outflow of economic benefit will be required to
settle the obligation.
• A reliable estimate can be made of the amount of the obligation.
Please note that if one of these conditions is not met then provision cannot be
recognized.
ii .Differences between Provisions and Contingent Liabilities
Provisions: Are recognised as liabilities (assuming that a reliable estimates
can be made) because they are present obligations and it is probable that an
outflow of resources embodying economic benefit will be required to settle
the obligation.
Contingent Liabilities: Are not recognised as liabilities because they are
either:
■ Possible obligations
■ Present obligations that do not meet recognition criteria for provisions
because either:
■ It is not probable that an outflow of resources embodying economic
benefit will be required to settle the obligation; or
■ A sufficiently reliable estimate of the amount of the obligation cannot be
made.
(b)
i .Otapiapia Plc
■ The present obligation is as a result of a past event. The available
evidence provided by expert indicates that it's more likely that no present
obligation exist at statement of financial position date because there is a
70 percent probability that the claims will be thrown out. Hence no
obligation event has taken place.
■ In view of the above no provision should be recognised.
■ The matter may be disclosed as a contingent liability unless the 30 percent
probability is regarded as being improbable.
ii. 1re Akari Motors Plc
This is in two folds
Contingent Liability
There is an obligation between 1re Akari Motors Plc and its 100 customers
as it has acknowledged the defects and has also notified the customers and
accepted to repair the cars with an estimate of N10.5m repair costs.
This satisfies the recognition criteria of 1AS 37:
• There is a present obligation
• 1t is probable that economic benefit will flow out
• There is a reasonably reliable estimate of the cost.
1n view of the above, 1re Akari Motors should recognise a contingent
liability of N10.5m and make the provision accordingly in the financial
statement by debiting profit or Loss with N10.5m and crediting Contingent
Liability with N10.5m.
Contingent Assets
There is a probable asset because the manufacturer of the defective steering
mechanism has accepted responsibility for the defect with an undertaking to
reimburse 1re Akari Motors Plc accordingly for all costs incurred in respect
thereof. This is a demonstration of the manufacturer's willingness to meet the
obligation. Where some or all the expenditure required to settle a provision is
expected to be reimbursed by another party (in this case with an equal sum of
the estimated cost of N10.5million), the reimbursement shall be recognised
when, and only when, it is virtually certain that reimbursement will be
received if the entity settles the obligation.
The reimbursement shall be treated as a separate asset. The amount
recognised for the reimbursement shall not exceed the amount of the
provision.
1t is also virtually certain that economic resources will flow out to 1re Akari
Motors Plc as the manufacturer is a quoted company with sufficient fund.
This is evidential of the ability to meet the obligation.
1n view of the above, 1re Akari Motors Plc should recognise contingent
assets of N10.5m by debiting Contingent assets and crediting Profit or loss
with the same amount of N10.5m.
iii. Abeokuta Electricity Company Plc
iAS 37 states clearly that a warranty is a constructive obligation and
satisfies all the recognition criteria. As such Abeokuta Electricity Company
Plc should provide for the warranty claims as follows:
- Provide for 40% of 300 customers at N52,500 each as at the reporting
date, amounting to N6,300,000.
- Recognise the claims of N3,000,000 paid during the year in the provision
for warranty account with the brought forward of N5,625,000. The
current year provision will be N6,825,000.
This is a provision. The provision account will be shown in the Statement of
Financial Position and details shown in a note to the account as follows:

WARRANTY
N'000
At January 1, 2015 5,625
Used in the year (3,000)
2,625
Charged to:
Statement of Profit or Loss 6.825
Balance as at December 31, 2015 (w1) 9.450

Workings
(W1) 300 x (100-40) 60% x 52,500 N9.450.000
The company grant warranties on certain categories of goods. The
measurement of the provision is on the company's experience of likelihood
and cost of paying out under the warranty.

EXAMINER'S REPORT
The question tests candidates understanding of the provisions of iAS 37- Provisions and
Contingencies. Candidates are required to explain the criteria for recognition of provisions
in the financial statements as well as distinguish between provisions and contingent
liabilities. They are also required to apply the provisions of iAS 37 to three different
scenarios.
Majority of the candidates did not attempt the question and performance was very poor.
The commonest pitfall was the inability of the candidates to apply the provisions to the
scenarios provided in the question .
Most of the candidates lack understanding of the topic and could not determine whether
provisions should be made in the financial statement or not. Few of them that applied the
provisions could not justify their claims with appropriate figures to illustrate their points
and this led to loss of valuable marks.
Candidates are advised to familiarize themselves with all relevant Accounting Standards at
this level of the Institute examinations for better performance in future.

MARKING GUIDE
SOLUTION 7 Marks Marks
(a) (i) Explanation of criteria for recognition of provisions
(any 3 points at 1 mark each) 3
(ii) Differences between provisions and contingent
liabilities 3
(any 6 points at y2 mark each)
(b) Explanation of the treatment of provisions and
contingent liabilities in the given scenarios:
(i) Otapiapia Plc
Review of scenario and conclusion
(any 3 points at 1 mark each) 3
(ii) Ire Akari Motors Plc
Review of scenario and conclusion
(3 points at 1 mark each) 3
(iii) Abeokuta Electricity
Company Plc Review of scenario and
conclusion
(any 3 ticks at 1 mark each) 3
9
Total marks 15
THE INSTITUTE OF CHARTERED ACCOUNTANTS OF NIGERIA
SKILLS LEVEL EXAMINATION - NOVEMBER 2017 FINANCIAL REPORTING
Time Allowed: 3V4 hours (including 15 minutes reading time)
INSTRUCTION: YOU ARE REQUIRED TO ANSWER FIVE OUT OF SEVEN
QUESTIONS IN THIS PAPER
SECTION A: COMPULSORY QUESTION (30 MARKS)
QUESTION 1
On April 1, 2017 Higherhigher Limited acquired 60% of the equity share capital of
Lowerlower Limited in a share exchange of two shares in Higherhigher for three shares in
Lowerlower. The issue of shares has not yet been recorded by Higherhigher Limited. At the
date of acquisition, shares in Higherhigher had a market value of N6 each.
Below is the summarised draft financial statements of both companies.

Statement of Profit or Loss and other Comprehensive Income for the year ended September
30, 2017
Higherhigher Lowerlower
Limited Limited
N'000 N'000
Revenue 2,720,000 1,344,000
Cost of sales (2,016,000) (1,024,000)
Gross profit 704,000 320,000
Distribution costs (64,000) (64,000)
Administrative expenses (192,000) (102,400)
Finance costs (9.600) (12.800)
Profit before tax 438,400 140,800
Income tax expense (150,400) (44,800)
Profit for the year 288.000 96.000

Statement of Financial Position as at September 30, 2017


Higherhigher Lowerlower
Limite Limite
d d
N'000 N'000
Assets
Non-current assets:
Property, Plant & Equipment 1.299.200 5 403.200
Current assets Total Assets 12.000 211.200 6
Equity & Liabilities Equity 1.811.200 14.400
shares of N1 each Retained
earnings 320,000 128,000
1.132.800 208.000
1.452.800 336,000
Non-current liabilities 10%
Loan notes Current liabilities 96,000 128,000
Total equities and Liabilities 262,400 150.400
1.811.200 614.400

The following information is relevant:


(1) At the date of acquisition, the fair value of Lowerlower Limited's assets was equal to
their carrying amounts with the exception of an item of plant which had a fair value
of N64m in excess of the carrying amount and had a remaining life of five years at
that date; using straight line depreciation method. Lowerlower Limited has not
adjusted the carrying amount of its plant as a result of the fair value exercise.
(2) Sales from Lowerlower Limited to Higherhigher Limited in the postacquisition period
were N256m. Lowerlower Limited made a mark-up on cost of 40% on these sales.
Higherhigher Limited had sold N166.4m (at cost to Higherhigher Limited) of these
goods by September 30, 2017.
(3) Other than where indicated, profit or loss items are deemed to accrue evenly on a time
basis.
(4) Lowerlower Limited trade receivables at September 30, 2017 include N19.2m due
from Higherhigher Limited which did not agree with Higherhigher Limited's
corresponding trade payables. This was due to cash in transit of N6.4m from
Higherhigher Limited to Lowerlower Limited. Both companies have positive bank
balances.
(5) Higherhigher Limited has a policy of accounting for any non-controlling interest at
fair value. For this purpose fair value of the goodwill attributable to the non-
controlling interest in Lowerlower Limited is N48m. The consolidated goodwill was
not impaired as at September 30, 2017.

You are required to prepare:


a. Consolidated
statement of profit or loss and other comprehensive
income for the period ended September 30, 2017 (10 Marks)
b. Consolidated
statement of financial position as at
September 30, 2017. (
2
0

SECTION B: YOU ARE REQUIRED TO ANSWER ANY TWO OUT OF THREE M


QUESTIONS IN THIS SECTION (40 MARKS) a
QUESTION 2 r
Quadri Top Nigeria Plc. prepares annual financial statements to September 30. At k
September 30, 2017 the company list of account balances were as follows: s
DR CR )
N'000 N'000
Revenue 185,000 (
Production costs 103,500 T
Inventory at October 1, 2016 17,375 o
Distribution costs 13,500 t
Administration expenses 18,250 a
Loan Interest expenses 3,000 l
Land at valuation 131,250
Building - cost 100,000 3
Plant and equipment at cost 160,000 0
Accumulated depreciation - building at 1/10/16 26,625
Accumulated depreciation PPE at 1/10/16 31,000 M
Trade receivables 51,500- a
Trade payables 28,000 r
Bank overdraft 1,000 k
Issued ord. shares at 50k each (Sept.30, 2017) 175,000 s
Share premium at (Sept.30, 2017) 50,000 )
Revaluation surplus 37,500
Retained earnings 39,250
12% Loan notes (payable 2021) _____ 25,000
598.375 598.375

The following are relevant to the preparation of the financial statements for the year ended
September 30, 2017.
(1) Inventory at September 30, 2017 amounted to N19.5m.
(2) Depreciation is to be provided on cost of the non-current assets as follows:
Building 2% per annum
5
Plant & equipment 20% per annum
80% of the depreciation is to be charged to cost of sales and 10% each to distribution
cost and administrative expenses.

(3) Land is to be revalued to N125m.


(4) Accrued expenses and prepayments were

Accrued Prepayments
Expenses
N'00 N'00
Distribution cost 0 0
Administrative expenses 2,375 1,500
87 75
(5) During the year ended September 30, 2017, 5 100million ordinary
0 shares were issued at
75k per share. The directors of Quadri Top declared an interim dividend of 2k per
share in September 2017. No dividends were paid during the year.
(6) Loan interest is paid annually, on September 30 each year.
Required:
Prepare in accordance with 1AS1:
i. Statement of profit or loss and other comprehensive income for the year ended
September 30, 2017. (8 Marks)
ii. Statement of financial position as at September 30, 2017 (12 Marks)

(Total 20 Marks)

QUESTION 3
(a) The basic financial ratios can be grouped into the following broad categories.
• Profitability and efficiency
• Long term solvency and stability
• Short term solvency and liquidity
• Shareholders investment ratios
Required:
Briefly explain the main aims and give TWO examples of each category of the above
financial ratios. (8 Marks)
(b) The following is the financial information extracted from the records of Nwokeke
Nigeria Plc for the year ended March 31, 2017.
FINANCIAL INFORMATION EXTRACTS

N000
Inventories:
Raw materials 142,500
Work in 57,000
progress
Finished goods 190,000
Purchases 475,000
Revenue 855,000
Cost of goods sold 712,500
Trade receivables 218,500
Trade payables 114,000

Additional Information
The directors of Nwokeke Nigeria Plc are of the opinion that the average cash
operating cycles of companies that operate in the same industry as Nwokeke Plc is 75
days.
Required:
i. Explain the term cash operating cycle. (2 Marks)
ii. Calculate the cash operating cycle of Nwokeke Nigeria Plc. (4 Marks)
iii. Assess the performance of Nwokeke Nigeria Plc's cash management
relative to the industry average performance. (2 Marks)
iv. Suggest TWO steps that should be taken by the directors of Nwokeke Nigeria
Plc to improve the cash operating cycle of the company.
(4 Marks) (Total
20 Marks)

QUESTION 4
a. i Explain what is meant by the terms "associate" and "significant
influence" (2 Marks)
ii. Explain the equity method of accounting which is used to account for
investment in an associate. (2 Marks)
iii. Distinguish between joint operation and joint ventures. (2 Marks)
b. On October 31, 2013, Y Limited paid N70,000 to acquire 40% of the share
capital of Z Limited (which became its associate). Draft financial statements of the
two companies for the year to October 31, 2017:

Statement of comprehensive income for the year ended October 31, 2017
Y Z
Limited Limited
N000 N000
Operating profit 325 7
Dividend received from Z Ltd 10 0-
Profit before tax 335 7
0
Income tax expense (85) (15)
Profit for the Year 250 55
STATEMENT OF FINANCIAL POSITION AS AT OCTOBER 31,
2017
Y Z
Limited Limited
N000 N000
Assets
Non-current asset
Property, plant & equipment 800 400
Investment in Z Ltd at cost 70 -

870 400
Current asset 390 145
1260 545
Equity
Ordinary share capital 500 100
Retained earnings 605 360
1105 460
Liabilities
Current liabilities 155 85
1260 545
Statement of Changes In equity (retained earnings only) for the year to
October 31, 2017
Y Z
Limited Limited
N'000 N'000
Balance at October 31, 2016 355 330
Profit for the year 250 55
Dividend paid - (25)
Balance at October 31, 2017 605 360

The following information is also available:


(i) 1n the draft financial statements of Y Limited, the company's investment in Z
Limited has been recognised at cost and the dividend received from Z Limited has
been recognised as income. The financial statements, showed the situation as it
would be without application of the equity method, either in the year October 31,
2017 or in previous year.
(ii) The retained earnings of Z Limited on October 31, 2013 were N50,000 and all of
its assets and liabilities were carried at fair value. None of the companies has
issued any share since that date.
(iii) During the year to October 31, 2017 Y Limited bought goods from Z Limited for
N15,000, which had cost Z Limited N10,000. One-quarter of these goods were
unsold by Y Limited at October 31, 2017.
Required:
Prepare the separate financial statements of Y Limited for the year ended October
31, 2017, incorporating the result of the associate Z Limited, using the equity
method of accounting.
(14 Marks) (Total
20 Marks)

SECTION C: YOU ARE REQUIRED TO ANSWER ANY TWO OUT OF THREE


QUESTIONS IN THIS SECTION (30 MARKS)
QUESTION 5

a. It is a general application of the concept of prudence that the carrying amount of an


asset should not be greater than its recoverable amount. IAS 36 Impairment of Assets
gives guidance on the application of this principle particularly for non-current assets.
Under the rules in IAS 36, an asset must be written down to its recoverable amount
when this is less than its carrying amount. The standard ensures that impairment loss
is measured and recognised on a consistent basis.
Required:
Explain the need for the application of IAS 36 and the concept of recoverable amount
as contained in the standards. (7 Marks)
b. The determination of related party status depends on the substance of the
relationship, not just the legal form.
Required:
i. Define a "related party transaction" and give TWO examples of such
transactions. (4 Marks)
ii. In the context of IAS 24-Related Party Disclosures, state FOUR conditions
under which an entity can besaid to be related to another. (4 Marks)
(Total 15 Marks)

QUESTION 6
a. Snow-Ball Nigeria Plc is a manufacturer of school bags that are sold in most
Nigerian modern markets. The following transactions and errors occurred
during the year ended October 31, 2017.
i. As at the beginning of the year, the remaining useful life of the plant
and equipment of the company was reassessed as four years rather than seven
years. (2 Marks)
ii. Bonuses of N24million, compared with
N4.6million in the previous
year had been paid to employees. Thefinancialmanager explained
that a new incentive scheme was adopted whereby all employees
shared in increased sales. (2 Marks)
iii. D
uring the year the company was responsible for the formation of
"Back to School Foundation". This foundation forms part of the company's
social investment programme. The company contributed N14million to the
fund. (2 Marks)
Required:
Briefly explain how each of the above transactions would be treated in the statement of profit
or loss of Snow-Ball Nig. Plc for the year ended October 31, 2017, stating whether a separate
disclosure is required or not.

Note: No calculation is required. (6 Marks)

b. The inventory balance of Papaya Nigeria Limited are as follows for the year ended
March 31, 2017.

N000
Opening 28,87
Inventory Closing 5
Inventory 31,42
5

In the course of preparing the financial statements at March 31, 2017, the need for a
number of adjustments emerged as stated below:
(i) The opening inventory was found to have been overstated by N-3,135,000 as a
result of error in the calculation of values in the inventory sheets.
(ii) Some items included in the closing inventory at the cost of N120,000 were
found to be defective and were sold after the end of the reporting period for
N78,000. Selling cost amounted to N4,500.

Required:
i. Explain how adjustment should be made for errors in the opening inventory
according to IAS 8 - "Accounting policies changes in accounting estimates
and errors. NB: No calculation is required.
(2 Marks)
ii. State two disclosure required by IAS 8 in the financial statements as
at March 31, 2017 for the adjustments above. NB: No calculation is required:
(3 Marks)
iii. Show how the final figures for inventory should be presented in the
statement of financial position as at March 31, 2017. (4 Marks)

(Total 15 Marks)

QUESTION 7
Manilla Nigeria Plc leased an equipment from Capa Finance Limited. The terms of the lease
are as follows:

Inception of the Lease January 1, 2015


Lease term 4 years; N-788,640 per annum payable
in arrears.
Present value of minimum lease payments N-2,500,000
Useful life of the assets 4 years
Required:
a. Briefly explain the term "interest rate implicit in the lease "under IAS 17
(2 Marks)
b. Calculate the interest rate implicit in the above lease, using the table below.
(4 Marks)
This table shows the present value of N1 per annum, receivable or payable at the end
of each year for n years.

Years Interest Rate


(n) 6% 8% 10%
1 0.943 0.926 0.909
2 1.833 1.783 1.736
3 2.673 2.577 2.487
4 3.465 3.312 3.170
5 4.212 3.993 3.791

c. How should Manilla Nigeria Plc treat this type of lease transaction (give
reasons for your answer). (3 Marks)
d. Briefly discuss the effect of classifying a lease incorrectly in the income
statement and statement of financial position. (6 Marks)
(Total 15 Marks)
SOLUTION 1

HIGHER - HIGHER LTD GROUP


(a) STATEMENT OF CONSOLIDATED PROFIT OR LOSS AND OTHER
COMPREHENSIVE INCOME FOR THE YEAR ENDED SEPTEMBER 30, 2017
N'000
Revenue (W1) 3,136,000
Cost of sales (W2) (2.304.000)
Gross profit 832,000
Distribution cost (W3) (96,000)
Administrative expenses (W4) (243,200)
Finance cost (W5) (16,000)
Profit before tax 476,800
Income tax expense (W6) (172.800)
Profit for the year 304.000
Attributable to:
Equity holder of the parent (bal) 297,600
Non-Controlling Interest (W7) 6,400
304,000
N.B URP = Unrealised Profit
WORKINGS
N'000 N'000
W1 Revenue
Parents 2,720,000
6
Subsidiary (1,344,000 x /12) 672,000
Intra-group sales (256,000)
3.136.000
W2 N'000 N'000
Cost of Sales
Higher-Higher Ltd. 2,016,000
6
Lower-Lower Ltd. (1,024,000 x /12) 512,000
Intra Group Sales (256,000)
URP on Inventory (w1a) 25,600
Additional Depreciation (64,000 5
6
years x /12) 6.400
2.304.0
N'000 N'000
W2a. URP on Inventory (256m - 166.4m x 40_)
140 = 25.600
OR
W2b. CONSOLIDATIONSCHEDULE FOR PROFIT OR LOSS AND OTHER
COMPREHENSIVE INCOME
HH LTD LL LTD ADJUSTMENT CPL
N'000 N'000 N'000S N'000
Revenue 2,720,000 672,000 (256,000) 3,136,000
Cost of sales (2,016,000) (512,000) 230,400
(6.400) 2.304.000
Gross profit 704,000 160,000 (32,000) 832,000
Distribution cost (64,000) (32,000) (96,000)
Administration (192,000) (51,200) (243,200)
cost
Finance cost (9.600) (6.400) (16.000)
Profit before tax 438,400 70,400 (32,000) 476,800
Income tax exp. (150,400) (22,400) (172,800)
Profit for the year 288.000 48.000 (32.000) 304.000

W3 N'000 N'000
Distribution Cost
Parent 64,000
Subsidiary (64,000 x 6/12) 32.000
96.000
N'000 N'000
W4 Administrative expenses
Parent 192,000
Subsidiary (102,400 x 6/12) 51.200
243.200
W5 Finance Cost N'000 N'000
Parent 9,600
Subsidiary (12,800 x 6/12) 6,400
16.000
W6 Income tax expenses N'000 N'000
Parent 150,400
Subsidiary (44,800 x 6/12) 22,400
172.800

W7 Non-controlling interests N'000 N'000


Share of subsidiary's profit (96,000 x 6/12)
48,000
Unrealized profit 25,600
Depreciation adjustment 6.400
16.000
NCI Consolidated = (16,000 x 40%) 6,400
HIGHER-HIGHER LTD GROUP STATEMENT OF FINANCIAL POSITION AS AT 30TH
B
SEPTEMBER, 2017

NOTE N'000
Property, plants & equipment (W13) 1 ,760,000
Goodwill (W8) 144,000
Current asset (W9) 684.800
2.588.800
Equity shares (W14) 371,200
Share premium (W10) 256,000
Retained earnings (W11) 1.142.400
1,769,600
Non-controlling interest Non- (W12) 195,200
current liabilities:
10% loan notes (W15) 224,000
Current liabilities (W16) 400.000
2.588.800
WORKINGS
W8 Goodwill N'000 N'000
Investment at Cost
Shares 2/3 (128,000 x 60%) x N6 307,200
Less: Equity Share of Lowerlower Ltd.
(128,000 x 60%) (76,800)
Pre-acquisition Profit (160,000 x 60%) (96,000)
Fair Value Adjustment (64,000 x 60%) (38,400) (211,200)
Parent Goodwill 96,000
Non-Controlling Interest Goodwill (per question) 48.000
Total Goodwill 144.000
W8a Pre-acquisition Profit N'000 N'000
Profit at 30 September, 2016 208,000
Earned in post acquisition period (96,000 x 6/12) (48,000)
160.000
W9 Current Assets N'000 N'000
Higher-Higher Ltd. 512,000
Lower-Lower Ltd. 211,200
URP on Inventory (25,600)
Cash in Transit Inter 6,400
Group Balance (19.200)
684.800

W10 Share Premium N'000 N'000


N51,200 shares = (128,000 x 60% x 2/3) issued by
higher Ltd would be recorded as share capital of
N51,200 and share premium of N256,000 (51,200 x
5)

W11 Retained Earnings N'000 N'000


Per question Higher-Higher Ltd 1,132,800
Add: Lower-Lower Ltd post-acquisition Profit
(96,000 x 6/12) - (25,600URP + 6400Depr) x 60%

9.600
1.142.400
N'000
W12 Non-Controlling Interest 336.0
Net Assets per Statement of Financial Position (25,600
URP on Inventory )
Net Fair Value Adjustment (64,000 - 6,400) 57,600
Non-Controlling Interest - 368,000 x 40% 368.000
Share of Goodwill per acquisition 147.200 4
8.000
195.200

W13 Property. plant and equipment N'000


Parent 1,299,200
Subsidiary Fan 403.200
value adj. 64.0 (
Depreciation adj. 6,400)
W14 Equity shares 1.760.000
Parent 320.0 5
Share exchange 1,200
371.200
96.000
128.0 2
W15 10% loan notes 24.000
Parent 262,400
Subsidiary

W16 Current liabilities


Parent
Subsidiary 150,400
Cash in transit (12.800
)
400.000

EXAMINER'S REPORT
The question tests candidates' knowledge of group accounts. Candidates are required to
prepare for a simple group consolidated statement of profit or loss and other comprehensive
income and consolidated statement of financial position.
All the candidates attempted the question and the performance was average as about 55% of
the candidates obtained up to 50% of the marks allocated to the question.
Candidate's commonest pitfall was their inability to correctly calculate figures for goodwill,
non-controlling interest and the consolidated retained earnings.
Candidates are advised to pay special attention to preparation of group financial statements
while preparing for examination at the skills level.
MARKING GUIDE MARKS MARKS
a. Consolidated statement of Profit or Loss

Determination of revenue Determination of 1


cost of sales Determination of distribution 31
cost Determination of administrative expenses /
Determination of finance cost Determination 1
of profit before tax Determination of income 1
tax expense Determination of profit for the 1
year Profit for the year attributable to parent
Profit for the year attributable to NCI 1

10
b. Consolidated Statement of Financial Position
Determination of PPE Determination of
goodwill Calculation of current assets
Determination of equity Determination of share
premium Determination of retained earnings
Calculation of non-controlling interests 1
Determination of non-current liabilities 1
Determination of current liabilities Calculation
of pre-acquisition reserves
1
- T '3
otal assets
- T 20
otal liabilities 30
Total
SOLUTION 2
(a) QUADRITOPNIGERIAPLC
STATEMENT OF PROFIT OR LOSS AND OTHER COMPREHENSIVE INCOME FOR
THE
YEAR ENDED SEPT. 30, 2017

Notes N'000
Revenue 185,000
Cost of sales (W1) (128.575)
Gross profit 56,425
Distribution cost (W2) (17,775)
Administration expenses (W3) (21,775)
Finance cost (W4) (3,000)
Profit before tax 13,875
Income tax 30% (4,162.5)
Profit for the year 9,712.5
Revaluation (loss)/reversal (6.250)
Total comprehensive income 3.462.5
(b) STATEMENT OF FINANCIAL POSITION
AS AT SEPTEMBER 30, 2017
Non-Current Assets: N000 N000
Property, plant and equipment (W8) 293,37
Current Assets: 5
Inventory 19,500
Trade receivables 51,500
Prepayments 2,250

73.250 3
66.625
175,000
Equity and Liabilities: 50.000
Ordinary shares of 50k each 31.250 4
Share premium Revaluation 1,962.5
reserves 298,212.5
Retained earnings (W7) 25.000

Non-Current Liabilities:
12% Loan notes (payable 2021)
Current Liabilities:
Trade payables 28,000
Accruals (W5) 3,250
Bank overdraft 1,000
Provision for tax 4,162.5
Interim dividend payable 7.000

43.412.
5
366.625
WORKING NOTES:
W1 Cost of sales (COS) N'000
N'000 17.375 1
Inventory at October 1, 2016 03,500
Production Cost 120,87
5
Inventory at September 30, 2017
(19,500
Depreciation:
)
Building (2% x N100,000) 2,000
101.375
Plant & Equipment (2% x N160,000) 32.000
34.000
Share of COS (80% x N34,000)

27,200
128.575

W2
Distribution cost:
Distribution cost (13,500 + 2.375 - 1,500 14,375
Depreciation (10% x 34,000) 3,400
17.775
W3
Administration Cost:
Administration expenses (18,250+875-750) 18,375
Depreciation (10% x 34,000) 3.400
21.775
W4
Finance cost = 12% x 25,000 = 3.000
W5
Accruals: N'000
Administration 2,375
Distribution 875
3.250
W6
Prepayments: N'000
Administration 1,500
Distribution 750
2.250

W7 STATEMENT OFCHANGES INEQUITY

Equity Share Rev. Res Ret. Earn. Total


Shares Premium
N'000 N'000 N'000 N'000 N'000
Bal. B/F 125,000 25,000 37,500 39,250 226,750
For the Year 50,000 25,000 (6,250) 9,712.5 78,462.5
Dividend - - - (7,000) (7,000)
175.000 50.000 31.250 41.962.5 298.212.5

W8 STATEMENT OF CHANGES IN PROPERTY, PLANT AND


EQUIPMENT
AS AT SEPTEMBER
Land 30, 2017 Plant & Equip.
Building Total
COST: N'000 N'000 N'000 N'000
Bal. B/f 131.250 100.000 160.000 391.250
(A) 131,250 100,000 160,000 391,250
DEPRECIATION:
Bal. B/f - 26,625 31,000 57,625
Dep. for the year - 2,000 32,000 34,000
Loss on revaluation 6.250 - - 6,250
(B) 6,250 28,625 63,000 97,875
Carrying amount (A-B) 125.000 71.375 97.000 293.375

EXAMINER'S REPORT
The question tests candidates' understanding and knowledge of the preparation of published
financial statements of an entity. The question requires candidates to prepare a statement of
profit or loss and other comprehensive income and statement of financial position in
accordance with IAS 1.
Over 90% of the candidates attempted the question and the performance was below average
since only about 40% of the candidates were able to obtain up to fifty percent of the marks
allocated to the question.
Candidates' commonest pitfall was their inability to present the required financial statements
in a manner required for publication as provided in IAS 1. Also, some candidates did not
show some relevant workings and notes to the financial statements.
Candidates are advised to prepare adequately for their examination using relevant materials
like the pathfinder and ICAN Study Text.

MARKING GUIDE MARKS MARKS


A Statement of profit or loss
- Determination of gross profit 3V2
- Determination of distribution cost 1V2
- Determination of administrative expenses 1V2
- Determination of finance cost V2
- Determination of profit before tax and for the year V2
- Treatment of revaluation loss reversal V4
- Total Comprehensive income
8
b. Statement of financial position
- Determination of PPE 4
- Determination of current assets 1
- Determination of equity 3V4
- Determination of non-current liabilities V4
- Determination of current liabilities 3
- Total assets V4
Total liabilities
12
Total 20

SOLUTION 3
a.
Classes of Financial Ratios
Profitability and Efficiency Main Aims Examples
- It is aimed at - Return on capital
determining the employed
performance of an - Return on total
organization assets
- Concerned with - Gross profit margin
relative efficiency in - Net profit margin
the utilization of - Capital employed
company's assets turnover
- Total asset turnover
- Return on
Long term solvency and - To determine a shareholders' equity
stability company's ability to - Fixed interest cover
meet its long term - Fixed dividend
obligation cover
- To show the degree - Total debt to equity
- Gearing ratio
of safety of a business Debt to assets ratio
from failure in the
long term To
Short term solvency and determine the ability Current ratio Acid
Liquidity of a firm to meet its test/quick ratio
short term financial Receivable turnover
obligations. Receivable collection
period
Trade payable turnover
Trade payable payment
period
Earnings per share
Dividend per share
Shareholder's investment To determine the
Price earnings ratio
ratios value of a company,
Earnings yield
consequently;
Dividend yield
To enable for
Dividend payout ratio
comparism between
investment alternative
before making
decision.

b.i.
Cash Operating Cycle
The cash operating cycle is also called the working capital cycle, it is the average time of one
cycle of business operations from the time that suppliers are paid for the resources they
supply to the time that cash is received from customers for the goods (or services) that the
entity makes (or provides) with those resources and then sells.

It can also be explained as the length of time between a firm's purchase of inventory and the
receipt of cash receivable. It is the time required for a business to turn purchases into cash
receipts from customers.
NWOKEKE NIGERIA PLC CASH
OPERATING CYCLE FOR THE
PERIOD MARCH 31, 2017

DAYS DAYS
Average Inventory Holding Period
- Raw materials (wi) 73
- Work-in-progress (wii) 29.2
- Finished goods (wiii) 97.3 1
Average receivable collection period (wiv) 99.5 93.3
2
Average payable period (wv) 9
Working capital cycle 2
.
Working Notes: 8
(wi) Number of days in raw materials
= Average RM inventory x 365 days (
8
Cost of sales 7
6
= 142.500.000 x 365 days
)
712,500,000
= 73 days
2
(wii) Number of days in work-in-progress
0
= Average WIP inventory x 365 days Cost
5
of Sales
.
= 57,000,000 x 365 days
2
712.500.0
= 29.2 days
(wiii) Number of days in finished goods
= Average FG inventory x 365 days Cost
of Sales
= 190.000.000 x 365 days
712.500.000
97.3 days
(wiv) Trade receivable collection period:
= Average trade receivable x 365 days
Revenue
= 218.500.000 x 365 days
855,000,000
93.3 days
(wv) Trade payable payment period:
= Average trade payable x 365 days
Purchases
= 114,000,000 x 365 days
475,000,000
87.6 days
b. (iii)
The cash management policy of Nwokeke Nigeria Plc as at March 31, 2017 is not
encouraging as the cash operating cycle of 205.2 days is far more than the industry average
performance of 75 days.
b. (iv)
Steps to be taken to improve Cash Operating cycle of Nwokeke Nig. Plc.
• Nwokeke Plc needs to reduce the raw materials inventory period e.g Just in time
purchases
• The company needs to speed up the production process without compromising the
quality of the goods produced by improving on the production method.
• The company may increase the period of credit taken from suppliers though the
credit period already seems very long.
• The company needs to reduce finished goods inventory holding period e.g through
JIT Production.
• Reducing receivable collection period by giving discount, prompt invoicing, and
regular follow up.

EXAMINERS REPORT
This question is in two parts. Part (a) test candidate‟s knowledge of the different categories
of financial ratios as they were required to state the main aim of each category with relevant
examples. Part (b) test candidates understanding of the cash operating cycle of an entity.
Over 80% of the candidates attempted the question and the performance was below average
as only about 30% of the candidates got up to 50% of the marks allocated to the question.
Candidates‟ commonest pitfall was their wrong calculation of the cash operating cycle and
their poor assessment of the company‟s cash management performance.
Candidates are advised to study hard while preparing for the examination of the Institute.
MARKING GUIDE MARKS MARKS
a. - Correct explanation of the main aim of each of the 4

four categories of the relevant financial ratios at 1


mark each
Stating two correct examples each of the four
categories of the relevant financial ratios at 1/2 mark
4
each
Explanation of cash operating cycle
B Correct Calculation of operating cycle: 2
i Average inventory holding period
i Average receivable collection period 11
i Average payable period Calculation of /
Operating Cycle V
;
4
V
Correct assessment of the cash management ;
iii 11
Performance of Nwokeke Nigeria Plc Stating two
/ 2
correct steps of improving the cash operating
cycle of the company Total
lo
SOLUTION 4

i. An associate is an entity over which the investor has significant influence. Significant
influence is the power to participate in the financial and operating policy decisions of
the investee, without having control or joint control over those policies.
Significant Influence is normally assumed to exist if the investors own between 20%
and 50% of the investee's ordinary shares excluding joint control.
ii. Under the equity method of accounting the investment made in the associate
is recorded initially at cost. In each subsequent year, the investor's share of the
associate's profit is added to the carrying amount of the investment and is also
recognized as income in the investor's financial statements. Dividends received from
the associate are subtracted from carrying amount of the investment. Only the
investor's share of the associate's profit is shown in the investor's statement of
comprehensive Income and only the investor's
share of the associate net assets is shown in the investor's statement of
financial position. These are items that are shown as a single line items and there is
no need to account for non-controlling interest.
This differs from the acquisition method used for subsidiaries, whereby all of a
subsidiary's assets, liabilities, income and expenses are incorporated on line by line
basis into the consolidated financial statements and then the non-controlling interest
(if any) is accounted for. Therefore, in the statement of financial position of the
reporting entity (the investor) an investment in associate is measured at carrying
amount, thus:
Carrying amount N
Cost of investment X
Parent's share of post acquisition profits/(losses) of the X
associate (or JV)
Impairment of the investment recognised (X)
X
iii. A joint operation is a joint arrangement whereby the parties that have joint control of
the arrangement have rights to the assets, and obligations for the liabilities, relating to
the arrangement. Those parties are called joint operators. While a joint venture is a
joint arrangement where the parties that have joint control of the arrangement have
rights to the net assets of the arrangement. These parties are called joint venturers.
b. Y Limited Statement ofComprehensive, Income
N'000 N'000
Operating profit 325
Share of associate profit 40% x N55 22
Unrealised profit (15-10) x 40% x y4 (0.5) 21.5
Profit before taxation 346.5
Income tax exp. (N85 + 40% of N15) (91)
255.5

Statement of Changes in Equity N'000

Balance b/f 355


Associate balance b/f: (N330- N50)x40% 112

Profit for the year 255.5


722.50

Statement of Financial Position as at October 30, 2017 Assets N'000


Non-current assets (PPE) 800
Investment in associate (W1) 204
1,004
Current assets (W2) 379.5
1.383.5

Equity and liabilities


Ordinary share capital 500
Retained earnings (W3) 722.5
1222.5
Current liabilities (155+ Associate Tax 161
40% of N15)
1.383.5
Workings
W.1 Investment in associates N'000
Initial cost 70
Share of post-acquisition (40% of 330-50) 112
Share of profit for the year (40% of N55) 22
204

W.2 Current assets N'000


Parent 390
Adjustment - for dividend receivable (10)
- Unrealised profit (40% of N5000) x y4 (0.5)
379.5
W.3 Retained earnings N'000
Parent 605
Share of post-acquisition 112
Share of profit for the year 22
Adjustment - dividend receivable (10)
- unrealised profit (0.5)
Associate tax (40% of N15) (6)
N722.5

EXAMINER'S REPORT
This question tests candidates' knowledge of IAS 28. In part (a) candidates were required to
explain the terms 'Significant influence' and equity method of accounting. They were also
required to distinguish between joint operation and joint ventures. Part (b) required
candidates to prepare separate financial statements using the equity method of accounting.
About 25% of the candidates attempted the question and the performance was very poor.
Candidate's commonest pitfall was their failure to correctly explain the relevant terms and
wrong calculation of the carrying amount of the investment in associate.
Candidates are advised to emphasise application of relevant accounting standards at this level
of the Institute's examinations rather than mere reading through the provisions of the various
standards.

MARKING GUIDE MARKS MARKS


Ai - Correct explanation of 'associate' and 'significant
influence' 2
ii - Correct explanation of equity method of accounting 2
iii - Distinguishing between joint operation and joint
ventures 2

b - Presentation of separate financial statements:


Statement of comprehensive income
1
- stating the operating profit /3
- stating shares of associate's profit 1
- calculating profit before tax and for the year n1
Statement of changes in equity
- Stating opening balance of retained earning V3
1
- Stating shares of Associates opening post /3
Acquisition J/3
- Stating group profit for the year 1V3
Statement of financial position
- Stating the non-current assets (PPE) V3
- Determination of investment in associate 2V3
- Determination of current assets 2V3
- Stating the equity capital V3
- Determination of retained earnings 3
- Determination of current liabilities 1
- Total assets V3
- Total liabilities and equity V3
10
Total 20

SOLUTION 5
a. NEEDS FOR APPLICATION OF IAS 36 - IMPAIRMENT OF ASSETS
IAS 16 has long required that property, plant and equipment should not be carried in
the financial statements at more than its recoverable amount. Recoverable amount is
defined as the higher of the amount for which it could be sold and the amount
recoverable from its future use. However, there was very little guidance as to how and
under what circumstances the recoverable amount should be identified or measured.
IAS 36 gives such guidance.
Recoverable Amount
IAS 36 defines recoverable amount as the higher of fair value less costs of disposal
and value in use. Fair value less costs of disposal is the amount at which an asset could
be disposed off, less any direct selling costs. Value in use is the present value of the
future cash flows obtainable as a result of an asset's continued use, including those
resulting from its ultimate disposal.
The definition takes into account management's ability to choose whether to sell or
keep the asset when provided with the information about fair value less costs of
disposal and value in use. The decision is based on the cash flows that can be
generated by following each course of action. An entity will not continue to use the
asset if it can realise more cash by selling it and vice
versa. This means that when an asset is stated at the higher-of net realisable value or
value in use it is recorded at its greatest value to the entity.
bi. A related party transaction is a transfer of resources, services or obligations between
parties that are connected or related, regardless of whether or not a price is charged.
Examples of such transactions include:
• Purchase or sale of goods;
• Purchase or sale of property and other assets;
• Rendering or receiving of services;
• Leases;
• Transfer of research and development;
• Transfers under license agreements;
• Transfers under finance arrangements (including loans and equity contributions in
cash or in kind);
• Provision of guarantees or collateral;
• Commitments to do something if a particular event occurs or does not
occur in the future, including executory contracts(recognized and
unrecognized); and
• Settlement of liabilities on behalf of the entity or by the entity on behalf of another
party
bii. Under IAS 24- Related Party Disclosures, an entity is related to a reporting entity if any
of the following conditions applies:
• Both entities are members of the same group thus a parent and a subsidiary and
fellow subsidiary are related parties;
• One entity is an associate or joint venturer of the other entity (or an associate or
joint venture of a member of a group of which the other entity is a member);
• Both entities are joint venturers of the same third party;
• One entity is a joint venturer of a third entity, and the other entity is an associate
of the third entity;
• The entity is a post-employment defined benefit plan for the benefit of employees
of either the reporting entity or an entity related to the reporting entity. If the
reporting entity is in such a plan, the sponsoring employers are also related to the
reporting entity
• The entity is controlled, or jointly controlled, by a person or a close member of
that person's family who, has control, or joint control, over the reporting entity
• Has significant influence over the reporting entity or is a member of the key
management personnel of the reporting entity or its parent
EXAMINER'S REPORT
This question tests candidates' Knowledge and understanding of IAS 36 and IAS 24. Part(a)
required candidates to explain the need for the application of IAS 36 and also to explain the
concept of 'recoverable amount' Part(b) required candidates to define with examples 'related
party transactions' and to state conditions under which entities are said to be related.
About 80% of the candidates attempted the question and the performance was poor as over
60% of the candidates who attempted the question obtained about one third of the marks
allocated to the question.
Candidates commonest pitfall was their inability to correctly define or explain the relevant
concepts.
Candidates are advised to study all the relevant standards examinable at this level of the
Institute's examination.

MARKING GUIDE MARKS


a. - Explanation of the need for the application of IAS 36 3
- Explanation of the concept of recoverable amount 4
b.i - Definition of related party transaction 2
- Two examples of related party transaction at one
mark each 2
li - Four conditions leading to related party at 1 mark
each 4
Total 15

SOLUTION 6
a. i. A change in the useful life of plant and equipment is a change in accounting
estimate and is applied prospectively.
Therefore, the carrying amount of the plant and equipment is written-off- over four
(4) years rather than seven (7) years. All the effects of the change are included in
profit or loss. The nature and the amount charged should be disclosed.
ii. The items should be expensed in profit or loss. However, given the nature and size, it
may be disclosed separately.
iii. The contribution should be expensed in profit or loss. It should be disclosed
separately, if it is material.
b. i The adjustment for the error in opening inventory should be made
retrospectively. The inventory at beginning will be credited with the
overstated amount of N3,135,000 while the retained earnings at the end of last
period, being the opening retained earnings for the period, will be debited with same
amount. This would be done in the statement of changes in equity.
ii. The nature of prior period error
The amount of the correction at the beginning of the earliest prior period in the
statement.
If the retrospective restatement is not practicable for a prior period, an
explanation of how and when the error has been corrected.
The amount of the correction for each financial statement item.
iii. Fi
nal figures of Inventory in the Statement of Financial Position as at March,

31, 2017

N‟000 N‟000
Balance per information Reduction 31,42
to net realizable value original cost 5
Net realizable value (78 - 4.5) 120
Closing inventory balance per SFP (73.5) (46.5)
312.378.5

EXAMINER‟S REPORT
The question tests candidates‟ knowledge and application of the provisions of IAS 8
- changes in Accounting policies, estimates and corrections of errors.
Part 'a‟ of the question required candidates to show the disclosure requirements in respect of
some specific transactions in the Financial Statements, while part 'b‟ tests corrections of
errors in accordance with the requirements of IAS 8,
About 70% of the candidates attempted the question and performance was below average.
Candidates‟ commonest pitfalls include the following:
• Inability to differentiate between changes in accounting policies and changes in
accounting estimates
• Failure to determine when to make prospective or retrospective adjustments when
correcting errors.
• Poor disclosure requirements in the financial statements after making necessary
corrections.
Candidates are advised to pay more attention to the applications of the provisions of all
relevant Accounting Standards at this level of the Institute‟s examinations for better
performance in future.
MARKING GUIDE MARKS
a. Explanation of the treatment of transactions in the
financial statement
i Correct explanation of the treatment of change in 2
useful life of plant and equipment
ii Correct explanation of the treatment of bonuses paid
to employees 2
iii Correct explanation of the treatment of N14million
contribution to a foundation 2
b. i Explanation of the adjustment needed to correct error 2
of overstatement of opening inventory
ii Stating two disclosure requirements of IAS 8 in the
financial statements for correcting of errors 3
iii Presentation of inventory in the statement of financial
position. 4
Total 15

SOLUTION 7
a. Interest rate implicit
The interest rate implicit in the lease is the discount rate that causes the aggregate present
value of the minimum lease payments and the unguaranteed residual value to be equal to
the sum of the fair value of the leased asset and any initial direct costs of the lessor at the
inception of the lease. It is the Internal Rate of Return (1RR) of the cash flows from the
lessor's viewpoint.
b. Calculation of Interest Implicit in the lease

„PV = Annuity x Cumulative Discount Factor CDF)


2,500,000 = 788,640 x CDF
CDF = 2,500,000
788,640
= 3.170
= 10%
OR
Year Cashflow DCF at 6% PV DCF at 10% PV

0 2,500,000 1.00 2,500,000 1.00 2,500,000


1-4 788,640 3,465 2.732.638 3.170 2.499.989
NPV 232.638 (11)
IRR=LR+=-------------------------------------------NPVlr------------------x (HR - LR)
. NPV lr -NPV hr V J

= 6% + 232638 x (10% - 6%)


232638 - (-11)
= 6%+ 1 x (4%)
= 6% + 4%
IRR = 10%
Where:
IRR = Internal rate of return LR =
Lower rate
NPV = Net present value
HR = Higher-rate
c. Manilla Nigeria Plc should treat the equipment as a Finance Lease. Reasons are:
i. The lease term is for the useful life of the asset.
ii. The present value of the minimum lease payment is considered the fair value
of the asset.
d. Wrong classification of finance lease as operating lease will result in the following
effects:
- The financial statements will not fairly present the financial position of the entity.
- The leased asset is not recognized in the statement of financial position, even
though the substance of the lease is that the entity owns it.
- The liability for the lease payment is not recognized in the statement of
financial position.
- The lease will become an off balance sheet finance.
- Both assets and liabilities are understated.
- Finance charge in the profit or loss is either over or understated.
EXAMINER'S REPORT
The question tests candidates knowledge of Lease - (IAS 17) and they were required to
briefly explain the term 'Interest rate Implicit in Lease', calculate the appropriate interest rate
implicit in the lease transactions from a given data, determine the correct type of lease based
on the data available and to state the implications of incorrectly treating lease transactions in
the financial statements.'
Only 25% of the candidates attempted the question and performance was poor as less than
10% of the candidates that attempted the question scored more than 50% of the marks
allocated to the question.
The candidates' commonest pitfalls are as follows:
• Inability to correctly explain interest rate implicit in lease transactions
• Wrong calculation of implicit interest rate and inability to explain the effects of wrong
classification of lease in the Income statement and the statement of financial position.
Candidates are advised to pay special attention on application of relevant accounting standard
at this level of Institute's examination rather than mere reading through the provisions of
various standards.
MARKING GUIDE MARKS
a. Explanation of 'interest rate implicit in the leasing of
an equipment 2
b. Calculation of interest rate implicit in the lease
transaction 4
c. Stating correct treatment of the lease 1
Giving any two correct reasons for the treatment 2
d. Discussing the effect of classifying a lease
incorrectly in financial statements giving any six points at 1
mark
each 6
Total 15
THEINSTITUTEOFCHARTEREDACCOUNTANTSOFNIGERIASKILLSLEVELEXAMINATION-NOVEMBER2018FINANCIALREPORTING
Time Allowed: 3% hours (including 15 minutes reading time)
INSTRUCTION: YOU ARE REQUIRED TO ANSWER FIVE OUT OF SEVEN
QUESTIONS IN THIS PAPER
SECTION A: COMPULSORY QUESTION (30 MARKS)
QUESTION 1
The financial statements of OSHODI Nigeria Limited for the year ended May 13 2017 are as
follows: Statement of Profit or Loss for the year ended May, 31 2017

N‟m
Revenue 3,820
Cost of sales (2,620)
Gross profit 1,200
Operating expenses (300)
900
Interest (30)
Profit before tax 870
Taxation expense (270)
Net profit for the year 600
Statement of Financial Position as at May 31 2017 (including comparative figures)

2017 2016
Non-Current Assets N‟m N‟m
Property, plant and equipment 1,890 1,830
Intangible assets 650 300
2,540 2,130

Current Assets
Inventory 1,420 940
Account receivables 990 680
Cash 70 nil
2,480 1,620
Total Assets 5,020 3,750
Equity and Liabilities
Ordinary shares of N1 750 500
each Reserves: 300 100
Share premium 190 Nil
Revaluation reserve 1,610 1,400
Retained earnings
Equity 2,8502,00
Non-current liabilities 8700
Current liabilities Total 1,300540
equity and liabilities 5.0201 ,2
1
03.7
50
OSHODI NIGERIA LIMITED Statement of Changes in Equity for the
year ended May 31 2017
Share Share Revaluation Retained Total
Capital Premium Reserve Earnings N‟m
N‟m N‟m N‟m N‟m 2,000
June 1 2016 Profit 500 100 Nil 1,400 600
for the year 600
Transactions
within Equity:
Dividend Bonus
(400) (400)
issue Issue of
50 (50) 450
shares Revaluation
200 250 200
reserve Transfer to 200
retained earnings (10)
May 31, 2017 190 10
750 300 1.610 2.850

Notes to the financial statements:


(1) Cost of sales includes depreciation of property, plant and equipment of N320 million and a
loss on the sale of plant of N50 million. It also includes a credit for the amortisation of
government grants. Operating expenses include a charge of N20 million for the amortisation
of goodwill.
(2) Intangible assets: 201 201
Deferred development 7 6
expenditure Goodwill N‟m N‟m
470 100
180 200
650 300
(3) Non-current Liabilities: 201 201
10% loan notes 7 6
Government grants N‟m N‟m
Deferred tax 300 100
4) Current Liabilities: 260 300
Account payables Bank 310 140
overdraft Accrued loan 870 540
875 730
interest Declared Nil 115
dividends unpaid 15 5
Taxation 280 200
130 160
1,300 1,210
The following additional information is relevant:
(i) Intangible assets:
The company successfully completed the development of a new product during the current
year, capitalising a further N500 million before amortisation charges for the period.
(ii) Property, plant and equipment/revaluation reserve:
The company revalued its buildings by N200 million on June 1 2016. The surplus was
credited to revaluation reserve.

• New plant was acquired during the year at a cost of N250 million and a government grant
of N50 million was received for the plant.
• On June 1, 2016 a bonus issue of 1 new share for every 10 held was made from the share
premium.
• N10 million has been transferred from the revaluation reserve to realised profits as a year-
end adjustment in respect of the additional depreciation created by the revaluation.
• The remaining movement on property, plant and equipment was due to the disposal of
obsolete plant.
Share issue:
In addition to the bonus issue referred to above, Oshodi Nigeria Limited made a further issue
(iii) of ordinary shares for cash.
Required:
a. Prepare statement of cash flows for Oshodi Nigeria Limited for the year ended May 31, 2017 in
accordance with IAS 7 „Statements of Cash Flows‟ using the indirect method.
(25 Marks)
b. Compare the direct and indirect methods of preparing statement of cash flows.
(5 Marks)
(Total 30 Marks)

SECTION B: YOU ARE REQUIRED TO ANSWER TWO OUT OF THREE (40


QUESTIONS IN THIS SECTION MARKS)

QUESTION 2
a. IAS 33 on earnings per share defined potential ordinary share as a financial instrument or
other contract that may entitle its holder to ordinary shares at some time in the future. State
the three examples of potential ordinary shares according to IAS 33.
(6 Marks)
b. Anambra Limited acquired 80% of Omambala Limited‟s ordinary shares for N210 million on
January 1, 2013. On the acquisition date, the retained earnings of Omambala Limited were
N105 million. The fair value of non-controlling interest in Omambala Limited at the date of
acquisition was N56 million. The financial statements of the two companies for the year
ended December, 31 2017 are stated below:

Anambra Omambala
Limited Limited
N‟000 N‟000
ASSETS
Non-Current Assets
Property, plant and equipment 210,000 157,500
Investments 280,000
Current Assets
Inventories 56.000 52.500
Trade and other 42.0 98,000
receivables Cash and cash 7,000 17.500 3
equivalents Total Assets 595.00 25,500
0
EQUITY AND LIABILITIES
Equity
Share capital 56,000 42,000
Share premium 14,000 7,000
Retained earnings 206,500 175,000
Non-Current Liabilities
Loans notes 210,000 59,500
Current Liabilities
Trade payables 108,500 42,000
Total equity and liabilities 595,000 325,500

The following additional information is relevant:


(i) Anambra Limited sold goods to Omambala Limited for N35 million with a gross profit margin
of 25%. Included in the inventories of Omambala Limited is 40% of the goods as at
December 31, 2017.
(ii) The fair values of net assets of Omambala are equal to their carrying amounts at acquisition
date, this does not include land. The company has land which was included in the accounts at
its cost amounting to N105 million and has a value of N126 million.
Required:
Prepare consolidated statement of financial position for the Anambra Limited group as at December
31, 2017. (14 Marks)
(Total 20 Marks)

QUESTION 3
Yemnike Nigeria Limited has an accounting profit before taxation of N225 million for the year
ended December 31, 2017.
The following are extracts of the financial position of Yemnike Nigeria Limited as at December 31,
2017.

NON-CURRENT ASSETS
N‟000
Building 157,500
Plant and machinery 250,000
Assets held under finance lease 200,000
RECEIVABLES
Trade receivables 182,500
Interest receivable 2,500
PAYABLES
Fines 25,000
Finance lease obligation 216,000
Interest payable 8,250
The following information is relevant:
(i) The building was acquired by the company at the cost of N175million at the start of the year
and it is the policy of the company to depreciate building at 10% p.a. on straight line basis.
The company tax consultants have stated that the company can claim N105million capital
allowance this year on the building.
(ii) The balance in respect of plant and machinery is after providing for depreciation of
N30million and the capital allowance claimable on it is N25million.
(iii) The asset held under finance lease was acquired during the year. The relevant tax law does not
distinguish between finance lease and operating lease. Rental expense for lease is tax
deductible. The annual lease rental is N72million and was paid on December 31, 2017. The
depreciation policy for leased assets is 20% p.a. on straight line while annual finance charge
amounted to N36,667million.
(iv) The receivables figure is shown net of an allowance for doubtful balances of N17.5million.
This is the first year that such an allowance has been recognised. A deduction for debt is only
allowed for tax purposes when the debtor enters liquidation.
(v) Interest income is taxed and interest expense is allowable both on cash bases. There were no
opening balances on interest receivable and interest payable.
(vi) Provision for fines and penalties are not allowable deductions for tax purposes. The fines
payable is a provision made during the year.

Required:
a. Calculate current tax expense for the period. (7
b. Calculate the deferred tax liability as at December 31, 2017 Marks)
c. Prepare notes showing the component of tax expense for the year. (12
Marks)
N.B: Income tax rate is 30%. (Total 20 Marks)
(1
Mark)
QUESTION 4

The following financial statements were extracted from the books of Adebayo Trading Company Plc
for the relevant years.

Statement of Profit or Loss and Other Comprehensive Income for the year ended March 31:
2018 2017
N‟000 N‟000
Revenue 250,000 400,000
Cost of sales (137,500) (225,000)
Gross profit 112,500 175,000
Administrative expenses (36,050) (44,500)
Distribution expenses (20,200) (24,250)
Finance cost (3,125) (3,125)
Profit before tax 53,125 103,125
Taxation expense (20,000) (40,000)
Profit for the year 33,125 63,125

Statement of Financial Position as at March 31:


2018 2017
Assets N‟000 N‟000
Non-current assets at 136,500 196,000
cost Accumulated (36,500) (52,250)
depreciation 100,0 143,75
Current assets 0
Inventory 79,250 20,750
Trade 50.0 1
receivables 2,500
Bank balance 12.0 9
Total Assets 1,750
141.250 1
Equity and Liabilities: 25,000
Equity 241.250 2
Ordinary shares of 50 kobo each 57,500
68.750 57,500
Retained earnings 43,000 25,000
Non-current liabilities: 100,500 82,500

10% Loan notes 31,250 31,250


12% Redeemable preference shares - 5,000
31,250 36,250
Current Liabilities:
Trade payables 18,750 26,875
Taxation 60,000 40,000
Bank overdraft 30,750 83,125
109,500 150,000
Total equity and liabilities: 241,250 268,750
Additional Information:
(i) Dividend paid to Equity holders are N15,125,000 for the year ended, March 31, 2018 and
N21,375,000 in 2017.
(ii) There was a drop in the market price per share of the company‟s equity shares from 36 kobo
in the year ended March 31, 2017 to 24 kobo in 2018.
(iii) The finance cost relates to the interest paid on the 10% loan notes.
Required:
a. Calculate in columnar form, for the two relevant years the following financialratios:
Return on capital employed Net
profit margin (use profit after tax)
Current ratio Quick
ratio Debt ratio
Fixed interest
cover Dividend
cover
Dividend yield (12 Marks)
b. Comment on the profitability and short term liquidity of the company based on the ratios
calculated. (4 Marks)
c. Management may use various forms of Creative Accounting to manipulate the view given by
the financial statements. Identify and explain four creative accounting techniques.
(4 Marks)
(Total 20 Marks)
SECTION C: YOU ARE REQUIRED TO ANSWER TWO OUT OF THREE
QUESTIONS IN THIS SECTION (30 MARKS)

QUESTION 5
Akawo Limited is a building contracting firm based in Abuja. ABC Limited awarded a contract to
Akawo Limited to construct a residential building in Lagos. The agreed contract price is N80 million
and the completion date is December 31, 2017.
The following are details of transactions on the contract to March 31, 2016.
The contract commenced July, 1 2015.

Contract costs: N’000


Architects and surveyor‟s fees 1,000
Materials 6,200
Direct labour costs 7,000
Overhead is 40% of direct labour costs 29,600
Estimated cost to completions (Excluding depreciation)
Plant and machinery used exclusively on the Contract 7,200

The value of the plant at the end of the contract would be N1.2m and the basis of
depreciation is period of usage. Material on site as at March 31, 2016 is N600,000.
Progress payment made by ABC Limited to Akawo Limited amounted to N25.6m as at
March 31, 2016.
The following information is also relevant to the contract as at March, 31 2017:
Cost incurred since the commencement of the contract to date-N40.8m.
Estimated cost to completion (excluding depreciation)N13.2m
ABC Limited paid additional N32.4m to Akawo Limited on March, 31 2017 Akawo Limited
uses percentage of completion to determine profit on a contract.
Required:
Prepare in relation to the building contract, the statement of profit or loss extracts for the years ended
March 31, 2016 and 2017 and the statement of financial position extracts as at the year ended on
those dates. (Total 15 Marks)
QUESTION 6
Intangibles assets by their nature do not exist physically under IAS 38 Intangible assets. The
following information on initial cost of intangibles asset were extracted from the Notes to the
financial statements of Igbo-hood Limited, a film production company on January 1, 2017:
^'000
Books and literary works 800
Quick books and SAGE 950
Patents 1,200
Video and motion picture films 2,500
Franchise 3,200
Pictures and photographs 3,400
Order or production backlog 4,000
Plays 4,200
Customers‟ contract 4,400
Trade marks for customers 4,600
Broadcasting rights 5,000
Internet 5,400
Trade secrets 4,800
Additional Information:

(i) Date of Acquisition Assessed Useful life


Quick books and SAGE January 1, 2016 5
Trade marks January 1, 2015 8
Plays January 1, 2014 7
Franchise January 1, 2013 8
(ii) Intangible assets are to be amortised on a straight line basis.
Required:
a. Calculate the costs of the following intangible assets:
i. Market based
ii. Customer related
i. Artistic related
iv. Contract based
v. Technology based
b. Calculate the carrying amounts of the following intangibles assets as at December 31, 2017
i. Quick books and SAGE
ii. Trade marks
iii. Plays
iv. Franchise (6 Marks)
c. Identify four internally generated intangible assets that are prohibited in IAS 38 (Intangibles
Assets.) (4 Marks)
(Total 15 Marks)

QUESTION 7
Financial statements identify financial position, performance and changes in cash flows over a period
of time. The main statements include statement of financial position, statement of comprehensive
income and statement of cash flows. These statements are intended to show how well a company has
performed and give indication of the value of the business. However, many accountants are of the
opinion that the financial statements are limited in value to the users.
Required:
a. Identify and discuss the limitations of financial statements (8 Marks)
b. Identify potential users of financial statements and their likely Information needs.
(5 Marks)
c. State the underlying assumptions of financial statements as enunciated by “The Conceptual
Framework for Financial Reporting”. (2 Marks)
(Total 15 Marks)
SOLUTION 1
(a)
OSHODI NIGERIA LIMITED STATEMENT OF CASH FLOW FOR THE YEAR ENDED MAY
31, 2017
Operating activities: Workings N‟m N‟m
Profit before tax 870
Add:
Finance cost 30
Depreciation 320
Loss on sale of plant 50
Amortization of goodwill 20
Amortization of other intangible assets 1 130 550
Less:
Government grant 2 (90)
Movement in working capital:
Account payables 145
Inventory (480)
Account receivables (310)(645)
Cash inflow from operations 685
Finance cost 3 (20)
Tax paid 4 (130)
Net Cash inflow from operating activities 535
Investing activities:
Development of intangible assets (500)
Purchase of plant (250)
Disposal of PPE 6 20
Government grant 50
Net Cash outflow from investing activities (680)
Financing activities
10% loan notes 200
Dividend paid 7 (320)
Ordinary shares 200
Share premium 250
Net Cash inflow from financing activities 330
Net cash inflow for the period 185
Cash and cash equivalent at the beginning of year 8 (115)
Cash and cash equivalent at the year end 70
Workings
1. Intangible assets N‟m
Opening balance 300
New development cost 500
Amortisation - goodwill (20)
Closing balance (650)
Amortisation - other intangible assets 130
2. Government grant N‟m
Opening balance 300
Addition for the period 50
Closing balance (260)
Charge to profit or loss 90
3. Finance cost N‟m
Opening balance 5
Charge to profit or loss 30
Closing balance (15)
Cash paid (20)

4. Tax paid N‟m N‟m


Opening balance:
Current tax 160
Deferred tax 140 300
Income tax for the period 270
Closing balance:
Current tax 130
Deferred tax 310 (440)
Tax paid 130

5. Disposal of PPE N‟m


Opening balance 1,830
Revaluation for the period 200
Purchase of plant 250
Depreciation (320)
Closing balance (1,890)
Disposed plant (w6) 70
6. Proceed on disposal of plant N‟m
Loss on disposal (50)
Carrying amount (w5) _70
Proceeds 20
7. Dividend paid
Opening balance 200
Charged to SOCIE 400
Closing balance (280)
Dividend paid 320

8. Cash and cash equivalent 2017


2016
N‟m N‟m
Cash 70 -
Bank overdraft - (115)
70 (115)

b.
COMPARISON BETWEEN THE DIRECT AND INDIRECT METHODS OF PREPARING
STATEMENT OF CASH FLOWS
i. The Indirect method of Cash flow uses net income as a base and non-cash expenses like
depreciation, non-cash incomes like profit on sale of scraps and net adjustments between
current assets and liabilities to produce the overall cash flow statement. The Direct method
uses only the cash transactions i.e. cash spent and cash received to produce the statement
cash flow.
ii. Indirect method - The net income is adjusted to generate the cash flows.
Direct method - Reconciliation is done to separate the cash flow from other items.
iii. Indirect method - All the factors are taken into account.
Direct method - All non-cash transactions like depreciation are ignored.
iv. Direct method-discloses information not available elsewhere in the financial statements,
whereas Indirect method involves adjusting the net profit or loss for changes in non-cash
items and movement in working capital.
v. Indirect method - it is easier to prepare though difficult to understand. Direct method - it is a
bit difficult to prepare but easier to understand.
EXAMINER‟S REPORT
The question tests candidates‟ knowledge of IAS 7 on statement of cash flows. Candidates are
required to prepare a statement of cash flow, and to compare the direct and the indirect methods of
preparing the statement of cash flows
About 98% of the candidates attempted the question and the performance was average.
Candidates‟ commonest pitfalls were their inability to derive the amount for some cash flow items
not given directly in the question, poor comparison of the direct and the indirect methods of
presenting statement of cash flows and wrong classification of cash flows.
Candidates are advised to pay more attention to details bothering on the preparation of statement of
cash flows when studying for future examination of the Institute.

Marking Guide Marks Marks


a. Preparation of statement of cash flows:
- Cashflows from operating activities 14
- Cashflows from investing activities 6
- Cashflows from Financing activities 3
- Cash and Cash equivalent 2 25
b. Comparison between the direct and indirect methods of 5
presenting statement of cash flows
Any 5 points from 3 comparison at 1 mark each
Total 30

SOLUTION 2
a. IAS 33 gives the following examples of potential ordinary shares:
i. Financial liabilities or equity instruments that are convertible into new ordinary
shares at some time in the future (convertible debentures, convertible preference
shares).
ii. Share options and warrants: Options and warrants are financial instruments that give
the holder the right (but not the obligation) to purchase new ordinary shares at
some
time in the future, at a fixed price.
iii. Shares that will be issued if certain contractual conditions are met, such as
contractual conditions relating to the purchase of a business.
b.
ANAMBRA LIMITED GROUP CONSOLIDATED STATEMENT OF
FINANCIAL POSITION AS AT DECEMBER 31 2017

Assets Note N‟000


Non-current assets
Property, plant and equipment
(N210,000,000+N157,500,000+N21,000,000) 388,500
Goodwill 3 91,000
Investments (N280,000,000 - N210,000,000) 70,000
Total non-current assets 549,500
Current assets
Inventories (N56,000,000+N52,500,000 - N3,500,000) 105,000
Trade and other receivables (N42,000,000 + N98,000,000) 140,000
Cash and cash equivalents (N7,000,000 + N17,500,000 24,500
Total current assets 269,500
Total assets 819,000

Equity and liabilities


Share capital 56.000
Share 14.0
premium 5 259,000
Retained 329,000
Non-controlling interest
earnings 6 70,000
Non-current liabilities (N210,000,000+N59,500,000) 269,500
Current liabilities
Trade payables (N108,500,000 + N42,000,000) 150,500
Total equity and liabilities 819,000

Workings
1. Group structure
Anambra 80%
Limited Non- 20%
controlling
2. Net assets of Omambala Date of Date of Post
interest
Limited Acquisition Consolidation Acquisition
N’000 N’000 N‟000
Share capital 42,000 42,000
Share premium 7,000 7,000
Retained earnings 105,000 175,000 70,000
Fair value of land 21,000 21,000
Total 175,000 245,000 70.000
3. Determination of goodwill
N‟000
Purchase consideration 210,000
Non-controlling interest (at fair value) 56,000
Net assets at acquisition (wk 2) (175,000)
Goodwill 91.000

4. Unrealised profit
N‟000
Goods sold 35,000
Unsold goods (40% x N35,000,000) 14 000
Unrealized profit (25% x N14,000,000) 3,500

5. Retained earnings of the group


Retained earnings of Anambra Limited N‟000
Share of post acquisition profit of 206,500
Omambala 56,000
(80% x N70,000,000) (3,500
Unrealized profit (wk 4) )
259,000

6. Non-controlling interests
Fair value N‟000
Share of post acquisition profit of 56.000
Omambala 14.000
(20% x N70,000,000) 70.000

EXAMINER’S REPORT
The question tests candidates‟ knowledge and understanding of IAS 33 on earnings per share and the
preparation of group financial statements. Part (a) of the question required candidates to state three
examples of potential ordinary shares stipulated in IAS 33 while Part (b) required them to prepare a
consolidated statement of financial position of a group.
Candidates showed good understanding of the question as over 90% of them attempted the question
and the performance was good.
Candidates‟ commonest pitfalls were their wrong calculations of the amounts for goodwill on
consolidation, unrealised profit on inventory and consolidated retained earnings. Also some
candidates could not give correct examples of potential ordinary shares.
Candidates are advised to endeavour to cover properly all aspects of the syllabus when preparing for
future examinations of the Institute.

Marking Guide Marks Marks


a. Examples of potential ordinary shares according to IAS 33
on EPS. Any 3 examples at 2 marks each 6
b. Consolidated statement of financial position:
- Group structure ^
- Non-current assets 2^
- Goodwill on acquisition 2
- Current assets 4
- Equity 3
- Non-controlling interests 1
- Non-current assets ^
- Current liabilities 14
Total 20

SOLUTION 3
a.
YEMNIKE NIGERIA LIMITED
TAX COMPUTATION FOR THE YEAR ENDED DECEMBER 31,
2017 N’000 N’000
Accounting profit 225,00
Add:
Disallowable expenses
Depreciation on building 17,500
Depreciation on plant & machinery 30,000
Increase in allowance for doubtful debt 17,500
Interest Payable (on accrual basis) 8,250
Fines 25,000
Depreciation of asset held under finance 40,000
Lease
Annual finance charge 36,667
174,917
Less:
Allowable deductions
Interests Income 2,500
Capital allowance on building 105,000
Capital allowance on plant & machinery 25,000
Lease rentals 72,000
(204,500)
Taxable Profit 195,417
Current Tax Expenses at 30% = 58,625,100

b. DEFERRED TAX LIABILITY AS AT DECEMBER 31, 2017

CARRYING TAX TEMPORARY


AMOUNT BASE DIFFERENCE
Building N‟000 N‟000 N‟000
Cost 175,00 175,000
Dep/capital 0 (105,000)
allowance 70,000 87,500L
(17,500
CA/TWDV )
Plant & machinery 157,50230.0 ( 280,000
Cost 0 3 (25,000)
Dep/capital 0 255,000 5,000A
allowance ,
CA/TWDV 0
Asset held under 0
Finance Lease )
Carrying amount 200,000
25
Obligation 216,000
0.
16,000
00 16,000
0 A
Trade receivables 182,500 17,500
C/A TWDV 2,500 200,000
A
Interest 25,000 2,500L
receivables Fines 8,250 Nil
Interest payable 25,000 8,250A
Temp. Diff. D/T (at 30%)
N‟000 N‟000
Deferred Tax Liabilities
(87,500 + 2,500) 90,000 27,000L

Deferred Tax Assets


(5,000 + 16,000 + 17,500 + 8,250) 46,750 14,025
Deferred tax liability as at December 31 2017 A
12975L
c.

TAX EXPENSE COMPONENT FOR THE YEAR


N‟000
Current Tax 58.625.10
Expense Deferred 12,975.00
Tax Expense 71.600.10
EXAMINER‟S REPORT

The question tests candidates‟ knowledge of tax computations involving calculation of current tax
expense and deferred tax liability.
Candidates showed very poor understanding of the requirements of the question. About 6% of the
candidates attempted the question and their performance was very poor.
Candidates commonest pitfalls were their mixing up of figures/amounts relating to current tax
expense with those for deferred tax liability in their computations, poor calculation of the carrying
amounts and tax bases of the relevant items and the inability to differentiate between allowable and
disallowable expenses.
Candidates are advised to study properly the relevant accounting standards on taxation and the
Institutes Financial Reporting study text when preparing for examinations in the future.
Marking Guide Marks Marks
a. Preparation of tax computations:
- Disallowable expenses 3/
- Allowable expenses 2/
- Calculation of current tax
expense 1 7

b. Deferred tax liability:


- Carrying amount 3/
- Tax base 3
- Temporary difference 2/
3 12
- Deferred tax liability
c. Components of tax expense:
- Current tax
/
expense
- Deferred tax
expense / 1
Total 20
SOLUTION 4
a.

RATIOS FORMULAE 2018 2017


PBIT 53,123+3,125 103,125 +
employed CE 131,750 3,123
42,69% 118,750
89.47%
OR PAT 33,125 63.125
Equity 100,500 82,50
32.96% 0
76.52%
ii. Net profit margin PAT 33,125
63.125
Revenue 250,000 400,0
13.25% 00
15.78
iii. Current ratio CA 141,250 %
125.000
CL 109,500 150.000
1.29:1 0.83:1
104,250
iv. Quick ratio 62,000 150.0

C

v.

109,500 0.70:
0.57:1 1
v. Debt Ratio Debt 5 000 +31250 ________ 312
50
Equity 57500 + 43,000 57500 +
31.09% 25,000
OR Debt Total Asset 31.250 100 43.94%
241.250 x 1 36.250 1
12.95% 00
268.250 x
vi. Fixed Interest PBIT 53125+3125 1
103125+31
13.
Covered Fixed interest 3,125 18 times 25
51
3,1
%
25 34
times
vii. Dividend Cover = PAT 33,125
63,125
Dividend Paid 15,125 2.19 21,375
times 2.95
times
viii. Dividend Yield = DPS
MPS 13k 18.6k
24k 36k
54.8% 51.6%

Note: CA - Current liability


Current asset CL Profit before interest and tax
INV - Inventory PBIT Dividend per share Capital
PAT - Profit after tax DPS employed
MPS - Market price per share CE

b. Comments on Performance and Short term Liquidity


i. Two ratios which are relevant for measuring the company‟s profitability are the ROCE
and the net profit margin. While the ROCE measures the efficiency or otherwise of
management in the employing or utilizing the resources available, the Net Profit
margin measures management control over operating expenses. The higher the ratios,
the better the performance of the company. For Adebayo Trading Company Plc, the
performance is on the decline as evidenced by the two ratios. The company recorded a
decrease in ROCE from 89.47% in 2017 to 42.69% in 2018. Also the net profit margin
declined from 15.78% in 2017 to 13.25% in 2018. Contributing factors are the
significant reduction in revenue without a reduction in overheads.
ii. Short term liquidity
The current and quick ratios are normally used to measure short term liquidity. The
ratios measure the ability of an entity to meet its short terms obligations as they fall
due. A current ratio of 1.29:1 though below the benchmark is better in 2018 than in
2017 for the quick ratio, the position in 2017 of 0.70:1 declined to 0.57:1 in 2018.
This could be attributed to slow moving inventory resulting in lower sales revenue in
2018.

c. Creative accounting techniques


(i) Window dressing: An entity enters into a transaction just before the year end
and reverses the transaction just after the year end. For example, goods are sold on the
understanding that they will be returned immediately after the year end; this appears to
improve profits and liquidity. The only reason for the transaction is to artificially
improve the view shown by the financial statements.
(ii) “Off Balance Sheet” finance: Transactions are deliberately arranged so as to enable an
entity to keep significant assets and liabilities out of the statement of financial
position. This improves gearing and return on capital employed. Examples include
sales and repurchase agreements and some forms of leasing.
(iii) Changes in accounting policies or accounting estimates: For example, an entity can
revalue assets (change from the cost model to the relevant model) to improved gearing
or change the way in which it depreciates assets to improve profits.
(iv) Profit smoothing: Manipulating reported profits by recognising (usually) artificial
assets or liabilities and releasing them to statement profit or loss as required.
(v) Aggressive earnings management: Artificially improving earnings and
profits
by recognizing sales revenue before it is earned.
(vi) Capitalizing expenses: Recognising “assets” which do not meet the definition in the
IASB conceptual framework for the recognition criteria. Examples include: human
resources, advertising expenditure and internally generated brand names.
EXAMINER‟S REPORT
The question, which is in three parts, tests the candidates‟ knowledge and understanding of
financial ratios and the techniques of creative accounting.
Parts (a) and (b) require candidates to calculate some ratios and to comment on the
performance and short term liquidity of an entity. Part(c) requires candidates to identify and
give examples of creative accounting techniques.
Candidates‟ understanding of the question was good as over 95% of the candidates attempted
the question and the performance was above average.
Candidates‟ commonest pitfalls were their wrong calculations of relevant ratios, inability to
comment properly based on the ratios calculated. Some of them were unable to identify and
explain creative accounting techniques.
Candidates are advised to also pay special attention to the interpretation of the results of the
calculations when attempting question on financial ratio analysis in future examinations of the
Institute.
Marking Guide
a. Calculation of financial ratios
- Return on Capital employed
- Net profit margin
- Current ratio
- Quick ratio Marks Marks
- Debt ratio
1
- Fixed interest cover 2
- Dividend cover
/
- Dividend yield
3
- Columnar presentation
1
b. Comments on Performance and
- Comment on performance
1
12
1
short-term liquidity
21

1
Comment on short term liquidity 2 4

c. Creative Accounting Techniques


-
Identification of 4 - techniques at ^ mark each 2 4
- 2
Explanation of 4 techniques at ^ mark each 2 Total 0

SOLUTION 5
AKAWO LIMITED
2016
N‟000
28,000 2017
Statement of profit or loss (extract) Notes
Revenue 3 (18,200) N‟000
Cost of sales 3 9,800 32,000
Profit for the period (26,800)
5,200

Notes 2016 201


Statement of financial position (extract) N‟000 7
Non-current assets 5.400 N‟00
PPE 5 600 0
Current assets 2.400 3,00
Inventory - Material (WIP) 02,000
4
Contract asset
2017
2016 N‟000
Workings: 40,800
1. Overall N‟000 N‟000 N‟00
profit Cost to 80,000 0
date: 80,00
1,000
Architect‟s and surveyor‟s fee 5,600
0
Materials 7,000
Direct labour costs 2,800
Overheads (40% x N7000) (18,200) 4,200 (45.00
1,800
Depreciation 13,20 0)
29,600 (33,800)
Estimated cost to completion 0 (15.00
4,200 (52,000)
Depreciation 1,800 0)
28,000
Total cost to completion (60.0) 2
Profit
2. Percentage of completion 0,000
2016 201
7
N‟000 N‟000
Cost to date 18,200 45,000
Total cost to completion 52,000 60,000
Percentage
3. 35% Profit 75%
or 201
loss 2016 7
N‟000 N‟00
Revenue 2016 (35% x 80,000) 28,000 0
2017 (75% x 80,000 - 28,000) 32,000

Cost of sales 2016 (35% x N52,000) (18,200)


2017 (75% x N60,000 -18,200) 9,800 (26,800
Profit for the period 2016 )
5,20
0
2017
4. Statement of N‟000 N‟000
financial 18,200 45.00
position 9,800 0
Cost to date (25,600) 15.0 (
Profit to 2.400 58,00
date 2016 0)
Bills N‟000 (2,000
received 7,200 )
Contract (1,800) 2017
asset 5.400 N‟000
5. Property, plant and equipment 7,200
Cost (4,200
Accumulated depreciation (w6) )
Carrying amount = N1,800,000 3,000
Depreciation = N4,200,000
Monthly Depreciation N7,200 - N1,200) = N200,000
30 months
2016
depreciation N 200,000 x 9months
2017
depreciation N200,000 x 21months
EXAMINER‟S REPORT
The question tests candidates‟ knowledge of construction contract account. Candidates were required to
prepare in relation to a building contract, the statement of profit or loss extract for two years 2016 and
2017 and a statement of financial position extract as at the end of the two years.
Candidates appeared to have a poor understanding of the question as only about 40% of the candidates
attempted the question and the performance was below average.
Candidates commonest pitfalls were their inability to determine the revenue and cost elements
attributable to each of the relevant years. Candidates also exhibited poor presentation of the statements
of profit or loss and statement of financial position extracts.
Candidates are advised not to ignore any aspect of the syllabus while preparing for the Institute‟s
examinations in future.

Marking Guide Marks


- Preparation of statementof profit or loss extract 8
- Preparation of statement of financial positionextract 7
Total 15

SOLUTION 6

(a) (i) Market based intangible assets: N‟000


Trade Marks 4,600
Internets 5,400
10,000
Customer‟s related N‟000
(ii) Intangible assets
Ordered or production backlog 4,000
Customers‟ contract 4,400
8,400
(iii) Artistic related intangibles assets N‟000
Books and literary works 800
Video and motion film pictures 2,500
Pictures and photographs 3,400
Plays 4,200
10,900
(iv) Contract based intangibles assets N‟000
Franchise 3,200
Broadcasting right 5,000
8,200
(v) Technology based intangibles assets N‟000
Quick books and SAGE 950
Patents 1,200
Trade Secrets 4,800
6,950
(b) Carrying
amount N’000
(i) Quick book
and SAGE cost 950
Accumulated amortisation
N950/5years x 2years (380)
570
(ii) Trade
marks N’000
Cost 4,600
Accumulated amortization:
N4600/8years x 3years (1,725)
2.875
(iii) Plays
N’000
Cost 4,200
Accumulated amortization:
N4200/7years x 4years (2,400)
1,800
(iv) Franchise
N’000
Cost 3,200
Accumulated amortization:
(c) N3200/8years
IAS x 5years
38 prohibits (2,000) assets:
the following internally generated intangibles
Goodwill Brands 1,200
Mastheads, Front page newspaper
masthead Publishing Titles Customer‟s List
EXAMINER‟S REPORT
The question tests candidates‟ knowledge and understanding of the provisions of IAS 38 Intangibles
assets. The candidates are required to calculate the costs of the following intangibles classes; market
based, customer related, artistic related, contract based and technology based intangible assets. They
also determine the carrying amounts of some items of intangibles and to identify four internally
generated intangibles prohibited by IAS 38.
Candidates had poor understanding of the question. About 50% of the candidates attempted the
question and the performance was below average.
Candidates‟ commonest pitfalls were their inability to calculate the correct costs of the intangible
assets due to wrong classifications and determination of the carrying amounts of some items.
Candidates‟ are advised to make effective use of the Institutes financial reporting study text while
preparing for examination in the future.

Marking Guide Marks Marks


a. Calculation of the costs of intangibles assets:
- Market based
- Customer related
- Artistic related
- Contract based
5
- Technology based
b. Calculation of Carrying amounts of intangible assets:
- Quick Books and SAGE 1/
- Trade marks 11/
- Plays 1/
- Franchise 1/ 6
c. Identification of four Internally Generated Intangibles assets
Prohibited in IAS38:
- Goodwill
- Brand
- Mastheads
- Publishing titles
- Customer‟s list 4
One mark each for any 4 15
points Total
SOLUTION 7

a. LIMITATIONS OF FINANCIAL STATEMENTS


• Complex and detailed information:
Financial statements prepared under IFRS conditions are complex and detailed. Most users
may not be able to fully understand what the financial statements are trying to communicate.
• Timely information:
For decision making, timely information is required, therefore, for many users of financial
statements, the information in it may not be received on time.
• Historical cost:
The financial statements are mainly historical in nature and give a summary of past events
and not what is going to happen in future.

• Not all information is captured:


Much useful information is not captured in the financial statements. For example, internally
generated intangible assets e.g. a brand name cannot be reflected in the statement of financial
position of the reporting entity.
• Creative accounting:
Management may manipulate the information in the financial statements so that it would not
reflect the reality of the entity.
• Financial market perception:
How the financial market perceives an entity cannot be recognised in the financial statement.
• Revenue recognition:
The statement of comprehensive income may be inadequate on reflecting the amount of profit
or loss made in a period because it is difficult to judge at what point revenue should be
recognised.
• Periodic financial statement: are only drawn up at a specific period of time.
• Events after financial year end: A lot can happen between the end of financial period and
when the statements are authorised for publication.
• Related parties transaction: Many transactions take place between related parties and
these may not be fully reflected in the financial statements.
• Mixed value: The values used in financial statements are mixed in nature some
transactions and balances are accounted for at historical cost while others are incorporate at
fair values.
• Inflationary effects: If the inflation rate is relatively high, the amount associated with assets
and liabilities in the statement of financial position will appear low, since they are not being
adjusted for inflation. This mostly apply to non-current assets.
• Incomparability across companies: If users want to compare the results of different
companies, their financial statements are not always comparable, because the entities may use
different accounting policies. These issues can be located by examining the disclosures that
accompany the financial statements.
• No Predictive value: The information in a set of financial statements provides information
either historical results or the financial status of a business as of a specific date. The
statements do not necessarily provide any value in predicting what will happen in the future.
For examples a business could report excellent results in one month, and no sales at all in the
following month, because a contract on which it was relying on has ended.
b. Potential users of financial statements and their likely information need.
• Community - For social responsibility
• Suppliers and trade creditors - are interested in information that will help them determine
whether the amounts owing them will be paid on time.
• Lenders - wants information that will enable them to decide whether their loans will be paid
when due, and whether or not to grant new loans to the entity.
• Investors - supply risk capital in the form of funding. This group is concerned with the risk
inherent and the return provided by their investments.
• Customers - will be interested in the continuity of the entity, especially if they depend on it
themselves.
• Employees - wish to know about the stability and profitability of the entity. This may give
them confidence about their jobs and could be used to discuss salary and conditions of
employment.
• Government/Government Agencies - They are interested in taxation and regulatory
purposes.
• General Public - May be affected by an entity in a number of different ways especially how
an entity may contribute to the local economy.
c. Underlying assumptions of the Financial Statements - The underlying assumption as
enunciated by the conceptual framework for financial reporting is the going_concern basis of
accounting. The going concern of basis of accounting is the assumption in preparing the financial
statements that the entity will continue to operate for the foreseeable future and does not intend to
go into liquidation and will not be forced to go into liquidation.
EXAMINER‟S REPORT
The question tests candidates‟ knowledge of “The Conceptual Framework for Financial Reporting”.
The candidates are required to identify and discuss the limitations of financial statements, identify
potential users of financial statement and their information need and also to state the underlying
assumptions of financial statements.
Although over 80% of the candidates attempted the question, their understanding of it was limited as
the performance was average.
Candidates‟ commonest pitfalls were their inability to explain the information need of the identified
users and lack of knowledge of the conceptual framework.
Candidates are advised to cover all aspect of the Financial Reporting Syllabus while preparing for the
examinations in future.

Marking Guide Marks Marks


a. Limitations of financial statements 4
- Identification of any 8 limitations at 1/2 mark each 4 8
1
- Explanation of any 8 limitations at /2 mark each
b. Users of financial statements
- Identification of any 5 users at 1/2 mark each 21/2
- Explanation of any 5 information need of users at 1/2 21/2
mark each 5

c. Identification of the assumptions -


Explanation of the assumptions 2
Total 15

You might also like